You are on page 1of 135

1|Page

Case 01 - Verizon to Enter Canadian Telecommunications Market

Consulting Firm: Samsung Global Strategy Group (GSG) first round full-time job interview
Case Interview Question: Verizon Communications (NYSE: VZ) is an American multinational telecommunications
conglomerate and a corporate component of the Dow Jones Industrial Average. The company is based at 1095 Avenue of the
Americas in Midtown Manhattan, New York City. Its Verizon Wireless subsidiary is the largest U.S. wireless communications
service provider as of September 2014, with more than 100 million mobile customers.
Wind Mobile Inc. is a Canadian wireless telecommunications provider headquartered in Toronto, Ontario. Founded in 2008,
Wind Mobile was one of several new mobile carriers launched in Canada in 2008 after a government initiative to encourage
competition in the wireless sector. With more than 600,000 active wireless subscribers (as of the end of May 2014) in urban
areas of Ontario, British Columbia and Alberta, it is Canada’s fifth-largest mobile operator.
Your client Bell Canada is a major Canadian telecommunications company headquartered in Montreal, Quebec. It is the
incumbent local exchange carrier for telephone and DSL Internet services in most of Canada east of Saskatchewan and in the
northern territories, and a major competitive local exchange carrier for enterprise customers in the western provinces.
The year is 2014. This past June, Verizon announced that it was seeking to buy Wind Mobile and thus enter the Canadian
market. Your client, the CEO of Bell Canada, has hired you to answer three questions. First, what impact will this have on the
Canadian telecommunications market? Second, how should they respond to ensure profitability in the long term? Finally, what
strategies can we implement aside from our response to Verizon to improve profitability and
reassure our shareholders?

Possible Answers:
1. Case Overview & Suggested Framework
This is a very open-ended case with no forced path to the answer, thus to solve the case, the
candidate must first focus on the impact of Verizon on the market as an analysis of the Canadian
telecommunications market. Next, their insights from this analysis will allow them to develop
strategies to counter Verizon’s entry into the Canadian telecommunications market. Finally, the
last question requires them to set aside Verizon’s move and approach the question as a standard
profitability case.
Additionally, this case tests the candidate’s ability to find insights from large amounts of data as only some of the exhibits are
ultimately relevant to solving the case. This is especially important as there is little information that the interviewer has to tell
the candidate, most of the data is contained within these charts.
2. Information Gathering
• Verizon is the largest telecommunications company in the United States, with around 140 million subscribers.
• Due to their size, Verizon will be able to buy phones at a lower cost from manufacturers.
• In addition, they can offer free roaming between Canada and the United States due to their existing networks in the US.
• All of the four major Canadian telecommunications companies (Bell Canada, Shaw Communications, Rogers
Communications, Telus) offer a full range of services, Wind Mobile solely operates in the wireless phone segment.
• Government regulations do not allow the incumbents to acquire a competitor and foreign companies can only acquire a
Canadian company if the target’s market share is below 10%
• Differences in cost of acquisitions come from two sources: scale (purchasing more handsets allows for purchase discounts)
and effectiveness of promotions (new subscribers per million advertising spend)
• Our research shows Bell Canada’s higher churn mainly results from having a higher proportion of single-service subscriptions,
who leave for package deals at Telus or Rogers.
• Postpaid subscriptions (contracts) have substantially higher revenues than prepaid phones.

2|Page
3. Exhibits

EXHIBIT 1. Canadian Market Breakdown – Number of Wireless Subscribers in Year 2013

* Entire Canadian telecom market has approximately 27.5 million wireless subscriptions.

EXHIBIT 2. Canadian Telecommunications Market – Size and Growth Rates

EXHIBIT 3. Market Penetration Rates (%) by Income Quintiles

EXHIBIT 4. Wireless Industry Profitability Data – Commonly Used Metrics

* ARPU refers to the average revenue per user.


** COA is the cost of acquisitions, which is the total cost (selling, setup and COGS) of acquiring one new wireless subscriber.
*** Churn is the percentage of subscribers that leave the company on a monthly basis.

3|Page
EXHIBIT 5. Wireless Subscribers (millions of subscribers)

EXHIBIT 6. Smartphone Activations for Q1 2013

* Smartphone activations is the sum of new smartphone subscribers and existing subscribers converted to a smartphone
subscription.

4. Detailed Analysis
Exhibit 1: The candidate should notice that the three leading companies form an oligopoly controlling around 90% of the
Canadian market. In addition, Wind Mobile has below a 10% market share, so the government regulations will not prevent
Verizon from acquiring it.
Exhibit 2: Wireless presents the greatest opportunity in the short term as it is already the largest and is growing considerably
faster than the rest of the telecommunications market. Over time, internet and data will become more important parts of the
market while wired voice and private line will become less important.
Since Verizon is entering the wireless segment of the telecommunications market, this move will put considerable pressure on
Bell Canada’s short and long-term profitability.
Exhibit 3: The market is nearly saturated as over 99% of Canadian households have either a wireless or wired connection.
However, there are three important trends to note. First, wireless penetration varies more widely over the distribution of
income. Second, wired connections still outnumber wireless connections, despite their lower share of industry revenue. Finally,
lower income households are far more likely to have only wired or wireless connections, while high income households tend
to have both.
Thus in the low income market, Bell Canada should focus on converting more customers to wireless connections since it earns
a higher revenue per subscription. In the high income market, they should leverage their full range of services to defend against
Verizon’s entry (since they can only offer wireless phone subscriptions).
Exhibit 4: Bell Canada’s ARPU (average revenue per user) is lower than both of the competitors and the cost of acquisitions as
well as churn are both higher than the average. Thus Bell Canada can improve its profitability by raising ARPU, lowering the
cost per acquisition and lowering churn.
If the candidate asks why Bell Canada’s ARPU is lower, ask them to make some suggestions themselves. When they suggest a
different rate of data usage/different breakdown of subscriptions, give them Exhibits 5 and 6.
Exhibit 5: The candidate should check to see if there are significant differences in the proportion of postpaid versus prepaid
subscribers. In this case, the differences are negligible and thus provide no insight on revenue differences.
Wireless Subscribers (millions of subscribers)

4|Page
Exhibit 6: The number of smartphone subscriptions is lower at Bell Canada than at the other two companies. This disparity is
the explanation of Bell Canada’s lower ARPU, as they are not converting low revenue customers to high revenue smartphone
subscriptions at as fast of a rate as their competitors.

5. Conclusion & Recommendation


• Verizon presents a significant threat to the Canadian telecommunications industry as wireless communication is the most
lucrative segment in the near future.
• To mitigate this threat, Bell Canada needs to increase the number of customers on multi-service contracts, which Verizon
cannot yet offer. This will allow help to reduce the costly high churn rate.
• In addition, Bell Canada should focus on converting low income customers to wireless subscriptions and upgrading high
income customers to smartphone subscriptions, which should allow them to reach the same per customer revenue as their
competitors.
• Finally, they should closely monitor their advertising spending to ensure it is used effectively to avoid driving up acquisition
costs.

Case 02 - Ford Kills Competing Models in Fear of Cannibalization


Consulting Firm: A.T. Kearney 2nd round job interview

Case Interview Question: The client Ford Motor Company (NYSE: F) is a multinational automaker based in Dearborn,
a suburb of Detroit, Michigan, United State. Ford is the second largest automaker in the U.S. and the fifth largest in the world
based on annual vehicle sales in 2010. In addition to the Ford and Lincoln brands, Ford Motor Company also owns a stake in
Mazda in Japan and Aston Martin in the UK. Ford’s former UK subsidiaries Jaguar and Land Rover were sold to Tata Motors
of India in March 2008. In 2010 Ford sold Volvo to Geely Automobile and Ford discontinued the Mercury brand after the
2011 model year.
Ford is a major OEM (Original Equipment Manufacturer) auto manufacturer in the US. The company is extremely vertically
integrated. It has a large number of nameplates that it fears are cannibalizing their own products. You have been hired by the
CEO of Ford to come up with a strategic plan to address this issue. What do you propose they do?

Possible Answer:
I proposed that we look at three things:

• The competitive landscape in each of client’s product segments.


• The customer segments and how our client’s vehicles are positioned in these segments.
• Our client’s cost position, and whether or not it can be improved through a portfolio rationalization.
1. Competitive landscape
Candidate: How competitive is each of our client’s product segment? Are we competing against ourselves in some segments?
Interviewer: Yes; competition is intense in each segment. Our individual products compete against our own products and those
of our competitors. However, the relative competition differs for each segment.
As a follow-on comment, I added that I would want to further analyze the segments, in order
to determine what the competition is and how these vehicles are positioned. This could lead to
information regarding which segments we would want to target.
2. Customer segments and product positioning
The first piece of analysis that I did for the customer segments and product positioning was to
draw a number of 2 X 2 matrices with a number of different attributes (speed, price, fuel

5|Page
efficiency, safety, etc.). Once I did this, I hypothesized that there were more than one product in each of the segments. I
highlighted this as a serious problem. I mentioned that this might have happened because through the evolution of the company,
and that it may have been a combination of a number of formerly independent brands.
The main problem with this is that the consumer may be having tremendous difficulty in differentiating the brands. I theorized
that therefore some automobile models might still be around because they always have been, without much thought about
optimizing the portfolio to maximize company profits. Once I analyzed this, I proposed two ways to eliminate models that
overlapped within the same segments:

• kill one of the models/brands, leaving behind the more profitable brand;
• keep both models/brands, but invest in advertising and product improvements such that consumers would be able to
differentiate the two and thus place the second vehicle into a different segment.
I mentioned that this second option might be difficult, as there is significant inertia in changing people’s minds about a brand
that has been a certain type of vehicle for a very long time. I suggested that perhaps the best way to do this would be to keep
the most profitable vehicles in each segment that we are in and eliminate duplicate brands.
3. Costs
The consolidation of models could lead to significant cost savings, as fewer number of models (with greater volume of each)
will lead to greater scale efficiencies in purchasing, production, engineering, advertising, etc.
Recommendations for Client:
Get rid of competing models in the same segment in order to better serve customer segments, reduce costs and reduce
competition. The use of matrices was a hit!

Case 03 - Scotch Whisky Johnnie Walker Develops Marketing Strategy

Consulting Firm: Fidelity Business Consulting (FBC) Group 2nd round job interview
Case Interview Question: Your client Diageo plc (LSE: DGE, NYSE: DEO) is a global alcoholic beverages company
headquartered in London, United Kingdom. It is one of the world’s leading manufacturers of
distilled spirits with a number of major brands. Its popular brands include Smirnoff (claimed
the world’s best selling vodka), Johnnie Walker (claimed the world’s best-selling scotch whisky),
José Cuervo (claimed the world’s best selling tequila), Baileys (claimed the world’s best selling
liqueur) and Guinness (claimed the world’s best selling stout).
You have been hired by the marketing department of Diageo to help them develop marketing
strategy. Diageo’s Chief Marketing Officer has asked you to address two specific questions:
(1) What should the focus of their advertising budget be?
(2) What sized advertising budget would be needed to follow through on your strategy?

Possible Solution:
The first thing that I did here was to separate this case into 2 distinct parts: I first addressed the focus question and then
addressed the budget issue. I used a basic 3 C’s framework to address what the focus of the advertising strategy should be.
Customers:
The first thing that they need to do is determine what their target segment is: What market segment is the client in?
I discovered that they were competing in the premium scotch segment. Once they determine this segment, they can do a
detailed analysis of the demographics of this segment. For arguments sake I assumed that the scotch was Johnnie Walker Black
and the target market was affluent males age 30+. Then they would need to do analysis of advertising media to determine how
to reach this segment, i.e., print-ads in selected magazines, billboards, sponsorship of sporting events (golf, for instance).
Once they had data on what media effectively reaches their target segment, they would decide exactly what mix of media to
use, and the specifics of the campaign to address the qualities that they wanted to associate with their brand (status, taste,

6|Page
distinguished, etc.). Market research could benefit the client in terms of determining what qualities they wanted to stress in their
ad campaigns.
Company:
After I completed the analysis of the potential customers, I looked at the company as well as the industry: Who are the major
players in this industry and where are we in the market?
I found out that currently we were the #3 brand with a little less than 20% market share, the #1 brand had a little higher than
20% and the #2 brand was a little lower than #2. The cost of advertising was not something I got too deeply into. I just noted
that advertising was a significant portion of the cost structure of premium scotch brands (over 25% of sales).
Another point to note is that because of the high margins on this product, the more volume that a brand sold, the more
resources the company would have to invest in advertising. There is a certain minimum scale that is required to be able to
profitably advertise in this market.
Competitors:
The third issue that I addressed was what the competition was doing: How is advertising related to market share? How much
do the competitors advertise?
I learned that last year, the #1 player had spent a lot of money on advertising and increased market share, while the #2 player
spent nothing and lost market share. We spent a little more than nothing and increased market share a little. The distant #4
player also spent nothing on advertising.
Recommendations for Client:
Based on the above analysis, I hypothesized that there was a pretty positive correlation of ad spending and sales volume
increases. Once I outlined what the focus of the advertising strategy should be, I addressed the budget issue. I proposed three
alternatives to determining our advertising budget for next year:
1. The first was to match the #1 competitor to attempt to keep pace.
2. The second was to increase ours a little relative to our historical (last year) ad spending.
3. A third way would be to do a detailed cost analysis of our proposed strategy, and actually quantify what it would cost to have
the number of ads and sponsorships in the types of media that we determined would satisfy our strategy.

Case 04 - Deutsche Post Forecast 20% Drop in Revenue After Deregulation

Consulting Firm: McKinsey & Company 2nd round job interview


Case Interview Question: Our client is Deutsche Post AG (FWB: DPW), one of the world’s largest logistics groups
headquartered in Bonn, Germany. Deutsche Post is the successor to the German mail authority
Deutsche Bundespost, which was privatized in 1995. The Mail division of Deutsche Post
inherits most of the traditional mail services formerly offered by the state-owned monopoly, for
which it uses the Deutsche Post brand.
Historically, they have been the sole provider of mail delivery service in Germany. They have
exclusive right to deliver letters under 50 grams in Germany. A few months ago, however,
following the implementation of European legislation the German government passed a bill to
deregulate the German mail delivery industry. The client is forecasting a 20% drop in revenue,
and they would like to know how to proceed. What would you recommend Deutsche Post to
do?

Possible Answers:
Candidate: I just want to make sure that I understand the problem correctly. Our client, Deutsche Post, is facing deregulation
in its industry. They are anticipating a 20% drop in revenue, and would like our advice on what they should do.
Interviewer: Correct.

7|Page
Candidate: I see. Before I structure my analysis, I would like to ask a few clarifying questions. What does this deregulation
mean?
Interviewer: Traditionally, any company other than Deutsche Post was legislated by law to charge at least 4 times the price of
Deutsche Post for delivering mail within Germany. This means that if Deutsche Post charged $0.25 per letter, then no
competitor is allowed to price their service below $1.00 per letter. This regulation has now been abolished.
Candidate: I think I understand. So an insurmountable barrier to entry has now been removed, and the 20% anticipated drop
in revenue is going to come as a result of competitors entering the industry.
Interviewer: Yes.
Candidate: Great. May I have some time to structure my thoughts? (Note: I would encourage you to do the same, and get some
practice with the upfront structuring of cases.)
Interviewer: Of course.
Candidate: I think I’m ready to begin. In order to understand the problem, I would like to start off by quantifying what the
20% drop in revenue will mean. To do this, I would like to examine the revenues and costs of Deutsche Post first.
Next, I would like to investigate four key areas to see how our client can respond.
To start off, I want to examine the Competition. I want to understand who they are, and how they are able to take business
away from us.
Next, I would like to look at our Customers. I want to have a good understanding of who they are, what they look for, and
the different segments that are present.
Third, I want to take a look at our Product offerings and see if they are in line with what the customers want. In addition, I
also would like to see how our offerings compare to our competitors.
Finally, I want to investigate the Company itself, and understand its core competencies, financial situation, and other factors
such as organizational structure to see what kind of a response our client is able to conjure.
Interviewee’s Note: The structure of my analysis was basically broken up into two major sections. The first section includes
revenues and costs. This section is dedicated to figuring out the quantifiable effects of the revenue drop, and “What is
happening” from a quantitative standpoint. The second section includes customers, competition, company, and products. This
section is dedicated to figuring out “What to do about declining revenues”, as well as gathering qualitative information
Interviewer: Sounds like a reasonable plan. What would you like to know first?
Candidate: To start off, I want to quantify the effect of this 20% drop in revenue. Before I dive in, is it our job to verify this
revenue forecast?
Interviewer: You can assume that it’s accurate.
Candidate: Excellent. May I ask how much revenue our client currently generates?
Interviewer: One billion dollars.
Candidate: I would like to take a closer look at the revenue and break it down into its components. Do we have any information
on that?
Interviewer: What do you think are the components?
Candidate: There are lots of ways to segment it, but I know that revenue is comprised of the average price per unit times the
volume of units sold.
Interviewer: That’s fine. So let’s say that on average, the price per delivery is $1.00
Candidate: So if our revenues are 1 billion dollars, and the price per deliver is one dollar, then our volume must be around a
billion deliveries last year. However averages can be deceiving. Is there a big price variation between certain types of deliveries?
Interviewer: Yes, but for this case you can assume all deliveries are made at one dollar.

8|Page
Candidate: You mentioned that our client is anticipating a 20% drop in revenues, and I am interested in know whether this will
be due to reduced volumes, or reduced pricing. Do we have any information on that?
Interviewer: They are expecting reduced volumes.
Candidate: Interesting. So it seems like the new entrants to the market will most likely be taking business away from us by
reducing our volumes. I think I understand enough regarding the revenues. Their revenues will drop from a billion dollars to
800 million dollars due to a drop in volume that is caused by new competitors. I would now like to move over to the cost side
of the analysis, and see what’s going on there.
Interviewer: Sure, what would you like to know?
Candidate: In the same time period where we generated a billion in revenue, how much were our costs?
Interviewer: 800 million.
My response: Do we have a further breakdown of costs?
Interviewer: How would you like to break them down?
Candidate: Let’s go with fixed and variable.
Interviewer: Sure. The fixed costs are 700 million, and the variable costs are 100 million.
Candidate: Wow, that’s a high fixed cost. Am I right in assuming that the fixed cost will not change at all when our revenue
drop 20%? If this is the case, and variable costs drop proportionally with revenues, then it means that our profits will go from
200 million to 20 million – a drop of 90%.
Interviewer: Your math is correct.
Candidate: I think I understand everything I need to know about revenues and costs. Our numbers before the deregulation was
a billion in revenues, and 800 million in costs. However, due to the extraordinarily high fixed cost, the 20% drop in revenue
will actually result in a 90% drop in profits. This is not good at all for the client. In order to see what we can do about this, I
would like to move on with the upfront structure I laid out, and examine the competition.
Interviewer: Sure.
Interviewee’s Note: Up to this point, I thought that the case was going fairly well. There was a bit of a trap with the high fixed
cost. If I didn’t segment the costs, and just assumed that they would also drop by 20%, then I would’ve missed a big point.
Candidate: The first thing I would like to understand is how these new competitors are going to take market share away from
us. Do we have any information on that?
Interviewer: What do you think?
Candidate: Since we are an established player, it’s probably going to take some sort of incentive for our customers to switch.
My guess would be that the competitors are going to come in with lower pricing.
Interviewer: You’re right.
Candidate: That’s very interesting. The reason I say this is because our client has an enormous fixed cost, and I thought this
would provide a pretty significant barrier to entry for new competitors. Do we have any information on how they are able to
have a lower price?
Interviewee’s Note: At this point, I realized that I probably should have segmented fixed costs further in my previous analysis.
This is where the case started to fall apart for me as it took me a very long time to figure out how the new entrants are able to
achieve a lower cost.
Interviewer: What would you need in order to figure that out?
Candidate: I think I would need to know what our costs are made up of, and compare those segments to that of the competition.
Interviewer: What do you think the fixed costs are? Walk me through it.
Candidate: Intuitively I would say that there are costs associated with collecting the mail from the customers, processing the
mail at a facility, and delivering the mail to the recipient.

9|Page
Interviewer: You are correct. What do you think is the largest cost?
Candidate: Processing the mail at a facility because this requires a large building with lots of sorting equipment.
Interviewer: That’s interesting, but incorrect. Let’s think about this on a per letter basis.
Candidate: Sure. If we’re looking at a per letter basis, I would say that the delivery cost would be the highest? This is because
the mail sorting facility processes so many letters that the cost for each letter is fairly low.
Interviewer: You’re right.
Candidate: OK great! So do we know if the new entrant’s fixed costs are comparable to ours?
Interviewer: They are much lower. In particular, their delivery cost is much lower than ours.
Candidate: Do we know how they are able to achieve this?
Interviewer: What do you think?
Interviewee’s Note: I spun my wheels on this one for about 15 minutes. It turns out that the new entrants focused on mail
delivery to large urban centers. In doing so, the workers can deliver many more units per hour – thus saving the new entrants
significant cost with regards to worker salaries. I will now jump to the next portion.
Candidate: It seems that in order to compete we need to lower our workers salaries. Has management tried this in the past?
Interviewer: The workers are unionized and will not take a salary cut without good reason.
Interviewee’s Note: I ran out of ideas at this point regarding what to do. It turned out that the key was to renegotiate the service
level agreement. Instead of delivering mail six days a week, the new agreement will deliver 4 days a week. At this point we ran
out of time. When I practice, we usually keep the cases to a maximum of 40 minutes to somewhat simulate a real interview.
Since I missed this key insight, the interviewer gave it to me and asked me to conclude.
Candidate: After preliminary analysis, the team has concluded that Deutsche Post should renegotiate the service level agreement
with the unionized workers in order to respond to the competitive pressure put forth by the new entrants – which is resulting
in a 20% drop in revenue. I believe this to be true for two reasons:
1. The new entrant’s competitive advantage is a low fixed cost due to the fact that they only deliver to urban areas where the
workers can maximize their productivity. Since we deliver to all of Germany, we must reduce our fixed costs through lower
worker salaries.
2. The unionized workers are willing to take a lower salary for a reduced workload. By reducing the working days by 1/3, we
will be able to justify the lower salary.
Interviewer: Excellent! You did a great job.
Interviewee’s Note: I believe that I could have cracked this case by doing the following things:
Segmenting fixed costs to see what was the largest portion at the beginning. I did this later on, but it would have helped had I
done it upfront.
Asked some qualitative questions regarding how the company currently operates. This may have uncovered the fact that they
currently deliver six days a week.
The feedback I got from the interviewer was that this case took some business intuition to solve. The options were basically to
increase productivity per mailman or decrease salary per mailman. I feel that the biggest problem with my performance was
that it took way too long. This is an actual case that a candidate received in one of the McKinsey Germany offices. Any feedback
or general comments would be much appreciated.

10 | P a g e
Case 05 - DHL Express to Partner with Competitor Express Ship

Consulting Firm: Bain & Company first round full time job interview
Case Interview Question: Your client DHL Express is a major air freight shipper. Originally founded in 1969 to deliver
documents between San Francisco and Honolulu, the company expanded its service throughout
the world by the late 1970s. Currently they operate in the United States and 60 other countries.

The client DHL Express has significant international air freight shipping market share, but a
much smaller share in the U.S. domestic market. Recently, some larger domestic competitors,
such as FedEx, UPS, Ship USA, and Express Ship, have entered the international market. Our
client DHL Express has had only marginal profits. And with the recent new entrants into the
international air freight shipping market, they are concerned that they may not be able to break
even this year. What should they do?

Additional Information: To be provided to candidate after relevant questions


After the interviewee develops a clear framework and asks appropriate questions, provide them with the following Exhibits.

Exhibit 1. Client’s Market Share in domestic and international market

11 | P a g e
Exhibit 2. Client’s Cost Structure vs. Competitors

Other additional information:


• Our client DHL Express recently reduced their retail prices to compete with lower prices competitor Ship USA. However,
this move did not increase their market share significantly.
• Retail prices provided in the cost structure slide are typical of U.S. rates.
• Competitor Express Ship has lower delivery costs because it has significant market share in the U.S. ground business, which
our client DHL does not compete in.
• International growth is expected over the next 10 years.
• The client DHL has optimized variable costs. There are no major cost savings (direct the interviewee away from this area if
they mention it)

Possible Solution:
Gross Profit calculations:

• Our client DHL: $1.20 – $0.80 – $0.30 – $0.10 = 0


• Ship USA: $1.30 – $0.60 – $0.40 – $0.10 = $0.20
• Express Ship: $1.15 – $0.70 – $0.20 – $0.10 = $0.15

It is important to recognize that our client DHL will not be profitable in the express air business because it does not have
domestic scale (only 15% share), which is still key for the shipping business. Even with growth in the international business
they will have significant difficulty competing, especially as new entrants come in.
Recommendation:
Attempt a merger or partnership with Express Ship. Our client could provide international shipping service and Express Ship
could provide domestic services. The interviewee should address the risks associated with this merger plan.

Case 06 - Treasure Island Communications to Deal with Deregulation


Consulting Firm: McKinsey & Company 2nd round full time job interview
Case Interview Question: Barbados is a sovereign island country in the Lesser Antilles, in the Americas. It is situated in
the western area of the North Atlantic and 100 km east of the Windward Islands and the
Caribbean Sea. Our client is the CEO of Treasure Island Communications (TIC), a
telecommunications company on Barbados Island which has enjoyed a government protected
monopoly for many years. The client TIC provides mobile services as well as fixed-line services
for both intemational and domestic lines.
In about a year’s time, the Barbados government will deregulate the telecommunications
industry and invite other companies to enter the Barbados market. Our client TIC has engaged
McKinsey & Company to discuss ways to deal with this new environment.
If the candidate asks: “Why does the Barbados government want to deregulate the telecommunications industry?”

12 | P a g e
Interviewer input: “Two main reasons: (1) Government collects more revenues in the form of spectrum licensing fees; (2) More
competition will reduce prices which is better for consumers”.

Possible Solution:
Question 1: Our client TIC would like help in drawing up some possible scenarios. What information would you need in order
to flesh out realistic scenarios?
Possible Answer:
The candidate may outline the following high-level issues:
• Market Size / Structure: Given the market size and the number of customers, is the market more likely to remain a natural
monopoly or evolve into a duopoly/oligopoly/monopolistic competition?
• Characteristics of Potential Competitors: What are their capabilities and which segments (mobile, domestic/international
fixed) are they likely to target?
• Entry Barriers: What entry barriers can TIC use or muster?
Natural barriers include high set-up costs, high fixed costs, etc.
Artificial barriers: TIC could lobby government to set higher spectrum-licensing fees etc.
Question 2: Could you tell me more about the financial information on the client TIC that you’d like?
Possible Answer:
The candidate should cover the basics of revenue and cost information. Suggested answers include:
Revenue: How is pricing done for the different service segments? What are the usage volumes? What are the trends across
time?
Costs: What are the set-up costs, recurring fixed costs, and variable costs?
Question 3: I will give you some data on the international fixed-line services.
Latest revenue (year 2015) = $90M
Total costs = $30M (all annual fixed costs)
Assume that with new entrants and competition, prices will drop 50% but usage volumes do not change. What market share
does TIC need to maintain in order to break even?
Possible Answer:
Candidate’s answer should walk through the following logic:
New Market Size = 50% * $90M = $45M
TIC’s break-even revenue = $30M
TIC’s required Market Share = $30M/$45M = 2/3 = 66.7%
Implication: the client TIC can afford to lose only 1/3 market share to competition.
Question 4: Let’s use more realistic assumptions this time. Assume that volume of usage does increase when prices fall 50%
due to competition. Please take a look at the following data and tell me what you see. Then tell me what market share is needed
in order for the client TIC to break even.
International Fixed Lines
Usage Revenues ($) Total User Minutes
2015 (latest) $90M 90 million
2014 $120M 60 million
2013 $120M 60 million
2012 $160M 40 million
Possible Answer:
Candidate should use the information to (1) calculate prices charged in the past and (2) derive price-volume (demand)
relationship.

13 | P a g e
Usage Revenues ($) Total User Minutes Price ($/user min)
2015 (latest) $90M 90 million 1
2014 $120M 60 million 2
2013 $120M 60 million 2
2012 $160M 40 million 4
Based on the data, the candidate may draw a demand curve (Total User Minutes vs. Price curve).
If the candidate tries (too hard) to extrapolate the demand curve to predict volume based on a price of $0.50, guide him/her to
the simpler heuristic that each time price dropped 50%, volume increased 50%.
Therefore:
New Price = $0.50/user min
New Volume = 90 million * (1 + 50%) = 135 million user minutes
New Market Revenue = $0.50 * 135M = $67.5M
Break-even TIC revenue = $30M
Required Market Share = $30M/$67.5M = 44.4%
Implication: With more competition, user volumes will also rise and TIC’s required market share to break-even is not as high
as initially assumed (44.4% vs. 66.7%).

Case 07 - Loblaws Responds to Walmart’s Expansion into Canada

Consulting Firm: Boston Consulting Group (BCG) 2nd round full time job interview

Case Interview Question: Your client Loblaws is the largest discount retailer in Canada, with more than 500 stores
spread throughout the country. Headquartered in Brampton, Ontario, Loblaws stores are
mostly located in Alberta, British Columbia, Ontario and Quebec. Loblaws is a division of
Loblaw Companies Limited, Canada’s largest food distributor.

For several years running, the client Loblaws has surpassed the second largest Canadian retailer
(with ~300 stores) in both relative market share and profitability. However, the largest discount
retailer in the United States, Walmart, has just bought out Loblaws’s competition Woolworth
Canada Inc. and is planning to convert all 300 Woolworth stores to Walmart stores. The CEO
of Loblaws is quite perturbed by this turn of events, and asks you the following questions:
Should I be worried? How should I react? How would you advise the CEO of Loblaws?

Additional Information: to be given to candidate upon request


(1) Company

• A discount retailer sells a large variety of consumer goods at discounted prices, generally carrying everything from grocery to
consumer products, from housewares and home appliances to clothing. Prime examples include Kmart, Woolworth, Target,
and Wal-Mart.
• Loblaws stores have a wide variety of brand names.
• Loblaws uses a franchise model in which each individual store is owned and managed by a franchisee who has invested in the
store and retains part of the profit.
• Loblaws sells about $750 million worth of goods annually.

14 | P a g e
(2) Competition

A. Canadian Competitor

• Loblaws’ stores are located in similar geographic regions as the competition’s. In fact, you might even see a Loblaws store on
one corner, and the competition on the very next corner.
• Loblaws’ stores tend to have a wider variety of brand names, but by and large, the product mix is similar.
• For certain items Loblaws is less expensive, and for others the competition is less expensive, but the average price level is
similar.
• Loblaws has higher profits than the competition on a per-store basis.
• Loblaws’ cost structure isn’t any lower than the competition’s. Its higher per-store profits are due to higher per-store sales.
• Loblaws’ average store size is approximately the same as that of the competition.
• The competitor’s stores are centrally owned by the company while Loblaws are franchised.

B. U.S. Competitor: Walmart

• Walmart owns 4,000 stores in the U.S. and the second-largest competitor in the U.S. owns approximately 1,000 stores.
• Walmart stores average 200,000 square feet, whereas the typical discount retail store is approximately 100,000 square feet.
• Walmart’s sales are approximately $5 billion, whereas the nearest competitor sells about $1 billion worth of merchandise.
• Its cost of goods is approximately 15% less than that of the competition (due to volume of sales giving it clout with suppliers).
• Its prices are on average about 10% lower than those of the competition.
• Walmart has larger selection of products, given the larger average store size.
• Walmart was started by building superstores in rural markets served mainly by mom-and-pop stores and small discount
retailers. Walmart bet that people would be willing to buy from it, and it was right.
• As it grew and developed more clout with suppliers, it began to buy out other discount retailers and convert their stores to
the Walmart format.
• When a store is converted to the Walmart format, it carries the same brands at prices that are on average 10% lower than the
competition’s.
• Walmart decides whether it should physically expand a store it’s just bought out based on several factors, such as the size of
the existing store, local market competition, local real estate costs, and so on (but it doesn’t plan to expand the Canadian stores
beyond their current size).
• Although members of the Canadian business community are certainly familiar with the company because of its United States
success, the Canadian consumer is basically unaware of Walmart’s existence.
• Loblaws and Walmart carry similar products, although the Loblaws stores lean more heavily toward Canadian suppliers.
• Canada has significantly higher labor costs than the U.S., but Loblaws must also cope with the same high labor costs.

Possible Answer:
1. Canadian Competitor

After comparing Loblaws to its Canadian competitors, the interviewee should realize that:

• Loblaws stores are better managed, since the individual store owners have a greater incentive to maximize profit.
• Loblaws’ higher sales are due primarily to a significantly higher level of customer service. The stores are cleaner, more
attractive, better stocked and so on. The company discovered this through a series of customer surveys last year.

2. Walmart

Similarly, after comparing Loblaws to its U.S. competitor Walmart, the interviewee will realize that:

• Walmart might incur higher distribution costs than Loblaws because it will have to ship products from its United States
warehouses up to Canada.

15 | P a g e
• Loblaws has the advantage in distribution costs, since its network spans less geographic area and it gets more products from
Canadian suppliers.

Once the interviewee reaches the above conclusion, the following additional information can be shared if asked for:

• However, since Loblaws continues to get a good deal of products from the United States, the actual advantage to Loblaws is
not great – only about 2% of overall costs.
• Walmart will be able to retain a significant price advantage over Loblaws’ stores: if not 10%, then at least 7%-8%.

3. Conclusions

In the near term, Loblaws might be safe. Its stores have a much stronger brand name in Canada than Walmart’s, and they seem
to be well managed. However, as consumers get used to seeing prices that are consistently 7%-8% less at Walmart, they will
realize that shopping at Walmart means significant savings over the course of the year.

Although some consumers will remain loyal out of habit or because of the high level of service, it is reasonable to expect the
discount shopper to shop where prices are lowest. Moreover, over time Loblaws’ brand-name advantage will erode as Walmart
becomes more familiar to Canadian consumers. Loblaws certainly has to worry about losing significant market share to Walmart
stores in the long term. They should probably do something about it now, before it’s too late.

4. Recommendations to the client Loblaws

• Find ways to cut costs and make the organization more efficient, so it can keep prices low even if its cost of goods is higher.
• Consider instituting something like a customer loyalty program or frequent shopper program, where consumers accumulate
points that entitle them to future discounts on merchandise (might not be that cost-effective, since it would be rewarding a
significant number of shoppers who would have continued to shop there anyway).
• Prepare a marketing or advertising campaign that highlights its high level of service. Perhaps even institute a Loblaws Service
Guarantee that surpasses any guarantees offered by Walmart.
• Consider offering fewer product lines, so that it can consolidate its buying power and negotiate prices with suppliers that are
competitive with Walmart’s. It might lose some customers who want the variety of products that Walmart has, but it may be
able to retain the customer who is buying a limited array of items and is just looking for the best price.

Sample Interview Transcript:

1. Establish understanding of the case


Candidate: So, the client, Loblaws, is facing competition in Canada from a United States competitor. Our task is to evaluate the
extent of the threat and advise the client on a strategy. Before I can advise the CEO I need some more information about the
situation. First of all, I’m not sure I understand what a discount retailer is!
Interviewer: A discount retailer sells a large variety of consumer goods at discounted prices, generally carrying everything from
housewares and appliances to clothing. Kmart, Woolworth, Target, and Wal-Mart are prime examples in the United States.
2. Set up the framework
Candidate: Oh, I see. Then I think it makes sense to structure the problem this way: First, let’s understand the competition in
the Canadian market and how Loblaws has become the market leader. Then let’s look at the United States to understand how
Walmart has achieved its position. At the end, we can merge the two discussions to understand whether Walmart’s strength in
the United States is transferable to the Canadian market.
Interviewer: That sounds fine. Let’s start, then, with the Canadian discount retail market. What would you like to know?
3. Evaluate the case using the framework
Candidate: Are Loblaws’ 500 stores close to the competition’s 300 stores, or do they serve different geographic areas?

16 | P a g e
Interviewer: The stores are located in similar geographic regions. In fact, you might even see a Loblaws store on one corner,
and the competition on the very next corner.
Candidate: Do Loblaws and the competition sell a similar product mix?
Interviewer: Yes. Loblaws stores tend to have a wider variety of brand names, but by and large, the product mix is similar.
Candidate: Are Loblaws’ prices significantly lower than the competition’s?
Interviewer: No. For certain items Loblaws is less expensive, and for others the competition is less expensive, but the average
price level is similar.
Candidate: Is Loblaws more profitable just because it has more stores, or does it have higher profits per store?
Interviewer: It actually has higher profits than the competition on a per-store basis.
Candidate: Well, higher profits could be the result of lower costs or higher revenues. Are the higher per-store profits due to
lower costs than the competition’s or the result of higher per-store sales?
Interviewer: Loblaws’ cost structure isn’t any lower than the competition’s. Its higher per-store profits are due to higher per-
store sales.
Candidate: Is that because it has bigger stores?
Interviewer: No. Loblaws’ average store size is approximately the same as that of the competition.
Candidate: If they’re selling similar products at similar prices in similarly-sized stores in similar locations, why are Loblaws’ per-
store sales higher than the competition’s?
Interviewer: It’s your job to figure that out!
Candidate: Is Loblaws better managed than the competition?
Interviewer: I don’t know that Loblaws as a company is necessarily better managed, but I can tell you that its management
model for individual stores is significantly different.
Candidate: How so?
Interviewer: The competitor’s stores are centrally owned by the company, while Loblaws uses a franchise model in which each
individual store is owned and managed by a franchisee who has invested in the store and retains part of the profit.
Candidate: In that case, I would guess that the Loblaws stores are probably better managed, since the individual storeowners
have a greater incentive to maximize profit.
Interviewer: You are exactly right. It turns out that Loblaws’ higher sales are due primarily to a significantly higher level of
customer service. The stores are cleaner, more attractive, better stocked, and so on. The company discovered this through a
series of customer surveys last year. I think you’ve sufficiently covered the Canadian market—let’s move now to a discussion
of the United States market.
Candidate: How many stores does Walmart own in the United States, and how many does the second-largest discount retailer
own?
Interviewer: Walmart owns 4,000 stores and the second-largest competitor owns approximately 1,000 stores.
Candidate: Are Walmart stores bigger than those of the typical discount retailer in the United States?
Interviewer: Yes. Walmart stores average 200,000 square feet, whereas the typical discount retail store is approximately 100,000
square feet.
Candidate: Those numbers suggest that Walmart should be selling roughly eight times the volume of the nearest United States
competitor!
Interviewer: Close. Walmart’s sales are approximately $5 billion, whereas the nearest competitor sells about $1 billion worth of
merchandise.
Candidate: I would think that sales of that size give Walmart significant clout with suppliers. Does it have a lower cost of goods
than the competition?
Interviewer: In fact, its cost of goods is approximately 15 percent less than that of the competition.

17 | P a g e
Candidate: So it probably has lower prices.
Interviewer: Right again. Its prices are on average about ten percent lower than those of the competition.
Candidate: So it seems that Walmart has been so successful primarily because it has lower prices than its competitors.
Interviewer: That’s partly right. Its success probably also has something to do with a larger selection of products, given the
larger average store size.
Candidate: How did Walmart get so much bigger than the competition?
Interviewer: It started by building superstores in rural markets served mainly by mom-and-pop stores and small discount
retailers. Walmart bet that people would be willing to buy from it, and it was right. As it grew and developed more clout with
suppliers, it began to buy out other discount retailers and convert their stores to the Walmart format.
Candidate: So whenever Walmart buys out a competing store, it also physically expands it?
Interviewer: Not necessarily. Sometimes it does, but when I said it converts it to the Walmart format, I meant that it carries the
same brands at prices that are on average ten percent lower than the competition’s.
Candidate: What criteria does Walmart use in deciding whether it should physically expand a store it’s just bought out?
Interviewer: It depends on a lot of factors, such as the size of the existing store, local market competition, local real estate costs,
and so on, but I don’t think we need to go into that here.
Candidate: Well, I thought it might be relevant in terms of predicting what it will do with the 300 stores that it bought in Canada.
Interviewer: Let’s just assume that it doesn’t plan to expand the Canadian stores beyond their current size.
Candidate: OK. I think I’ve learned enough about Walmart. I’d like to ask a few questions about Walmart’s ability to succeed
in the Canadian market. Does Walmart have a strong brand name in Canada?
Interviewer: No. Although members of the Canadian business community are certainly familiar with the company because of
its United States success, the Canadian consumer is basically unaware of Walmart’s existence.
Candidate: Does Loblaws carry products similar to Walmart’s, or does the Canadian consumer expect different products and
brands than the United States discount retail consumer?
Interviewer: The two companies carry similar products, although the Loblaws stores lean more heavily toward Canadian
suppliers.
Candidate: How much volume does Loblaws actually sell?
Interviewer: About $750 million worth of goods annually.
Candidate: Is there any reason to think that the costs of doing business for Walmart will be higher in the Canadian market?
Interviewer: Can you be more specific?
Candidate: I mean, for example, are labor or leasing costs higher in Canada than in the United States?
Interviewer: Canada does have significantly higher labor costs, and I’m not sure about the costs of leasing space. What are you
driving at?
Candidate: I was thinking that if there were a higher cost of doing business in Canada, perhaps Walmart would have to charge
higher prices than it does in the United States to cover its costs.
Interviewer: That’s probably true, but remember, Loblaws must also cope with the same high labor costs. Can you think of
additional costs incurred by Walmart’s Canadian operations that would not be incurred by Loblaws?
Candidate: Walmart might incur higher distribution costs than Loblaws because it will have to ship product from its United
States warehouses up to Canada.
Interviewer: You are partially right. Loblaws has the advantage in distribution costs, since its network spans less geographic
area and it gets more products from Canadian suppliers. However, since Loblaws continues to get a good deal of product from
the United States, the actual advantage to Loblaws is not great—only about two percent of overall costs.
Candidate: All this suggests that Walmart will be able to retain a significant price advantage over Loblaws stores: if not ten
percent, then at least seven to eight percent.

18 | P a g e
Interviewer: I would agree with that conclusion.
4. Summarize and make recommendations
Candidate: I would tell the CEO the following: In the near term, you might be safe. Your stores have a much stronger brand
name in Canada than Walmart’s, and they seem to be well managed. However, as consumers get used to seeing prices that are
consistently seven to eight percent less at Walmart, they will realize that shopping at Walmart means significant savings over
the course of the year.
Although some consumers will remain loyal out of habit or because of your high level of service, it is reasonable to expect the
discount shopper to shop where prices are lowest. Moreover, over time your brand-name advantage will erode as Walmart
becomes more familiar to Canadian consumers. You certainly have to worry about losing significant share to Walmart stores in
the long term. You should probably do something about it now, before it’s too late.
Interviewer: Can you suggest possible strategies for Loblaws?
Candidate: Maybe it can find ways to cut costs and make the organization more efficient, so it can keep prices low even if its
cost of goods is higher.
Interviewer: Anything else?
Candidate: It might consider instituting something like a frequent shopper program, where consumers accumulate points that
entitle them to future discounts on merchandise.
Interviewer: What might be a potential problem with that?
Candidate: Well, it might not be that cost-effective, since it would be rewarding a significant number of shoppers who would
have continued to shop there anyway.
Interviewer: Any other suggestions?
Candidate: Loblaws might want to prepare a marketing or advertising campaign that highlights its high level of service. It might
even institute a Loblaws Service Guarantee that surpasses any guarantees offered by Walmart.
Interviewer: Assuming the only way to keep customers is through competitive pricing, is there anything Loblaws can do to
appear competitive to the consumer?
Candidate: It might want to consider offering fewer product lines, so that it can consolidate its buying power and negotiate
prices with suppliers that are competitive with Walmart’s. It might lose some customers who want the variety of products that
Walmart has, but it may be able to retain the customer who is buying a limited array of items and is just looking for the best
price.
Interviewer: All of your suggestions are interesting, and you would want to analyze the advantages and disadvantages of each
in more detail before making any recommendations to the CEO.

Case 08 - Bausch & Lomb Responds to Threat of Disposable Lens

Consulting Firm: Bain & Company 2nd round full time job interview
Case Interview Question: Our client Bausch & Lomb is an American contact lens manufacturer. Founded in 1853 in
Rochester, New York, Bausch & Lomb is one of the world’s largest suppliers of eye health
products, including contact lenses, lens care products, medicines and implants for eye diseases.
Today, the company is headquartered in Bridgewater, New Jersey.
The client Bausch & Lomb sells lens cleaning solution, prescription long use contact lens, and
trifocal/bifocal lenses. Recently, global pharmaceutical and medical device giant Johnson &
Johnson (J&J) introduced the first disposable lens (consumers wear for two weeks then throw
away). The J&J lenses do not need to be cleaned and offer a much better user experience. J&J
spent decades developing not only the disposable lens, but also a patented
automated production process. Using their computerized production, J&J can manufacture the
disposable lens for a tenth of the cost of our time equivalent offering. Already J&J has gone from O% to 25% of all new installs

19 | P a g e
(first time contact prescriptions), matching the number of prescriptions for our client Bausch & Lomb’s product only six
months after launching the product.

Show the Candidate the following three charts:


Exhibit 1. Breakdown of Client Revenue by Product Line

Exhibit 2. Gross Margin (GM%) and Global Market Share by Product Line

Exhibit 3. Average Standard Lens Customer Spend (Client vs. J&J)

Interviewer: What do you see? What should our client Bausch & Lomb do?

Possible Answer:
Candidate should ask for time to go over the three charts and build a framework.
Candidate: Well, first a few observations. Our client Bausch & Lomb has a large share of the global bifocal/trifocal market
(70%). While solution and general lens are nice complementary businesses, the money is in bifocal/trifocals. Also, it costs more
to outfit a consumer with J&J lens for a year than it does to use our client’s product.
In order to make solid recommendations to our client, I would like to flesh out three key dimensions of our client’s dilemma.
First, I would like to learn more about J&J’s new products, next I would like to explore how contacts/solution are sold, and
finally I would further map out the contact lens market.
I would like to start with J&J’s product.
Interviewer: Sounds Reasonable.

20 | P a g e
Candidate: Great. Can you tell me how the overall user experience of our product compares to that of J&J?
Interviewer: The product is fabulous. In fact, 20 years later it is still the market leader (30% market share). People love it.
Candidate: Obviously good news for our client. To clarify: the J&J product line includes Bifocal/Trifocal lenses?
Interviewer: Due to the way the J&J lens is produced, it is impossible to create bifocal or trifocal capabilities.
Candidate: That is great news for our client. A final J&J question: can we develop similar technology?
Interviewer: J&J is years ahead in research and development (R&D) and controls all the patents that make the disposable lens
possible. It may be a decade before we can offer a competing product.
Candidate: OK. Let’s shift our attention to the sales channel. I do not wear glasses, but from what I know about contacts a
doctor must write a prescription for a contact lens. Do doctors receive a commission from the sale of lenses or solution? Also,
is the insurance co-pay the same for either product?
Interviewer: The insurance co-pay is $35 for both products. Doctors do receive a commission on the sale of lens and solution,
although most solution is not bought at the doctor’s office.
Candidate: So Doctors are receiving around a 33% larger commission from selling J&J lens than our client’s product? That
might be a problem. We know customers will be happier with the J&J lenses, and now we know that the J&J lenses have the
same out of pocket cost to customers as our client’s lenses and Doctors have a financial incentive to sell J&J lenses. Also, given
the cost differential it means that J&J is making a much larger profit per user than our client.
Interviewer: Right. What do you think can be done about it?
Candidate: In the short term, we might consider raising the price of our lens to ensure Doctors’ financial incentives are not a
problem for our client. We should also explore training our sales force to counter the J&J sales pitch and consider running a
sales contest for Doctors. All three might help stabilize our market share in the short term. However, our client’s conventional
lens business would seem to be a cash cow. It has a good brand, large installed base, and steady profits – but no chance of
growth. Our client should consider selling it while it remains attractive or our client could use the profits from the general lens
business to expand into another area. I think it’s a great time to look at the product market. Can you tell me about any trends
in the contact lens market?
Interviewer: Well, other than disposable lenses, the other big trend is specialty vanity lens. Think cat eyes for Halloween or red
tint for athletes.
Candidate: Interesting. How many competitors are there in the specialty lens market? Also, is there any patented technology
required to enter the market?
Interviewer: The market is highly fragmented but growing. There is no proprietary technology. In fact, our current facilities
could produce most types of specialty lenses today.

Recommendation:
Interviewer: Oh! The CEO just walked into the room. Please give us a recommendation.
Candidate: Our client Bausch & Lomb is facing a major threat to both its general lens business from J&J, as well as its solution
business since consumption of solution is much lower for disposable lenses. Our client has a large established customer base it
should work to protect for as long as possible. To do so, it should raise prices of our lens and increase our client’s sales force’s
interaction with doctors.
Our client should invest cash generated by its general lens business into buying a specialty lens producer, then work to
consolidate its market position. All the while, the client should work to innovate and patent protect in the bifocal/trifocal
market to ensure long term profits. It cannot compete long term with J&J disposable lenses, but it should be able to survive
and grow by focusing on its core bifocal/trifocal business and seizing the specialty lens opportunity.

21 | P a g e
Case 09 - Unilever Should Not Lower Price for Its Dove Soap

Consulting Firm: Cognizant Business Consulting (CBC) first round job interview.
Case Interview Question: Your consulting team has been hired by Unilever (NYSE: UN for Unilever N.V. and NYSE:
UL for Unilever PLC), a multinational corporation that owns many of the world’s consumer
product brands in foods, beverages, cleaning agents and personal care products.

Dove is a personal care brand owned by Unilever. Dove’s product lines include:
antiperspirants/deodorants, body washes, beauty bars, lotions/moisturizers, hair care, and facial
care products. Recently Unilever has noticed that it is losing market share in the soap product
and suspects that its pricing is to blame. The company currently charges $1.20/bar for the Dove
soap as opposed to $1.00/bar for the Safeguard soap charged by major competitor Procter &
Gamble Co. (P&G, NYSE: PG). Should Unilever lower its price to $1.00?
Additional Information:
Market
• Dove soaps are currently selling 15 million bars/year; were selling 20 million bars/year before the brand started losing
market share.
• The soap market is a mature industry (not growing rapidly).
• The marketing department of Unilever believes that lowering its price to $1.00/bar would boost volume back to 20
million bars/year. (How would you test this? Consider a demand analysis using demand instruments.)

Industry
• Unilever has a reputation of producing the highest-quality product on the market and Dove is a highly recognized
brand.
• The soap market is dominated by four main competitors. Currently the client’s market share is 12 percent. The four
competitors (Procter & Gamble, Johnson & Johnson, Henkel, L’Oreal) have market shares of 30, 20, 17 and 10
percent respectively.
• Currently, the client Unilever has the capacity to handle virtually any increase in demand.

Cost Structure
The company cannot specify the overall cost of a unit (except that it is less than $1.00 and greater than $0.80), but it does know
the cost structure to be the following:

• 30 percent labor
• 20 percent inputs
• 20 percent general and administrative
• 20 percent overhead
• 10 percent other
The company is unsure if it has any cost advantage over other competitors, but it clearly enjoys a reputation for the highest-
quality soap products.

Possible Approach:

In this case, a profitability framework would probably work best here. Focus on incremental revenue and cost numbers since
total revenue and total cost numbers are not available.

Competitive Response: Reducing price would probably lead to a price war. Since it is improbable that the client has a cost
advantage, it would lose a price war.

22 | P a g e
1. Incremental Revenue: Using the assumption that demand would go from 15 million units to 20 million units with a $0.20
price decrease, the incremental revenue would be $1 x 20 million – $1.2 x 15 million = $20 million – $18 million = $2 million.
2. Incremental Cost: Since the company has the additional capacity, assume that labor and inputs rise linearly with volume
(variable cost = 50%) and that everything else is fixed (the candidate ought to suggest this assumption). Since cost per unit
ranges from $1.00 to $0.80, possible incremental cost numbers are:
▪ $1.00 leads to $0.50/unit incremental cost, giving rise to $2.5 million.
▪ $0.80 leads to 0.40/unit incremental cost, giving rise to $2.0 million.
3. Incremental Profit: The numbers above give an incremental profit ranging from -0.5 million to 0.
Conclusion: The incremental profit numbers, combined with the probability of a price war, make reducing the price to $1.00
a bad idea. The client should focus instead on quality, brand image, or segmenting the customer base.

Case 10 - How to Win a Price War in a Duopoly Market?

Consulting Firm: AlixPartners first round full-time job interview.


Case Interview Question: Two companies (Your client and its competitor) are the only players in an industry and
produce exactly the same product. Your client is the pioneer in the industry and has controlled
70% of the market for many years. Their competitor (with 30% market share) has always
followed price changes initiated by your client. Recently though, the competitor has aggressively
lowered prices by 15% and has cut into your client’s market share reducing it to 60%. Your
client’s profit margins are only 14%, so they are hesitant to match the price cut, but they are
afraid that they will continue to lose market share if they don’t.
Assume that there is no threat of new competitors entering the market and that there are no
substitute products. All inputs are commodities and are readily available. The end-users are
sophisticated and make their purchasing decisions based mostly on price. How has the
competitor managed to cut prices so dramatically and still make money? And what would you advise your client to do in
response to competitor’s price cut?

Additional Information: (to be provided only if interviewee asks probing questions)


• Industry growth has historically been 5% per year, but has flattened out completely in the last year.
• There are many buyers of the product and the price of this product is a negligible input cost for them.
• Your client and its competitor both are financially strong divisions of larger unrelated companies.
• Raw materials make up 50% of the total cost of producing this product. All other costs are fixed.
• Both your client and its competitor use essentially the same process and have very similar cost structures.
• Capacity can only be modified in large increments and the competitor brought on a new production line 6 months ago.

Possible Answer:
This case is all about capacity utilization with some game theory and defies most frameworks. Trying to apply Porter’s five
forces or an equivalent model will lead to series of “no, that’s not an issue” comments from the interviewer. Don’t get caught
up trying to figure out what the product is – it doesn’t matter in this case! It cannot be differentiated.

Analysis:
• Industry growth has flattened out completely in the last year, so the total market size is constant.
• Market share for your client fell from 70% to 60%, so their production output fell by about 10/70 = ~14%.
• Since 50% of the total cost is variable, 50% must be fixed.
• A 14% drop in volume would therefore equate to a (14% * 50%) = 7% increase in average unit cost for your client (fixed unit
costs increase!).
• Although specific cost data for the competitor is unknown, the increase in volume they have experienced has reduced their
fixed unit cost in much the same way.
• When the competitor increased market share from 30% to 40%, its volume increased by 33%.

23 | P a g e
• If we assume that the competitor has a similar variable cost component of 50%, then their average fixed unit costs would
have gone down by approximately (33% * 50%) = 16.5% as they spread their fixed costs over a larger volume.
Question 1: How has the competitor managed to cut prices so dramatically and still make money?
The competitor had an overcapacity problem and figured that they could make more money with higher volume by cutting
prices. The 16.5% fixed unit cost reduction offset the 15% price cut they incurred and their volume increased 33% so they
came out way ahead.
Question 2: What would you advise your client to do in response to competitor’s price cut?
The rest of the solution is in game theory. Advise your client to match the competitor’s price and follow their price changes to
show them that they cannot undercut your price. Aggressively try to regain customers who were recently lost by offering extra
inducements to try to get back lost market share. Since your client is still the market leader, they have a slight cost advantage
and can fare better in a price war.
Once the competitor is made to understand this, advise your client to incrementally raise prices and make sure the competitor
follows. Since there are many buyers and the product cost is only a small contributor to their overall costs, it is unlikely that the
buyers will respond negatively to the price increases.

24 | P a g e
Case 01 - HSBC Considers Buying a Stock Brokerage Firm in Europe
25 | P a g e
Case 01 - HSBC Considers Buying a Stock Brokerage Firm in Europe
Consulting Firm: ZS Associates first round full-time job interview
Case Interview Question: Your client is HSBC Bank, one of UK’s big five banks.
Headquartered in London, HSBC Bank is one of the four major clearing banks in the United
Kingdom. Their business ranges from the traditional high street roles of personal finance and
commercial banking, to private banking, consumer finance as well as corporate and investment
banking.
The chairman of HSBC Bank has hired your consulting firm. On your first day meeting with
the client, the chairman asks, “Should we acquire a stock brokerage firm on the Continent?”
How would you respond to the client? What would you recommend?

Possible Answers:
Note: There really isn’t any one single right answer for a case question. The interview transcript below is meant only to suggest
the types of issues you might want to talk about with your interviewer. By walking you through a real, live case, we hope that
you won’t get stuck with this kind of case questions in your consulting interview.
Interviewer: You’re working for HSBC Bank, one of the big five banks in UK. The chairman comes to you and says, “I think
we ought to acquire a brokerage firm on the Continent, but I’d like your opinion on that.” What should you say?
(Note: Beware of the overly general question! The interviewer will often start off with something so broad and general that it
is impossible to answer. The interviewer is really trying to see if you will take the bait, or if you will keep your wits about you
and ask for more information.)
Candidate: Well, I’d probably start by asking, “What do you mean? Are you thinking about acquiring a specific firm? Are you
interested in entering a specific country or market? Are you thinking about getting into the stock brokerage business in general?”
Alternately, I might ask the chairman about the fundamental objectives he is trying to accomplish. Is he seeking an entree into
the brokerage business? Is he trying to boost revenues? Does he want to expand in Europe?
(Note: It is usually okay to ask for more information, especially if you have just received the case question. Often, the
interviewer’s response will give you a hint as to what angle to pursue. However, don’t just ask questions for the sake of taking
up time. They should be designed to give you enough information to proceed with the case.)
Interviewer: Well, the chairman says that he wants to explore opportunities generally and that he doesn’t have a particular firm
or target country in mind. He is interested in increasing the bank’s base of operations on the Continent and the brokerage
industry seems to have a lot of smaller, profitable players that may be ripe for acquisition.
Note: The interviewer is directing the candidate towards a general strategy response. Therefore, the key issues will center on
the firm (e.g. its strategic objectives, its strengths) and the opportunity (e.g. macroeconomic issues as well as the identification
and evaluation of specific acquisition candidates).
Candidate: It sounds like a general strategy/acquisition study might be in order. Since the client doesn’t have any specific
markets or acquisition targets in mind, I’d want to start by analyzing both internal and external issues for the client.
Note: By starting with a general statement, you can check in with your interviewer to see if you are on the right track. If you
have missed the boat, some interviewers will gently push you in another direction (but beware, others may let you drown!).
Interviewer: Why don’t you start by telling me some of the key internal issues you’d look at?
Note: The interviewer likes the candidate’s top-level internal/external framework, but is a little wary of the use of acquisition
study. The interviewer wants to make sure the candidate really understands what information he/she needs to identify.
Candidate: On the internal side, I’d want to look at four or five key things:
– the client’s strategic objectives for the acquisition,
– its core strengths and weaknesses,
– its existing business operations and systems, and how those might fit with a potential acquisition,
– the resources it has available for the acquisition.
Note: The candidate has set up a sub-framework to focus his response about the internal issues the client will face. Although
these points don’t follow any standard framework, they do include a mix of strategic and operational issues.

26 | P a g e
Interviewer: Sounds like a plan. Where would you like to start first?
Candidate: Of these issues, I’d start with a look at the client’s strategic objectives for the acquisition. Frankly, the first question
I would ask is whether or not the client HSBC Bank ought to be acquiring a new business in a different market. Is this part of
a broader diversification or internationalization strategy? Is the client just trying to use up some excess cash?
Note: A key challenge in many real-life consulting cases is to accurately define the right question to ask. In this response, the
candidate questions the initial assumption that an acquisition is called for. An alternative approach would be to assume that the
decision to acquire has been made and the consultant has been called in to implement. In this case, the candidate would probably
want to state the assumption and move on to analyze the various internal factors he has outlined.
Interviewer: Good. Go on please.
Candidate: After determining its strategic objectives, I’d like to analyze any strengths and weaknesses that would be relevant to
the acquisition.
– Does the client have a particular set of resources that make an acquisition of one type or another fit better with its existing
organization?
– Perhaps it already has banking affiliates in several other European countries.
– Perhaps it has relationships with a number of banking customers that are active in certain areas on the continent.
– Are there particular service or product offerings that it is seeking to add to its portfolio?
By looking at these issues, I would be attempting to establish some parameters by which to evaluate different investment
options. In particular, I would be looking for areas in which the client may be able to achieve synergies or significant cost
savings.
Note: The candidate works his way through his framework, focusing first on the issues that will yield the greatest insight. He
also offers some examples, in the form of questions, that indicate the types of issues he will be looking for. Finally, he mentions
how he will use this analysis to produce a recommendation for the client.
Interviewer: What other things would you need to know to give the client your recommendation?
Note: Here the interviewer poses a question that could be approached in different ways. The candidate could continue to
explore internal issues, or he could move over to some of the more important external issues.
Candidate: There are still several issues on the internal side that I haven’t yet explored, such as the internal systems issues and
the budget for the acquisition and so on.
However, for the purposes of making a recommendation to the client, some of the external issues are even more important,
and I’d like to talk about some of those now.
Note: It is a good idea to briefly point out instances in which you stray from your initial framework. That way the interviewer
won’t assume that you just lost your way.
Here the candidate interprets the question fairly literally and moves on to issues of primary importance to making a
recommendation to the client.
Candidate: On the external side, I’d want to break this into three basic pieces.
– The first involves taking a macro look at the marketplace for brokerage services across Europe.
– For the second piece I would identify potential acquisition candidates and assess their attractiveness based on the parameters
we had identified.
– After a broad-based look at a number of candidates, the third piece would involve a detailed evaluation of three or four of
the most promising firms.
Note: Although the candidate hasn’t called on a Porter’s Five Forces/3Cs gourmet flavor of consulting framework, he has
identified a logical approach to getting at the answer. In addition, he has helped his presentation by saying that there will be
three pieces to the analysis and clearly stating what those pieces will be.
Interviewer: Let’s use your approach and start with a look at the market for brokerage services in Europe. What would you
look at?
Candidate: Well, Europe consists of a lot of different countries and, I expect, a lot of different rules and regulations on brokerage
services. I think the best way to start would be to analyze the various markets we were thinking about entering and determine
which would offer the most opportunity for an investment by a British bank.

27 | P a g e
To do this, I would want to look at two sets of factors for each country.
(1) First, I’d start with a survey of the overall market situation.
– How big is the country’s GDP?
– What is the structure of its economy?
– How fast is it growing?
– I’d also want to analyze its stock market. Some countries may not even have stock markets, others may have really small
capitalization. There may be regulations about foreign ownership.
(2) Simultaneously, I’d want to look at the brokerage business in each market.
– How is the stock market set up?
– How many firms compete in the brokerage business?
– Who owns them?
– How profitable are they?
– What are the key resources that give one an advantage over others?
Note: Again, the candidate gives a structure to his response. “First….Second….” This helps the listener follow the presentation.
Also, he identifies a number of areas of potential concern: underlying economics, regulatory issues, and competitive rivalry.
Interviewer: How would you go about getting that information?
Note: Consultants are data hounds. A key concern in many interviewers’ minds (especially for MBA candidates) is how
resourceful the candidate will be in gathering information.
Candidate: Well, I’d probably start by seeing if there were any internal resources that might contain some of that information.
It’s possible that somebody within the organization might have researched the same topic for another study or might be familiar
with certain aspects of brokerage markets in Europe.
Beyond that, I’d probably identify resources within each market that provided the kind of information we needed. These might
include the equivalent to the SEC or the authorities in charge of the various stock markets. They might include other banks or
competitors in the industry, or they might include printed resources or databases.
At some point, when it came time to evaluate the various acquisition candidates on the shortlist, I’d think we’d need to utilize
some secondary means of evaluating their performance, such as utilizing customer surveys or speaking with experts in the field.
Note: If there are particular pieces of analysis with which you feel more comfortable than others, you can often jump directly
to those by saying something along the lines of what this candidate just said.
The candidate starts by looking as close to home as possible. He also seems to be aware of many of the favorite resources for
consulting studies. Importantly, he hasn’t suggested any ideas that would be completely unfeasible, such as interviewing the
heads of the SEC equivalents in each country.
Interviewer: Okay. Let’s say you’ve gone out, collected a pile of information about three or four different potential candidates.
How would you finally choose among them?
Note: Here the interviewer is testing to see whether the candidate will be able to do something with all of the analysis he has
offered.
Candidate: Well, I’d want to go back to the initial parameters I developed after looking at the internal factors. In particular, I’d
want to see which acquisition opportunity seemed to offer the best fit with the client’s long-term business strategy and
organization. In particular, I’d want to see evidence that the acquisition would bring some source of competitive strength to
the client, whether it be in terms of market access, or in terms of a set of synergies between the acquisition target and the client
that would allow the client to operate the company more efficiently.
Interviewer: Good. What else?
Candidate: I think I’d also want to assess the various types of risk with the proposed acquisition.
Note: The candidate goes back to the start of his analysis and incorporates the internal factors identified and analyzed early in
the case problem as the relevant considerations in developing a recommendation. In addition, he focuses on several issues that
are critical to any successful acquisition.
Interviewer: Oh? What types of risk would you look at?

28 | P a g e
Note: The interviewer chooses to examine one small piece of the candidate’s answer and see if he really has thought this through
carefully. The interviewer is also testing to see if the candidate can identify a key issue about deregulation.
Candidate: Well, a key risk in Europe would be to think about the impact of European deregulation. Any profits that we had
anticipated earning as a result of the acquisition would have to be considered in light of future stock market deregulation. In
addition, we’d also want to look at other risk categories such as management risk, technology risk, and other market risks.
Note:The candidate does a good job of listing several risks and offers the information the interviewer is seeking about the
effects of deregulation.
Interviewer: Good. Well, it looks like we’ve covered most of the issues I wanted to go through with this case. Now let me ask
you about another situation.
Candidate: OK.
General Summary Comments:
This was clearly a case question about business strategy. In his answer to the question, the candidate identified a few simple
frameworks and structures that helped him touch on a number of issues associated with the problem. He also did a relatively
good job of letting the interviewer know where he was and what was coming next.

Case 02 - Kellogg Seeing Unexpected Jump in Frozen Waffle Sales


Consulting Firm: Huron Consulting Group first round full time job interview
Case Interview Question: Your client, The Kellogg Company (NYSE: K), is an
American food manufacturing company headquartered in Battle Creek, Michigan, United States.
Kellogg’s produces cereal and convenience foods, including cookies, crackers, frozen waffles,
and toaster pastries and markets their products by several well known brands including Eggo,
Corn Flakes, Keebler, and Cheez-It.
Eggo is a brand of frozen waffles owned by the Kellogg Company, and sold in the United States,
Canada and Mexico. Several varieties are available, including homestyle, miniature, cherry,
blueberry, strawberry, vanilla bliss, brown sugar cinnamon, apple cinnamon, buttermilk, and
chocolate chip.
Recently, our consulting firm was hired by Kellogg Company’s senior management. They told us that in the past two quarters,
sales of their frozen waffles rose significantly. Kellogg asked our consulting firm to investigate and find out the reason behind
their unexpected jump in frozen waffles sales. How would you go about it?

Possible Answers:
Candidate: Frozen waffles? I don’t know a lot about breakfast food marketing, but that doesn’t seem like a product that would
suddenly experience a big jump in sales.
Interviewer: That’s right. That’s why they asked us to look into the matter. What would you do?
Candidate: Can I take a moment to structure my thought?
Interviewer: Sure, take your time.
Note: Fortunately, you’ve done your research on how to structure cases. You think the 4P’s model might be a good way to
structure this particular case. The 4Ps of marketing, as you recall (without looking at your note pad), are Price, Position,
Promotion and Product. You then tell the interviewer how you’re going to proceed with your analysis.
Candidate: Let’s break down the situation like this. First, I’m going to try to find out about the market position of the frozen
waffles. Then, I’m going to try to figure out how the frozen waffles are being promoted. After that, I’ll ask whether there are
any price-based reasons for the surge in waffle sales. Finally, I’ll ask about the frozen waffles themselves.
Interviewer: That seems like a reasonable way to proceed.
Candidate: Okay. What is the overall market for frozen waffles? Has it grown recently?
Interviewer: Not really. Demand for frozen waffles seems to be holding steady.
Candidate: That means we’re taking our waffle sales from other frozen waffle manufacturers.

29 | P a g e
Interviewer: Yes, it seems so.
Candidate: Is there a seasonal reason for this increase in frozen waffle consumption? Perhaps people eat more waffles when
it’s cold?
Interviewer: No, our frozen waffles sales are normally steady year-round.
Candidate: Have sales of other breakfast foods or our own breakfast products increased as well?
Interviewer: No, they haven’t. There seems to be no uptick in the breakfast food market.
Candidate: Has our client Kellogg entered any new markets recently? Perhaps they’ve entered the Asian market? I’m assuming
that these frozen waffles are only sold in North American markets.
Interviewer: Your assumption is correct, and that is a good question. In fact, the company does plan to expand into the
European and Asian markets. That’s why they want to know the reason behind the sales upsurge – so they can replicate it. But
for the time being, no, they haven’t expanded the sales area.
Note: You’re getting a little frustrated, but so far, the interviewer hasn’t indicated that your reasoning is unsound.
Candidate: Okay, let’s talk about promotion and price then. Has our client lowered prices recently?
Interviewer: No, prices are stable.
Candidate: Have other frozen waffle manufacturers raised their prices?
Interviewer: Nope.
Candidate: Is our product a premium product – have we raised prices recently?
Interviewer: No. There is little price differential between our frozen waffles and competing waffle products. And prices have
not varied in the past two quarters. In fact, our waffles are slightly more expensive than some waffles and a bit less than others.
Note: You get the feeling that the interviewer is telling you to back off the price issue.
Candidate: Then maybe there’s some kind of promotion going on. Has the client company started advertising its frozen waffles
recently?
Interviewer: No.
Candidate: I see. Is the client promoting a related product, like waffle syrup?
Interviewer: Nope.
Candidate: Maybe the price of all syrup has gone down recently?
Interviewer: That would affect the consumption of all frozen waffles, not just our client’s.
Candidate: Is there a celebrity who’s recently become famous for eating our frozen waffles?
Interviewer: (laughs…) That’s not it.
Candidate: Then tell me about our client’s promotion process.
Interviewer: Our client Kellogg spends about USD $20 million yearly on frozen waffle promotion. Every month, we run
advertisements in three selected magazines. We also have coupon promotions and a commercial that runs on TV.
Candidate: Have we started to put our commercials on more popular TV programs? Has the quality of our advertising
improved?
Interviewer: Not really. We haven’t significantly changed our promotion campaign. The ratings of the shows we advertise on
are about the same. We have noticed more use of our coupons, however.
Candidate: Has the demographic of our advertising changed? I mean, is the client now pitching to people who are more likely
to eat frozen waffles?
Interviewer: As far as we can tell, the makeup and audience of our advertising targets hasn’t changed.
Note: At this point, you’re determined to uncover the reason behind this waffle case – and you don’t have that much time left.
Candidate: Then perhaps something has changed about our product. Has the shape of the frozen waffles changed?

30 | P a g e
Interviewer: Well, it has changed a bit.
Candidate: How so?
Interviewer: It’s flattened out somewhat.
Candidate: Why? Has the production process changed?
Interviewer: Yes, it has. We’re using a new template for the waffles as part of a general factory modernization.
Candidate: Are the ingredients the same?
Interviewer: Those have changed too. We’re using a new supplier.
Candidate: Then, I would venture to guess that consumers in North America prefer a flatter, larger waffle. There must be
something about the new production process that has created a better frozen waffle.
Interviewer: Yes, that’s correct. Our focus groups show that the texture and crispiness of the waffle has improved. Consumers
are responding to our inadvertently improved waffle product.
Candidate: Then, I recommend that we investigate whether European and Asian markets would respond in a similar manner
to our new waffle product. We should also promote our waffle advantage in the market. Perhaps we could enter new markets
for frozen waffles, like using them for dinner, as crepes.
Interviewer: Your suggestions are very interesting, and we’ll look at them later. I think you did a good job! Do you have any
questions for me?

Case 03 - GSK to Assess Malaria Vaccine Market in South Africa


Consulting Firm: ClearView Healthcare Partners first round full time job interview.

Case Interview Question: South Africa is an emerging market country in Africa. It is the 25th-largest country in the
world by land area and, with close to 60 million people, is the world’s 24th-most populous
nation as of 2018. The World Bank classifies South Africa as an upper-middle-income economy,
and a newly industrialized country. Its economy is the second largest in Africa, and the 34th-
largest in the world.
Our client GlaxoSmithKline plc (GSK) is a global pharmaceutical company headquartered in
Brentford, London, United Kingdom. Established in 2000 by a merger of Glaxo Wellcome and
SmithKline Beecham, GSK is the world’s sixth largest pharmaceutical company as of 2018, after
Pfizer, Novartis, Merck, Hoffmann-La Roche and Sanofi.
The government of South Africa has recently approached our client GSK to apply to
manufacture a malaria vaccine for their citizens. To win the bid, the government of South Africa requires that the vaccine be
manufactured in their country. The government will not decide which pharmaceutical company wins the contract until after all
bidders have an in-country plant. Should our client GSK make the investment to build a manufacturing plant in South Africa?

Additional Information: (provide only if corresponding questions are asked)


• What is the primary objective for the client GSK?
– To maximize profit, though other considerations may be important.
• What other products does our client have? / Is the vaccine our client’s only product?
– Currently GSK’s major product is an older product with declining sales in developed markets.
• Who are our client GSK’s competitors for this bid?
– GSK has two primary competitors for this particular bid:
(a) Competitor A’s malaria vaccine product is a new innovation, which costs more to produce and is currently of lower quality
than GSK’s product.

31 | P a g e
(b) Competitor B’s malaria vaccine product is similar in product lifecycle, price, manufacturing cost, etc. to GSK’s product.
Competitor B also has comparable size, scale and brand recognition.

Possible Answers:
1. Suggested Framework / Structure
A good framework for this case could include:
I. Investment decision
• Revenues
– Number eligible for vaccine, market penetration, any potential substitutes, growth of customer base.
– Price to customers.
• Costs
– Vaccine production – raw materials, labor, etc.
• Plant investment cost
II. Likelihood of winning bid
• Selection criteria
• Competitors
– Likelihood to enter bid
– Performance against criteria
• GSK performance against criteria
III. GSK Capabilities
• International operating experience?
• Financial situation – is GSK able to take on risk of project and up-front investment?
• Company brand.
2. Detailed Analysis
Question 1: What would you advise the client GSK in an absence of competition (certainty of winning bid)
Possible Answer:
The candidate should immediately recognize that this is a net present value (NPV) analysis – prompt him/her if not.
Additional Information to be provided (if asked):
• One-time plant investment: USD $50M
• South Africa’s population: 60 million, assume zero population growth.
– Assume life expectancy of 60 years and the population is evenly distributed across age groups.
– Malaria vaccine is given one time to children at age 2; requires 3 doses for immunity.
• Guaranteed price: $4/dose
• All-in cost to produce: $1/dose
• Assuming the client GSK gets the contract, they are guaranteed a lifetime patent.
• The client company uses a 10% discount rate in perpetuity.
Calculation:
• 60M total population across 60-year life expectancy = 1M children age 2 receiving vaccines each year.
• Revenue = 1M vaccines * 3 doses / vaccine * $4/dose = $12M revenue per year
• Cost = 1M vaccines * 3 doses/ vaccine * $1/dose = $3M cost per year
• Profit = $12M – $3M = $9M per year
• Perpetuity value = $9/10% = $90M
• NPV = $90M – $50M one-time investment = $40M
Conclusion: If the client GSK is guaranteed to receive the contract, they’re expected to have a positive NPV of $40M for this
project.
Question 2: Given competitive landscape, is the investment still attractive?

32 | P a g e
Possible Answer:
Additional Information:
The interviewer should share that the South African government will make a decision based on
(1) the bidder’s ability to meet the $4 per dose price guidance, and
(2) quality of malaria vaccine product.
After revelation of this information, the candidate should recognize that competitor A’s higher-cost, lower-quality product is
not competitive in the bidding process.
Therefore, the candidate should focus on competitor B – provide the information that competitor B has just announced the
intention to bid and, because they have similar quality product and economics to the client GSK, they have a 50% chance of
winning the bid.
At this point, the candidate should realize that GSK’s NPV is now negative, given the $50M required investment and 50%
chance of winning the bid.
• $90M value * 50% chance of winning = $45M expected value
• NPV = $45M – $50M investment cost = -$5M
This can be set up as a prisoner’s dilemma!
Additional Insights:
• If the client GSK can manage their costs, they can make the investment decision more attractive.
– The candidate should recognize and explain this; the interviewer can then share that it is not possible for the client to achieve
a better NPV situation through managing costs.
• Or, if there is a way to see whether Competitor B is going to make a bid before committing, the client can act accordingly.
The candidate could communicate NPV for each player in the different bid scenarios using a 2×2 matrix:
Client GSK
Bid Not bid
Competitor B Bid GSK: -$5M, B: -$5M GSK: 0, B: $40M
Not bid GSK: $40M, B: 0 GSK: 0, B: 0
Question 3: What recommendation should we give GSK?
Possible Answer:
The candidate can provide any recommendation as long as it is backed up with data and logic.
Sample recommendation:
• The client GSK should not enter the market of South African and should not bid.
– With a 50% chance of winning bid, the contract will have a negative NPV of -$5M.
– Only if competitor B chose not to bid, NPV will be $40M with an investment of $50M.
Risks:
– Not entering a bid allows competitor to lock in the full $40M NPV.
– If competitor B also chooses not to bid, profit is left on the table.
– GSK will lose opportunity to enter a new emerging market, potentially expand to other products – this value is not included
in NPV analysis.
Next steps:
– The client should explore risks further, particularly strategic importance of this market.
– This malaria vaccine seems to have value; the client may consider looking for similar emerging markets while Competitor B
is working on this bid.

33 | P a g e
Case 04 - Global Beer Maker Anheuser-Busch to Enter Brazil
Consulting Firm: Huron Consulting Group first round full time job interview

Case Interview Question: Your client is Anheuser-Busch InBev (Euronext: ABI, NYSE: BUD), the world’s largest beer-
maker, marketing brands such as Budweiser, Bud Light, Corona, Stella Artois, Alexander Keith’s
and Labatt. With its global headquarters in Leuven, Belgium, the company is one of the largest
fast-moving consumer goods (FMCG) companies in the world.
Over the past 10 years, the client Anheuser-Busch InBev has managed to increase their market
share in North America through a combination of new brands and acquisitions of their
competitors. However, the market has become saturated and they have started to look for a new
market to enter. As a result, they have hired our consulting firm to identify the best international
expansion opportunity and develop a strategy to enter that market.

Possible Answers:
This is an archetypal market entry case: the candidate needs to identify what would make a market attractive for the client to
enter, then perform some analysis to identify the most profitable market. The majority of the case is driven by the information
given on the two exhibits, so there is limited additional information for the interviewer to give. However, they should make
sure that the candidate is driving towards a solution when looking at the charts, rather than just discussing the information and
not getting anywhere productive.
2. Information Gathering
Additional Information: only give to candidates if requested
• At the moment, the client Anheuser-Busch InBev is only looking to enter a single market and is concerned about both
shortterm profits and long-term profitability.
• Assume that the client Anheuser-Busch InBev does want to enter an international market (e.g., this is not a go/no-go decision,
just which country)
• Assume that the client has the financial resources to support any expansion effort that we recommend they take.
• Brazil has 200M potential customers, Nigeria has 250M and China has 1.4B.
• Based on our market research, we expect the client to achieve a market share of 25% in Brazil, 40% in Nigeria, or 5% in
China, depending on which market they enter.
• The average Brazilian or Nigerian customer will buy 100 units of beer per year; the average Chinese customer will buy 50 units
of beer per year.
• Our client Anheuser-Busch InBev plans to allocate $2B to cover the fixed costs of market entry and operating in the chosen
market (e.g., marketing, set-up of factories and other facilities, and the overhead on their operations); this amount will be the
same irrespective of what market they enter.
EXHIBIT 1. Global Beer Market – Country Map

Note: Prices are adjusted for currency differences

34 | P a g e
EXHIBIT 2. Financial Viability of Chosen Short-Listed Markets

A. Estimated Rvenues for Client Operations in Each Market

B. Estimated Costs for Client Operations in Each Market

3. Detailed Analysis

35 | P a g e
The candidate should start by laying out all of the different criteria that they will use to determine if a particular market is
attractive to the client. This line of analysis should very quickly lead them to Exhibit 1.
There are three important pieces of information that the candidate can pull out of this chart. The first two are the obvious ones,
since they are the axes of the graph: the five-year CAGR and average price level. The other one is the approximate size of the
market, which can be proxied by the size of the country as a whole. Thus generally speaking, the larger bubbles correspond to
larger markets.
For a candidate who has not previously seen a bubble chart, this insight might require some prompting from the interviewer in
terms of what the size of a bubble might mean. An astute candidate will notice that any information about costs or profit
margins is missing from this graph. However, encourage them to initially focus on the markets that offer the greatest revenue
potential and choose the best three. This should cause them to identify Brazil, Nigeria and China as the best markets. All three
of these are large markets (over 100M people); Brazil and Nigeria have above average prices; and China and Nigeria have high
growth rates. They may lean towards the United Kingdom because it has the highest prices, but the size and growth rates of
the market are just too low to represent a significant revenue opportunity.
After identifying these three markets, give them Exhibit 2. Based on these figures, as well as the information listed above, they
should calculate the annual profits for each region, as shown below:
Market Brazil Nigeria China
Price $2.75 $2.25 $1.25
Variable Cost $1.75 $1.85 $1.05
Gross Margin $1.00 $0.40 $0.20
Market Size (#) 200M 250M 1400M
Market Share (%) 25% 40% 5%
Market Share (#) 50M 100M 70M
Purchases/Year (#) 100 100 50
Total Sales (#) 5,000M 10,000M 3,500M
Gross Margin ($) $5,000M $4,000M $700M
Fixed Costs ($) $2,000M $2,000M $2,000M
Profits ($) $3,000M $2,000M -$1,300M
Conclusion: Based on the above calculations, they should see that Brazil offers the best short-term profits.
Next, ask about what risks the client may face moving into this market.
Some potential areas that they could bring up are: political risks, social risks (e.g., social unrest), operational risk, etc. A good
candidate should identify three or more areas of risk and give a sense of how pressing they are.
Finally, ask them to decide which market to recommend.
Although Brazil offers the best short-term profits, Nigeria is also growing faster. Thus they can recommend either as long as
they justify why they are making their choice.
4. Conclusion & Recommendation
• To expand their business internationally, the client should move into Brazil as it offers $3B in annual profits immediately after
entering.
• Although it is growing more slowly than Nigeria, it will take several years for Nigeria to catch up, by which time our client
may be ready for another market entry.
• The most significant risk our client may face is operational risk when opening several new large factories in the country, but
their depth of experience should allow them to mitigate this risk.

36 | P a g e
Case 05 - Should You Enter Health Insurance Market in India?
Consulting Firm: Ernst & Young first round full time job interview.

Case Interview Question: A company wants to enter the health insurance market in India. What do you think? Should
they enter or not?
Possible Answer:
Note: The interviewer intentionally gave a very vague case introduction. The candidate was
supposed to ask clarifying questions in the beginning.
Candidate: First of all, I would like to clarify a few things before I start analyzing the case. Is
that OK with you?
Interviewer: Sure.
Candidate: Is it an India-based company or a foreign player?
Interviewer: It is a foreign company which wants to enter India.
Candidate: I would first like to analyze the current business of the client company and its market entry targets and expectations.
Can you tell me the core business of the company?
Interviewer: I do not want you to go into the details. I want to know the structure that you will follow to analyze the situation.
Candidate: Sure. Kindly give me a minute.
Interviewer: Sure.
Candidate: The structure that I would use to gauge whether the company should enter health insurance market in India is as
follows:
1. Company
Current business
New business targets
Expection
2. Customers
Current market segment
Margins
Distribution system
3. Competitions
Competitors
New products
New entrants
Interviewer: The structure is absolutely fine. Now let me tell you that there aren’t too many international companies in the
health insurance sector in India. Can you tell me why?
Candidate: To be honest I do not know much about this industry but let me think about it. In an industry like health insurance,
there could be either regulatory issues or profitability issues.
Interviewer: Yes, I would like you to focus on profitability issues.
Candidate: Now let’s look at the costs incurred. The different types of costs incurred are claim costs (incurred if claims are
raised by policy holders), distribution costs and fixed costs (in operating the firm). Am I right in naming these costs?
Interviewer: Kindly restrict yourself to claim costs for now.
Candidate: Claim costs could be of two kinds – Administrative costs and treatment costs.
Interviewer: Good. Go on.
Candidate: Admin costs would be high if we have to check on every claim made.
Interviewer: So how would we counter this?

37 | P a g e
Candidate: We can counter this by having an incentive for lesser number of claims made or we can have a minimum claim
amount to avoid admin costs of small claim amounts.
Interviewer: Ok. What about treatment costs?
Candidate: Treatment costs would be large if a lot of claims are made. Moreover, for genuine claims, treatments costs can be
quite high when the illness is severe.
Interviewer: Good. So can you suggest some ways to reduce these costs?
Candidate: According to me, we can do three things: Firstly, we can develop benchmarks which would give us an idea about
the treatment costs. Secondly, we can give doctors an equity share in the company to incentivize them to bill treatment costs
on the company (on behalf of the patient) only to a reasonable extent. Lastly, we can have doctors directly on our payrolls and
these would be the doctors who would serve patients in the hospital.
Interviewer: Fine. Thank you for your ideas and suggestions.

Case 06 - Deutsche Telekom to Bid for 5G License in UK


Consulting Firm: Siemens Management Consulting (SMC) first round full time job interview.

Case Interview Question: Our client Deutsche Telekom AG (FWB: DTE) (English: German Telecom) is a large
German telecommunications company headquartered in Bonn. Deutsche Telekom was formed
in 1996 as the former state-owned monopoly Deutsche Bundespost was privatized. Deutsche
Telekom also holds substantial shares in other telecom companies in Europe.
Recently, the client Deutsche Telekom is considering making a bid for one of 5 licenses to
operate a new generation of mobile phone network (5G) in the United Kingdom. 5G (5th
generation mobile networks or 5th generation wireless systems) denotes the next major phase
of mobile telecommunications standards beyond the current 4G/IMT-Advanced standards.
Deutsche Telekom has engaged Siemens Management Consulting to help with the bidding issue
and to determine the appropriate strategy for the auction. Should the client bid for the first 5G
license in UK? If yes, how much should the client bid?

Additional Information:
a. If the candidate asks for more information on the auction, provide the following info:
The auction will be a sealed bid auction with all bids received simultaneously. The licenses will go to the highest 5 bidders. Each
bidder can attain only one license. The British government intends for the auction to open the mobile phone market to
competition to help control consumer prices. As a result, one license has been reserved for a new entrant, but the other four
are open to both new entrants as well as established competitors. Because it is a sealed bid auction, our client will not know the
true bidding strategies of its competitors with any certainty. The auction is the first of its kind for 5G networks in Europe. The
licenses will be valid for 5 years.
b. If the candidate asks for more information on the technology, provide the following info:
The most important characteristics of the new 5G network are the high data speeds and increased capacity for service that it
offers. 5G networks will enable data speeds of up to five times greater than the most advanced 4G networks available today.
These high data speeds could enable applications such as speedy access to corporate networks ond the Internet, video-on-
demand, online shopping, and video conferencing, all via a mobile device. More importantly, however, the additional 5G
spectrum enables incumbent wireless companies to build out their capacity in order to support more customers and to enable
high-bandwidth services in an environment that is increasingly becoming capacity-constrained. Following the introduction of
5G technology, old technology networks will be phased out by the government over the span of 3 years.
Let the candidate drive the case. The candidate should, at this time, draw out a framework. Here is some of the information
that a candidate may ask for. Provide this only if the candidate asks.
c. More about our client Deutsche Telekom
• Deutsche Telekom is the clear market leader in Germany.
• Has large presence in the rest of Europe, but no presence in the United Kingdom currently.

38 | P a g e
• Has been experiencing stagnating growth.
• Looking for opportunities to expand.
• Has the financial capability to bid what is needed, but does not want to overpay for license.
• Has the financial capability to build network that will be needed to operate in UK.
d. The competition
• It’s unclear how many competitive bids will be received or who will bid.
• 4 major wireless network operators exist in the current UK market.
– They roughly split the market equally.
– They already have established networks as well as retail outlets that they can leverage for the introduction of 5G technology.
e. The market
• Approximately 30 million people have a mobile phone in the UK.
• Growth has been slow recently (3%-5% per year).
• Converting to the new 5G network will require the purchase of a 5G network-enabled mobile device.
• Customers pay $30 per month for their current mobile phone plans.

Possible Answer:
1. Suggest Framework
The candidate should approach this “market entry” case in two sections. First, the candidate must assess if the market is
attractive enough for our client to enter. To do this, they must assess the profitability when considering entry into this market.
Profit = Revenue – Cost = (Volume * Price) – (Fixed Cost + Variable Cost)
Second, after assessing the profitability, the candidate should come to a conclusion about the attractiveness of market entry. If
the candidate concludes that it is a profitable venture for our client, he or she must next develop a methodology for the bid
recommendation.
2. Analysis
Do not provide the candidate with any information about the framework above. Rather, push him/her to make assumptions
about each factor. Some of these qualitative considerations should include:
a. Volume
• Some potential for growth beyond current market due to new services.
• Must consider that conversion of customers will be critical and that client is at a disadvantage compared to established
competitors.
• A likely outcome is that our client will receive less than 1/5 of the total customers due to new entrant status.
b. Price
• Likely able to charge more than what customers currently pay.
– Could charge additional fees for additional services and allow customer to pick and choose what they want.
– Could charge more for a fixed plan with all services included.
• However, there will also be increased competition in the market.
• Candidate should consider both the factors that will increase the price (new services) and decrease the price (increased
competition).
c. Fixed Cost
• Significant up-front cost for our client as they must sec up network and retail chains.
• Up-front costs not as significant for established competitors. This is a key disadvantage to our client.
• Once network is established, fixed costs consist of:
– Network operation
– Maintenance
– Retail operations, etc.
d. Variable Cost
• Minor once network is established (i.e. one extra customer costs very little)
If the candidate struggles to provide quantitative estimates of the above issues, provide the following data:

39 | P a g e
a. Volume
• Increase of 33% in customers (to 40M) overall
• Client will receive 1/8 of market annually (5M total)
b. Price
• Average prices will increase to $50 due to new service adoption among key cusromer segments.
c. Fixed costs
• $740M to build network
• $50M to run network annually
d. Variable costs
• $50M to run network annually
At this point, instruct the candidate not to worry about discounting future cash flows (or that the discount rate is 0%).
After assessing the profitability, the candidate should come to a conclusion about the attractiveness of market entry. If the
candidate concludes that it is a profitable venture for our client, he or she must next develop a methodology for the bid
recommendation. Points to consider when making the bid recommendation include:
• Competitors’ strategies
– Established competitors willing to pay far more as their fixed costs are significantly lower.
– It is unlikely that our client would be willing to outbid the established competitors as a result.
• The client’s real competition comes from other new entrants for the license reserved for them.
• Our client’s bid is based on our assessment of the profitability
– The bid could be as high as the total value of the cash flows from the license (i.e. at break-even)
– However, the candidate should consider if the bid should be lower to realize some profits from the venture.
If the candidate uses the above assumptions, their calculations should appear as the following:
Year 1 Year 2 Year 3 Year 4 Year 5
Price per Customer $50 $50 $50 $50 $50
Volume of Customers 5M 5M 5M 5M 5M
Revenues $250M $250M $250M 250M $250M
Total Revenue: $1,250M
Fixed Costs:
Build Network $740M 0 0 0 0
Run Network $50M $50M $50M $50M $50M
Variable Costs: $50M $50M $50M $50M $50M
Total Costs: $1,240M
Profit (no discounting) $10M
As a result, the bid should be no higher than $10M.
3. Recommendation
The client has asked us for our recommendation. What would you tell the client?
Here is a possible response:
The 5G mobile network in the United Kingdom is a profitable market for our client. Although our client will face significant
competition from established players, I believe they will be able to operate profitably over the life of the license. As a result,
our client should enter a bid in the auction. Based on my calculations, the most our client should bid is $10M to breakeven on
the license. However, there are considerable risks surrounding this bid that should also be considered.
4. Additional Considerations
What are some of the potential risks and potential upsides?
Here are some potential answers for risks:

40 | P a g e
• Customer adoption of new services may not be strong.
• Competitive response is unclear and could drive prices down.
• Estimates are unclear at this point and could swing the bid either way very easily.
Here are some potential answers for upsides:
• May be able to attain a greater share of the market with good market entry strategy.
• Entry into market could provide platform for continued presence in UK.
• Winning one of the first 5G license in Europe might be important and set the tone for the remaining auctions.

Case 07 - Procter & Gamble to Take Its Deodorant Products Global


Consulting Firm: Boston Consulting Group (BCG) first round full time job interview.

Case Interview Question: Procter & Gamble Co. (NYSE: PG) is an American consumer products conglomerate
headquartered in downtown Cincinnati, Ohio, United States. Its main products include cleaning
agents, and personal care products. The company structure has been categorized into four
sectors: Beauty Care; Baby, Feminine, and Family Care; Fabric and Home Care; Health and
Grooming.
Old Spice Inc. is a subsidiary of Procter & Gamble. The company’s main product line is
deodorant and anti-perspirant sprays, roll-ons and sticks. Sales over the last five years have been
steadily growing in North America, and now the Old Spice team is ready to take its products
global. As the team’s leader, how would you assess whether or not international expansion is a
good idea? If so, which country or countries offer the greatest three-year revenue opportunity?
Estimate the greatest three-year revenue opportunity.

Possible Answer:
The candidate should first determine a structure for their approach. Then, asking probing questions to get as much information
as possible.
1. Additional Information: (to be provided to the candidate if asked)
• Location preferences: None. The Old Spice management wants the project team to tell them.
• Product line: Any or all of the three deodorant types can be launched into new locations. Each product (stick, spray, roll-on)
lasts 3 months long on average.
• Team: Internal resources appear sufficient to handle this launch. Candidate is the team leader.
• Capital, production and distribution: Parent company Procter & Gamble will support any well defined need. Worldwide
production and distribution facilities will help. Relationships with retailers are good.
• Financial targets: 25% annual revenue growth for 3 years and year one sales of at least $50M.
• North American sales: Last year’s sales were $350M. North American growth is about 10% per year.
• Suppliers: Non-issue, North American suppliers.
• Government/Legislation: No barriers.
• Financing this venture: Non-issue.
2. Insight Checkpoint (Redirect the discussion to these areas if necessary)
• In terms of market potential, Brazil, Germany and Japan are the largest.
• Each country is about equal in terms of deodorant usage rates.
• Prices do vary by country but in general the price points by product type do not vary much.
• Spray tends to be the highest and stick is the cheapest.
• Certain markets are more crowded than others: France, Spain and Germany. The UK, Brazil and Japan have significant
fragmentation with many small competitors.
• No one type of deodorant is dominant in terms of usage.

41 | P a g e
3. Exhibits
At this point, the interviewer should tell the candidate that they are going to show them 4 handouts (Exhibit 1-4). After each
handout, ask the Candidate for the key observations from each slide.
Exhibit 1. Population and deodorant usage

Exhibit 2. Average price paid per 2 oz.

42 | P a g e
Exhibit 3. Key Competitors by Market

Exhibit 4. Deodorant User Preferences

4. Market Sizing
The interviewer should then guide the Candidate to estimate the total market size, or annual revenue, of selling into the three
largest countries (Brazil, Japan, and Germany).
Brazil Japan Germany
Total market ~84M customers ~90M customers ~60M customers
Average price $2 (weight toward spray) $3 (weight toward spary) $2 (even weighting)
Usage ratio (no info) 4x (1 per quarter year) 4x (1 per quarter year) 4x (1 per quarter year)
10% (0.25 of fragmented 10% (0.2 of fragmented
Estimated market share 5% (not fragmented)
40%) 50%)
Annual estimated
84M * $2 * 4 * 0.1 = $67M 90M * $3 * 4 * 0.1 = $108M 60M * $2 * 4 * 0.05 = $24M
revenue

43 | P a g e
The estimated total annual revenue should be about $200M (round $67M + $108M + $24M = $199M)
A strong candidate will recognize that there may be some error in this margin and might safeguard with a 20% margin. Go with
$160M to be safe.
Now the interviewer should guide the candidate to calculate the required growth:
- in the first year, they could sell about $160M
- in the second year, they could grow at requested 25% ($40M), so sell $200M in total
- in the third year, they could grow another 25% ($50M) and have revenues of $250M.
5. Final Recommendation
In the end, the candidate should present a short, summarized version of their findings which should include their target
countries to enter as well as an estimate of the total possible annual revenue from each.

Case 08 - Pfizer to Introduce Anti-smoking Drug Chantix in India

Consulting Firm: Putnam Associates second round job interview.

Case Interview Question: Your client Pfizer, Inc. (NYSE: PFE) is a large multinational pharmaceutical corporation.
Headquartered in Midtown Manhattan, New York City and with its research headquarters in
Groton, Connecticut, United States, Pfizer produces a wide range of drugs. Global revenues in
year 2010 are USD $67.8 billion.
Pfizer is in the business of making anti smoking drugs such as Chantix. Chantix (trade name
Chantix in the USA and Champix in Canada, Europe and other countries, usually in the form
of varenicline tartrate) is a prescription medication used to treat smoking addiction. It both
reduces cravings for and decreases the pleasurable effects of cigarettes and other tobacco
products. Through these mechanisms the drug can effectively assist some patients to quit
smoking.
The client wants to sell Chantix at a premium price. You have been hired as a consultant to find out if the product can be
introduced in a country like India – and if so, what is the expected target market, market share and a feasible price at which the
drug should be sold. How would you go about it?

Possible Answer:
Candidate: Since this is a market entry and new product launch case, I would like to structure my discussion around theproduct
characteristics (development and customization) for the Indian market and then move on to the launch (competition,
distribution and promotion) part of the case.
Interviewer: This sounds fine to me. Also, please note that this product Chantix is
not entirely new; it has been introduced in other countries like the U.S. and Canada
already.
Candidate: OK, that experience should definitely help us. To start with, can you
tell me something more about the product? How is it different?
Interviewer: Unlike the lozenges or patches, this product is completely nicotine
free. It is 5 times more effective as proved by lab results and 50% of the test results
responded to the pill (which in this industry is an extremely high number thus
indicating success).
Moreover, it is a drug that cannot be sold over the counter – it requires a prescribed
dosage given by the doctor. It is to be taken for 3 months daily, 3 times a day.
Candidate: That is good. It gives us the advantage to position our product as superior due to the higher efficacy of treatment.
I would like to take up the competitive scenario next so that we can decide the price before determining the overall market size.
Interviewer: That’s a fair point. So, there is no similar product in the Indian market. Cheaper products like lozenges exist but
they contain nicotine and sell for 1 Indian rupee (INR) per unit.

44 | P a g e
Candidate: There are two ways that we can price a new product in a non-competitive market: Cost based pricing and
‘willingness-to-pay’ based pricing. In the first case, I would calculate the cost to the client company and charge a margin on the
same while in the second case, I would calculate the propensity of the consumer to pay for this drug. This would vary with my
target segment chosen. The curve would look something like this (Figure 1).
Ideally, we should be able to calculate the optimal profit case by considering the trade-off in sales volume vs. price for various
price points. The solution will also be influenced to an extent by the growth rates of the different target segments overall, say
movement of people to upper-class from lower-middle class.
Interviewer: Hmm…that is good. In our case, let us assume we did this and came up with INR 8 per unit. You think that
sounds reasonable?
Candidate: I think a price of INR 8 per pill is feasible because of the lab results – people will be convinced that it is a medically
prescribed drug and since it is a pre-scheduled dosage for 3 months, results are guaranteed. We can also stress on the fact that
it is nicotine-free and indirectly position this as a life saving drug.
Interview: OK, let’s estimate the market size assuming we decide to price it at INR 8 per unit.
Candidate: Let’s take Delhi as a base case. Population: 15 million. Target segment: 40% of the population smoke * 20% of them
would want to quit smoking * 75% can afford the Chantix drug = 9 million people. Each person will need: INR 8 per unit * 3
pills per day * 90 days = INR 2160 to quit smoking. So, 9 million * INR 2160 ~ INR 20 billion, or USD $400 million.
We can now assume that this drug will reach out to 25% of the population across India (2011 population 1.2 billion, urban +
rural since its effective and one-time payment to quit smoking), which means the total market size is USD $400 million * (1.2
billion/15 million) * 25% = $8 billion.
Interviewer: Very interesting. What will drive the market growth for our market share?
Candidate: The market growth rate will be affected by the sales and distribution coverage, willingness of people to quit smoking
and addition of new smokers who would want to quit after sometime. We can look to capture about 80% of this market
eventually, assuming no major competitor enters the market, which can be prevented by intellectual property rights and patent
support.
Since this is a prescription drug, the bulk of the promotion costs in this industry are in targeting the doctors and pharmacists
via direct sales agents or Medical Representative to convey the pros and cons for them to (a) prescribe the drug and (b) keep it
in their pharmacies. This will drive our market share from the potential market size.
Interviewer: Good. Any other costs/concerns that you would like to address?
Candidate: The training costs for the direct sales agents will also be critical as this is a new product and local agents would need
an in-depth understanding of the product. The number of sales people can be calculated by total workload method: Assuming
Doctor/Population ratio and say 3 doctors per day and repeat visits every 2 months; and pharmacist/population ratio and 3
pharmacists per day and repeat visits every 15 days.
The supply chain will have to be considered too – the warehousing, distribution network, retail chains, etc. We can perform the
cost benefit analysis for using middle distributors vs. direct distribution.
Interviewer: Great, I think we have covered all the aspects of the case. Thank You!

45 | P a g e
Case 09 - Royal Bank of Scotland Group to Expand into India
Consulting Firm: KPMG Advisory first round job interview.

Case Interview Question: The Royal Bank of Scotland Group plc (a.k.a. RBS Group, LSE: RBS) is a British banking
and insurance holding company headquartered in Edinburgh, Scotland, UK. The RBS Group
operates a wide variety of banking brands offering personal and business banking, private
banking, insurance and corporate finance throughout its operations located in Europe, North
America and Asia. As of December 2011, RBS had a market capitalisation of approximately
£12.2 billion.
Recently, the senior management of RBS Group wants to expand into the Indian market which
has a population of 1.2 billion as of 2011. They have hired you as a consultant to guide them
with this decision and advise them on various aspects of this new move. Is this a good idea?
How would you go about the case?

Possible Answer:
Candidate: First and foremost, I would like to know more about the RBS Group to evaluate their entry into the Indian market.
Is the client an investment bank or a retail bank?
Interviewer: The client RBS is into universal banking, i.e. they have both investment banking and retail banking arms.
Candidate: Can you tell me more about the core strong business areas of the client?
Interviewer: The client RBS has a strong presence in the retail banking business. It is the market leader in retail banking in UK,
Belgium, Netherlands, Luxembourg, Germany and Austria. Personal loans and business loans for small and medium enterprises
has been a big driver of its growth globally.
Candidate: Alright, in that case, I believe that the client should probably enter the Indian market with an initial focus on retail
banking. The client clearly has a lot of expertise in this segment and can efficiently leverage on its strengths to make an impact
in the Indian market. However, I’d also like to look at the Indian market before making this recommendation and first gauge
whether entering the Indian market in itself makes good business sense.
Interviewer: OK, that sounds reasonable. How would you ascertain the attractiveness of the market?
Candidate: Foremost, the market should have attractive growth prospects in the near future. Can you help with an estimate of
the growth projections in the Indian retail banking industry?
Interviewer: The retail banking industry is booming ever since the liberalization of the economy was initiated. The industry is
expected to grow at a CAGR (Compound Annual Growth Rate) of 28% to touch a figure of INR (Indian rupee) 9,800 billion
by the end of year 2012.
Candidate: The industry shows quite promising growth prospects and definitely looks attractive for the client to enter. Can you
tell me something about the degree of competition in the industry?
Interviewer: The industry is characterized by significant competition from numerous public sector banks, domestic private
banks and other international banks like the client; besides the numerous smaller scheduled and co-operative banks throughout
India.
Candidate: That’s not very encouraging for the client as it might significant barriers in penetrating the market. However, this
does not necessarily mean that the competition is fierce in all services. As I understand, most of the business in the retail
banking industry is currently generated out of corporate and consumer loans where banks primarily cater to the larger corporate
and the upper classes of the society. Would you agree with me?
Interviewer: That’s right but I am not quite sure where this is leading. Please make your point.
Candidate: Sure. I was trying to explore was the presence of untapped opportunities for the client in the market. As you had
mentioned earlier, the client has a strong presence in the personal and business loans business for small and medium enterprises.
However, the other banks in the Indian banking industry focus primarily on the larger enterprises. This means that the client
can strongly leverage its core expertise in the Indian market and tap into the huge market of SME enterprises.
Interviewer: Okay, I follow your arguments now. Do you see any other roadblocks besides competition for the client’s growth
prospects?

46 | P a g e
Candidate: I believe the existing regulations in the banking sector significantly restrict the number of branches foreign banks
can operate and it is very difficult for them to get the authorization for every new branch. This would pose a serious bottleneck
to the client’s growth.
Interviewer: So would you recommend the client to enter India?
Candidate: Yes, I believe the Indian retail banking business holds immense potential and based on this discussion about the
market, the competition and the client’s existing core businesses and strengths, I’d make a favorable recommendation. The
regulatory restrictions exist, but given the presence of numerous other international banks like HSBC, Standard Chartered Bank,
Citibank, etc, I believe the opportunities surpass the threats. Also, I might sound too pragmatic but given the pace of India’s
liberalization and development, I would definitely expect the regulatory restrictions to get relaxed in the near future.
Interviewer: Excellent job! I think we have covered all the main issues. We’ll just close the case here. Thank you.

Case 10 - Luxury Fashion House Chanel to Enter China Market

Consulting Firm: OC&C Strategy Consultants final round job interview.

Case Interview Question: Our client Chanel is a Parisian fashion house recognized as one of the most established in
haute couture, specializing in various luxury goods such as haute couture, ready-to-wear,
handbags, perfumery, and cosmetics among others.
OC&C Strategy Consultants have been approached by Chanel to help them think about their
online strategy in Asia Pacific region, with particular focus on the business opportunities in
China. Your specific task on this case is to scale the opportunity – i.e. how big is the current
market size of online luxury fashion goods in China and how many potential buyers currently
exist in the Chinese market.
Question 1: How would you estimate the market for online high-end fashion in China?
Possible Answer:

The good candidate explains that the market for online high-end fashion is driven by the size of the overall high-end fashion
market, the number of internet users in China (i.e. the internet penetration rate), the likelihood of internet users buying fashion
online, and the share of their online spending as a percent of total spending on high-end fashion.
The excellent candidate points out in addition to the above that we need to consider the impact of Chinese people buying high-
end fashion abroad while they are travelling. We also need to consider worries on counterfeit goods by Chinese consumers and
the lack of trust between buyers and sellers online would require adjusting the online model from the West to fit the unique
market situation in China.
Question 2: Given that the addressable high-end fashion market for our client in China is €400 million, how much do you
think the market for online high-end fashion is worth?
Possible Answer:
The interviewer will provide you with some data points to work out the market size, but you also need to figure out what are
the missing links and how to make reasonable estimates or assumptions on such missing information. Good candidates will
work through the logic piece by piece, whilst the excellent candidates will lay out methodology and information required upfront
and then work through them systematically. Fundamentally however you need to work out a ratio to split up the existing market
into online and offline portion by making assumptions on the online purchase behaviour.
The good candidate thinks about the internet penetration rate among high-end fashion shoppers in China (data point provided
here that the overall internet penetration in China is around 12%) and estimates how many of them would shop online for
fashion. Next, estimate the share of online spending as a percent of total spending on fashion. Finally multiply all these
parameters with the addressable market size for our client to get the answer for the market for online high-end fashion in China.
The excellent candidate does not just guess those numbers, but can justify where those numbers come from by drawing
reference to statistics, surveys, government data or derive a rational approach for the estimation. For example, when estimating

47 | P a g e
the internet penetration rate among high-end fashion shoppers, the excellent candidate contemplates the proportion of people
in urban centres likely to be the target for high-end fashion, uses urban statistics, adjusted for the high-end segment of the
population to come up with an estimate on internet penetration for high-end consumers. The candidate should also raise the
issue on how the existing state of e-commerce infrastructure, such as online payment, credit card availability, online security,
delivery, counterfeit, etc, would affect the online shopping behaviour and create potential hurdles for online shopping.
Question 3: What are the key hurdles for online purchase of high-end fashion in China?
Possible Answer:
The good candidate considers obvious factors that counterfeit products, poor online payment system, unreliable delivery
system, etc.
The excellent candidate also identify more subtle issues such as young age profile of internet users, lack of credible or branded
online retailers, lack of consumer protection, low credit card penetration and usage, etc.
In all of these calculations and discussions, there is an expectation that the candidate will be able to calculate the numbers
correctly. This does not mean that if you make a mistake you have failed, but that you can identify when you have made mistakes
(execute that all important sanity check) and ultimately arrive at a sensible and correct answer. There is also an expectation that
where you are forced to make estimates they are vaguely sensible!
Question 4: Estimate the number of potential buyers in China.
Possible Answer:
To estimate the number of potential buyers in China, you need to come up with an estimate on the average purchase value per
customer and divide the market size by that number.
Question 5: If our client decided to enter the China market, what are the points of
differentiation our client need to build in order to succeed in the market?
Possible Answer:
• The excellent candidate is very unlikely to get all of the items below but should be able to generate some interesting and
non-obvious issues for discussion, like:
• Build a credible brand name by advertising on target media, such as magazines or internet websites for high-end consumers
and leverage presence of physical stores to emphasize that it is a credible real business, not just a virtual store online.
• Provide special product selection, an edited offer with unique brands, unique merchandise or latest products that
customers cannot find elsewhere.
• Offer a risk free shopping environment to customers by providing free exchange and return.
• Provide loyalty programme and superb customer service to increase repeated purchase.
• Provide user generated content and community features in the website to enhance stickiness of the website.
Interviewer’s Note: This case interview question is based on a real case that OC&C worked on in 2007, sizing the potential
of the online luxury fashion market in China. This case study requires sound numerical skills, the ability to come up with some
sensible estimates, some awareness of consumer behavior and online knowledge. Excellent candidates differentiate themselves
by the quality of the answer, the logic and methodology in estimating the parameters used in the calculation and some
understanding of the China e-commerce market.

48 | P a g e
Case 11 - German Luxury Car Maker BMW to Enter Bangladesh Market
Consulting Firm: KPMG Consulting final round job interview.

Case Interview Question: Our client BMW (FWB: BMW) is a German luxury car maker that manufactures automobile,
motorcycle and engines. It also owns and produces the Mini marque, and is the parent company
of Rolls-Royce Motor Cars. Headquartered in Munich, Germany, the BMW group
produced more than 1,400,000 automobiles and over 110,000 motorcycles worldwide across all
its brands in 2010.
Recently, The CEO of BMW wants to grow business and is looking into selling cars in
Bangladesh (a sovereign state located in South Asia and bordered by India). The current
population of Bangladesh is estimated to be 142 million and the country’s GDP growth is 5%
per year during the last three years. The CEO of BMW wants you to help find out whether they
can break even in three years if his company enters the Bangladesh market. How would you go
about it?
Additional Information: (to be given to you if asked)
• Currently, the only luxury car sold in Bangladesh is Mercedes-Benz (FWB: DAI) and they have been in this market for
the past 10 years.
• Mercedes-Benz imported and sold 10,000 luxury cars in this market over the past 10 years, and has their own dealership
in Bangladesh.
• There are ~1000 new buyers for luxury cars in Bangladesh each year.
• Existing luxury car owners replace their cars every 10 years (the interviewee should calculate how many new cars are sold
to existing owners – 1000 per year, and therefore the total market size per year is 2000 new cars)
• If BMW enters the market, they will have 30% (The interviewer should not give this info out right away, ask the candidate
to estimate first) market share each year.
• The price Mercedes-Benz charges is $100,000 per car.
• Assume the discount rate is zero.
Possible Answers:
1. Ask the candidate how to estimate BMW’s market share.
Possible Approach:
• Understand customer needs through survey and estimate how well we could meet those needs and therefore how much
market share we could gain.
• Find benchmark in other markets. It turned out that our client already entered Vietnam and other Southeast Asian
markets similar to Bangladesh and on average gained 30% market share in each of these markets.
2. Break-even analysis
Cost Structure
▪ Fixed Costs: Initial one-time investment: $7 MM.
▪ Variable Costs (per car):
▪ Manufacturing $20,000.
▪ Transportation 120% of manufacturing cost. $20,000 * 120% = $24,000
▪ Customs/Taxes 95% of manufacturing + transportation costs. ($20,000 + $24,000) * 0.95 = $41,800
▪ SG&A (Selling, General and Administrative Expenses) 12% of all the above costs combined. ($20,000 +
$24,000 + $41,800) * 0.12 = $10,296
The total variable cost per car rounds up to $96,000/car. Therefore, if BMW charges the same price for their luxury car as
Mercedes-Benz, $100,000 per car, the profit on each car is $4000. With 30% market share, the client’s annual profit will be
2,000 cars * $4,000 per car * 30% = $2.4 MM. Given the initial investment of $7 MM, BMW will likely break even in 3 years.
3. An excellent candidate should also evaluate potential risks of entering this new market
▪ Assumptions might be inaccurate.

49 | P a g e
▪ Client BMW may not be able to grab 30% market share like they did in other countries.
▪ Economic downturn could cause less people to buy luxury cars.
▪ BMW may have a hard time competing with established player in this new market, price may have to be reduced.
▪ Bangladesh may not be stable politically and economically, therefore our client will bear more risk.
▪ There could be other new entrants like Audi, Jaguar, Volvo, Lexus, Infiniti, etc.
▪ Existing competitor Mercedes-Benz could react to our entry by
▪ Reducing price (the best answer might also mention that with the low income level in the country, customers who
can afford luxury cars might not be price sensitive at all). Ask candidate what price Mercedes-Benz will reduce their
car to. Possible Answer: $96,000/car or less.
▪ Improving BMW’s services if that’s the differentiating factor of our product.
▪ Blocking some local resources.

50 | P a g e
BUSINESS TURNAROUND CASES
51 | P a g e
Case 01 - Finisar Corp Experiencing a 50% Revenue Decline

Consulting Firm: Siemens Management Consulting (SMC) final round full time job interview.

Case Interview Question: Our client Finisar Corporation is a manufacturer of optical fibers based in Sunnyvale,
California, United States. An optical fiber is a flexible, transparent fiber made by drawing glass
(silica) or plastic to a diameter slightly thicker than that of a human hair. Optical fibers are used
most often as a means to transmit light and data between the two ends of the fiber and find
wide usage in fiber-optic communications, where they permit transmission over longer distances
and at higher bandwidths (data rates) than electrical cables.
The CEO of Finisar Corporation is new and is trying to assess the firm’s business situation. At
a surface levels, she knows that the firm is experiencing a 50% decline in revenues, but has hired
our consulting firm to dig further.
Specifically, the CEO of Finisar Corp. has three questions for you:
• Why is the client firm experiencing a 50% revenue decline?
• When will the firm rebound?
• How does the client firm relate to its competition?
Additional Information: to provide to candidate upon request
• Data types transmitted over these optical fibers are internet, TV, phone, etc.
• The client Finisar Corporation specifically sells these optical fibers to big telecom companies like AT&T, T-Mobile and Sprint,
and they put them underground and use them to transmit data over long distances.
Possible Answers:
The candidate should tackle each one of the three questions in this order.
Question 1. Why is the client firm experiencing a 50% revenue decline?
Possible Answer:
The answer to this question is qualitative in nature. The optical fiber industry as a whole is suffering because 4 years ago more
optical fibers were put in the ground then there was demand for. Each optical fiber cable can support about 1 tera bit of
information, and the demand per second (unit of time we’ll use) is less than 1 tera bit per hour.
The candidate should then be looking for data that will help them see when the market or demand for these optical fiber cables
will come back.
Question 2. When will the firm rebound?
Additional Information:
When prompted, the interviewer will reveal:
• Candidate should use the market of fibers between D.C. and NYC as representative of the world for the purposes of this
case.
• There are 1,000 optical fibers between these two cities (D.C. and NYC) and the peak of demand is 100 terabits/second.
• Every year the peak demand doubles. Use time zero of demand as 100 terabits, year 1 as 200, and so on.
Possible Answer:
The candidate should determine that:
Year 0 = 100
Year 1 = 200
Year 2 = 400
Year 3 = 800
Year 4 = 1,600

52 | P a g e
Demand will exceed capacity in year 4. So, somewhere between year 3 and year 4, more cables will have to be installed and the
company can rebound. This is sufficient for this part of the case (there is no need to get into how much of that market they
can capture, etc.)
Question 3. How does the client firm relate to its competition?
Additional Information:
Note to interviewer: Ask the candidate what they would look at to compare the client firm to the competition. Don’t let them
go until they’ve exhausted most of the things on the list below. First have them list all things, then give them information on
the firm as it relates to each category on their list.
Category Information on Firm
Price Commodity good – so the same price as everyone else
Product Quality Not relevant to customers, commodity market
Manufacturing Capacity Largest capacity, most idle capacity
Cost Structure Lowest cost structure/best profits
Share of Market #1 in market, have scale benefits
Customer Base Sell to the major companies in market like AT&T, T-Mobile and Sprint
Geographic Footprint/Distribution Same as competitors
Cash/Assets (Balance Sheet) Lot of cash on hand, no debt
Possible Answer:
The candidate should identify that:
Some of these are things you would always look at, but some of them are important to this case. For example, capacity matters
because the firm with idle capacity will be able to grab the new business when the market picks up again. Also the firm with
cash on hand and no debt will be able to get through tough times when the market is slow.
Question 4. After answering the 3 initial questions, tell the candidate that he/she’s done a good job getting through the 3
questions put forth, but you still have one more. Someone on the CEO’s team wants to get a quick sense of the firm’s variable
and fixed costs. This is an ad hoc question they want a quick answer on.
Additional Information:
Two months of data.
• Month 1
– Produce 3MM km of optical fiber
– Costs are $50MM
• Month 2
– Produce 2MM km of optical fiber
– Costs are $40MM
Possible Answer:
To obtain the firm’s variable costs, look at the difference between Month 1 & 2 costs.
1MM km of optical fiber costs $50MM – $40MM = $10MM
Thus, fixed costs = $40MM – 2MM km * ($10MM/1MM km) = $20MM per month, or
fixed costs = $50MM – 3MM km * ($10MM/1MM km) = $20MM per month
Question 5. Let the candidate know that the CEO of the firm has a couple minutes between meetings and is looking for a
quick summary and recommendation. What would you tell the CEO?
Possible Answer:
The candidate should use this opportunity to update the CEO on questions 1, 2 and 3 (not the variable/fixed costs question).
Sample Recommendation:
• Your company’s revenues have declined because the industry as a whole has declined. The current optical fiber supply exceeds
demand.
• The optical fiber market demand will bounce back in 3 years.

53 | P a g e
• You’re in a great position to capture the new demand (excess capacity, largest player, stellar customer base) and weather the
market slow down (have lot of cash on hand, no debt).
A strong candidate can also offer next steps or potential risks, but this part is not critical.

Case 02 - Medline to Reverse Trend of Declining Profitability


Consulting Firm: ClearView Healthcare Partners first round full time job interview.

Case Interview Question: Our consulting firm has been hired by the CEO of Medline Industries, Inc., a regional medical
device company headquartered in Northfield, Illinois. It mainly produces consumable medical
supplies and markets its products in all 50 states in the US. The company is one of the
largest privately held manufacturers and distributors of medical supplies.
The client is very profitable and doing well. In fact, its business has grown from a small, regional
medical supplies manufacturer and distributor with annual sales of USD $100 million ten years
ago to having annual sales of $2 billion today. However, the CEO of Medline is not sure that
all of this fast growth has been great. In fact, he knows it hasn’t: the CEO knows that its profit
margins have decreased over the last 10 years. The CEO has hired our consulting firm to explain
why that is, and what they should do to reverse the trend of decreasing profitability. How would
you go about it?

Additional Information: to be provided to candidate upon request


• The client company Medline Industries makes medical supplies. While the specific product itself is not a focal point for the
case, if the candidate asks, the interviewer will tell them that the company makes consumable products like surgical packs with
clean surgical tools, syringes, swabs, gloves, etc.
• The company started out with a small product line – cotton swabs, hygienic swipes and medicated tissues were the basis of
its original product line. It has expanded tremendously to include disposable surgical clothing, surgical packs with sterile tools,
syringes, plastic containers for blood and other bodily fluids, etc.
• The company was a small regional player in the midwest area of the United States originally. Now, it serves all 50 states and
some hospitals in Canada and Mexico, too.
• The company’s customers are primarily hospitals and hospital chains, but it also serves some large physician group practices
and clinics, depending on the regional area.

Possible Answers:
1. Case Overview
This is a profitability case, but it moves quickly from the profitability framework (Profit = Revenue – Cost) to data analysis,
brainstorming and problem solving. While the client does want our consulting firm to answer the question about why
profitability has declined, the interviewer will likely hint up front that the candidate should push even further to fixing the
problem.
This is a good case for practicing looking at data and brainstorming different causes for the resulting data. It also is a great
example of how an candidate can take a case past just structuring and solving the question to actually trying to “fix” the problem
and implement a solution, which is what some consulting firms are looking for.
2. Detailed Analysis (including quantitative and qualitative evaluations)
Interviewer: First, do you have any questions about the case?
Candidate: Yes, I want to make sure that I understand how the client’s business works. You said that it was a small, regional
manufacturer and distributor of medical supplies. What kind of medical supplies does it make?
Interviewer: Sure, that makes sense. The client makes consumable products like surgical packs with clean surgical tools, syringes,
swabs, gloves, etc. The company started out with a small product line — cotton swabs, hygienic swipes and medicated tissues
were the basis of its product line. It has expanded tremendously to include disposable surgical clothing, surgical packs with
sterile tools, syringes, plastic containers for blood and other bodily fluids, etc.

54 | P a g e
Candidate: Great. And I’m assuming that we’re focused only on the U.S. market — or should I consider other markets as well?
Interviewer: The company was a small regional player in the midwest of the United States originally. Now it serves all 50 states
and some hospitals in Canada and Mexico, too. But, for the purposes of this case, just focus on the U.S. market.
Candidate: And before I get started, can you just give me a sense of who its customers are? I mean, does it market to physicians
like big pharmaceutical companies do, or to someone else?
Note: One thing that’s great to do during this initial information gathering is to drive some of the descriptions. For example,
even though the candidate would really just like to know who the customers are, he/she throws in comparables – pharmaceutical
companies. It shows the candidate’s knowledge of the industry and use of comparables.
Interviewer: That’s a great question. The company’s customers are primarily hospitals and hospital chains, but it also serves
some large physician group practices and clinics, depending on the regional area. Does that give you the sense
that you were looking for?
Candidate: Yeah, that’s great. And just so I’m clear on the client’s goal: It has seen its profitability decline as the company has
grown, and it’s just curious as to why and how to reverse that trend?
Interviewer: That’s right. Now, how would you try to answer that question? Feel free to take a few minutes to collect your
thoughts.
Candidate: Great — thanks. (A few minutes go by…)
Candidate: Since this is a profitability issue, I would want to look at the two levers of profitability — revenue and costs — to
understand whether there have been changes on either side that might have led to eroding profitability. I’d like to start on the
costs side, as I’m wondering while the company has grown, if it’s had to incur some costs due to expansion that has made it
less profitable.
Note: The candidate’s structure for the case is spot on. But because profitability cases are so easy and clear to structure, it’s a
good idea to suggest which lever you’d like to look into and your hypothesis as to why that might lead to the answer—it shows
that you are really driving the case.
Interviewer: That’s great. And we actually looked at expansion in its cost base as it grew, and the C suite of the client had looked
at that, too — but that’s not where the answer actually lies. The client had kept a good eye on its costs, and its cost base. So,
that’s actually not where the answer is, but it’s a good hunch.
Candidate: I see…
Interviewer: After we eliminated costs, that left revenue. What would you look at there?
Candidate: Well, revenue is price times units. Since its business has grown in terms of regions and customers, I’m going to
assume that units sold has increased. Has anything changed in terms of price? Has it gone down?
Interviewer: That’s right on track. Let me show you some data that we collected on the discounts that it’s giving to its customers.
What does this data tell you?
Exhibit 1: Units Sold by Discount

55 | P a g e
Candidate: Well, it looks like the number of units sold under a higher discount (20% and 15%) has grown over the years. In
Year 1, it looks like 120 units were sold in total, and about 105 of those were sold at a 5% to 10% discount. Whereas, in Year
4, it looks like the number of units sold grew to 250, but only almost 150 of those were sold at a 5% to 10% discount.
Interviewer: So, what percent were sold at the higher discount in Year 1 vs. Year 4?
Candidate: Well, in Year 1, it looks like (15/120 = 12.5%) 12.5% were sold at the higher discount (20% and 15%), and in Year
4, it looks like (100/250 = 40%) 40% were sold at the higher discount. So, they’re discounting higher on more of their sales,
which is why its profitability is eroding.
Interviewer: That’s right. Now, let me show you another piece of data. What does that data tell you?
Exhibit 2: Discount by Customer Relationship

Candidate: Well, it’s not what I would expect. It looks like the company is giving some of the highest discounts on its fees to
its midsized and small customers, not to its largest ones. This may be keeping the profitability high on its large clients, but it’s
probably destroying margins on its midsized clients, and bringing down its overall fee average. I think this shows the answer to
the client’s question.
Interviewer: Sure. Now here’s another question: if you were the client, what would you want to see here?
Note: Here, the candidate should generate an idea or two, maybe even draw on the exhibit to illustrate his or her point. There
are three ideas below.
Candidate: Well, I would want to see a few things.
First, I would want to be giving the biggest discounts, or the most discounts, to my larger clients — like volume discounting.
Here, it doesn’t look like the client is doing that — in fact, it looks like it’s doing the opposite.
Second, it seems pretty random where the discounts are given — I wonder what is driving them. There are a few clusters of
discounts by relationship size, or even by discount size — I wonder whether the clusters of discounts represent a certain
salesman, a certain region a or a certain client type, like hospital or clinic or hospital chain.
Interviewer: That’s great. We went over this data and looked into the clusters specifically, and it turns out that there’s high
variation to discounting by sales person, and that this correlates to regional area. It turns out that salesmen in the midwest,
where the company was founded, were giving much lower discounts to their clients than say salesmen in other regions, where
the company was expanding into. Knowing that, how might you try to fix the problem?

56 | P a g e
Candidate: I would make sure that all of sales force incentives were tied to low discounting, as well as revenue targets, which I
think sale force members are usually incentivized on.
Interviewer: Yep, incentives are good, and we definitely recommended that to the client. But incentives are hard to change
overnight, because they’re in people’s contracts; they often need a year to take effect after you put them in place, because it
takes a while for people to adjust business practices. What else would you do?
Note: Here, the interviewer is testing the candidate to come up with more solutions — it’s a creative/short brainstorming test.
Candidate: I might also share this new data with the sales force in some sort of training so that they can see how they’re affecting
company’s overall profitability — I mean, they’re probably not aware that they’re doing this as a whole.
Interviewer: That’s right — train them. Now, what else?
Candidate: I might make a company-wide discount cap. Does it have that right now? I mean, from what I see on Exhibit 1, in
Year 1, it looked like most sales didn’t go over a 10% discount, but by Year 3, 20% seemed to be the cap. Maybe the company
would like to set a cap at 15% — or at least figure out a cap that made sense for its profitability target.
Interviewer: That’s great, and that’s exactly what we suggested to the client.

Case 03 - Bombardier to Turn Around and Boost Investor Confidence


Consulting Firm: PricewaterhouseCoopers (PwC) Advisory first round full time job interview.

Case Interview Question: Your client Bombardier Inc. is a Canadian multinational aerospace and transportation
company. Bombardier was founded by Joseph-Armand Bombardier as “Bombardier Snow Car
Limited” in Quebec, Canada in 1942. Starting as a maker of snow machines or snowmobiles,
over the years the company has grown into a large manufacturer of regional airliners, business
jets, mass transportation equipment, and recreational equipment.
Bombardier’s aircraft division “Bombardier Aerospace” focuses on regional and business jets.
Lately, Bombardier’s C-series aircraft, its largest jet, has a number of problems including cost
overruns, delays, and mechanical issues. Investors are losing confidence in the company, as its
stock price has declined almost 70% within the past year, and as its credit ratings have fallen. A
large number of orders of the C-series that are scheduled to be fulfilled in the next 3 years, are
at risk of cancellations and further delays. What should the client Bombardier consider and do to change the direction of the
company, and increase investor confidence?

Possible Answers:
1. Information Gathering
Additional Information to provide if asked by candidate:
• Aircraft revenue makes up more than 80% of Bombardier’s revenue.
• Although Bombardier has other aircraft types, the C-series is the costliest of their aircraft, as well as the type with the greatest
number of orders.
• Bombardier competes with major aircraft manufacturers, such as Boeing and Airbus, and sells their aircraft to airlines around
the world.
2. Suggested Framework
Prompt 1: What should Bombardier consider and do to change the direction of the company, and increase investor confidence?
Possible Answer:
Possible responses or areas that the candidate could consider:
• Market environment: Demand for Bombardier aircraft, air travel; competitor production trends and market share; risk of
customers cancelling Bombardier orders for competitors.
• Bombardier operations: What is leading to delays and cost overruns? Mechanical issues, supplier issues, distribution
inefficiencies, employee productivity?

57 | P a g e
• Turnaround strategies: Marketing (e.g. Paris Air Show), discounts to customers to build loyalty, spin-off train division to raise
capital, ask for loan/subsidy from Canadian government.
3. Detailed Analysis
Prompt 2: To understand the significance of Bombardier’s issues with the C-series, Bombardier needs to consider which of
their customers are at high risk for cancellation. Hand Exhibit #1 to the candidate, and ask the candidate the following
questions.
• What do you observe from the following exhibit?
• What percent of the orders are at high risk of cancellation?
The candidate should drive towards the significance of this. If not, ask the candidate for the implications.
Exhibit 1. Bombardier’s C-Series Deliveries

Possible Answer:
• The candidate should note that 78 aircraft or 40% of total deliveries received the highest risk rating due to geopolitical or
financial uncertainty.
• The candidate should point to the potential worst-case scenario of Bombardier ending up with an accumulation of planes
from cancelled orders.
Prompt 3: Anticipating the possibility that “high risk” aircraft orders will be cancelled due to production delays, what can
Bombardier do?

58 | P a g e
Possible Answer:
This is a brainstorming exercise for the candidate. In addition to noting the potential strategies, a strong candidate should also
consider the implications of those strategies. Possible responses include the following:
• Cease manufacturing aircraft that are “high risk.”
– Implication: Loss of desperately needed revenue. But with fewer aircraft to manufacturer, there’s a possibility of reducing the
delay on other orders.
• Reaffirm orders with high risk customers.
– Gain confidence in orders from high risk customers.
• Provide discounts to airlines with orders
– Loss of revenue. Discounts may not be enough to convince airlines to maintain their orders. This may incentivize airlines to
bargain the price downward.
• Find other airlines or entities that could be potential customers for C-series aircraft.
• Invest in R&D to modify or enhance C-series to make the aircraft more attractive to airlines
– Bombardier already has too high of debt to equity ratio. This could lead to further delays with existing customers.
4. Recommendation
Prompt 4: The CEO of Bombardier is about to enter the room to learn your recommendations. Please prepare your findings
and conclusions for him.
Possible Answer:
The candidate should succinctly state the problem that the client is facing, and provide strategies and potential risks to those
strategies. Examples of strategies and risks are below:
• To boost investor confidence and turn Bombardier around, the company should find a way to reduce the risk of cancellations
for the C-series aircraft. In the short term, Bombardier could appeal to the Canadian government for additional funding, and
try to boost customer loyalty to reduce the risk of cancellation. However, Bombardier should also determine the root cause of
the delay to find a way to expedite production.
• In continuing to invest in the C-series to turn the company around, there is a risk of further delays and cost overruns, which
could lead to bankruptcy. However, by not following through on existing orders, Bombardier would be risking its credibility as
the leading aircraft manufacturer in Canada. This could lead to a loss of loyalty of customers, and further issues in the long run.

Case 04 - Gold Mining Company to Mitigate Risk of Political Protests

Consulting Firm: NERA Economic Consulting first round full time job interview.

Case Interview Question: Your client Gold Mining USA Inc. (GMUI) is a large gold mining company based in
Greenwood Village, Colorado, United States. Gold mining is the resource extraction of gold by mining. The environmental
impact of gold mining includes erosion, formation of sinkholes, loss of biodiversity, and
contamination of soil, groundwater and surface water by chemicals from mining processes.
Besides creating environmental damage, the contamination resulting from leakage of
chemicals also affect the health of the local population.
Recently, the client GMUI has seen its revenue decline 20% over the past month. It has
hired your consulting firm to (1) investigate the cause of the recent revenue decline, and (2)
help reverse this trend. How would you go about it? What would you recommend the client
do?

Possible Answers:
1. Case overview and suggested framework
The candidate should first seek contextual information to better understand the client company’s operating scope and
geography, then develop a framework that covers the following areas:

59 | P a g e
• gold extraction/sale value chain
• industry trends
• competitors
• geopolitical issues.
2. Additional Information
The following information is available upon request:
• The client company has two gold mines. One in Kazakhstan and one in Mongolia. Both are high elevation mines.
• The mine in Mongolia is being ramped down, and is not a meaningful driver of revenues or costs.
• Reserves in the Kazakh mine are plentiful.
• There are no issues with the mining or processing parts of the value chain.
• Within the Kazakh mine, the amount and location of gold is precisely known.
• Gold prices have been remarkably stable and are projected to remain so.
• The Kazakh mine operates 24/7 365 days per year.
3. Detailed Analysis
Prompt 2: As it turns out, the client’s recent revenue decline has largely been due to a reduction in available gold from the
company’s mines. There is only one road that leads to the Kazakh mine, and it has recently been blocked by local villagers who
live in the area. The opposition political party seems to be organizing everyone.
The client company has learned through local connections that the opposition political party is demanding USD $2MM for the
road to be cleared today. The company has also been informed by the Kazakh government that they can clear the road in three
days by military force. What information would you want to determine whether it makes sense to pay the $2MM demand?
Additional Information:
• Not illegal to pay – but the client has never done so in the past
• No financial outlay associated with military action
• Opposition party could clear road as soon payment is promised
• Gold sale price per ounce = $1,200 per ounce
• Production capacity = 1,500 ounces per day
• Production capacity is 60% lower with blockage of road
• Costs (brainstorm here: “What kinds of costs should we consider?”)
– Transportation Gasoline = $200,000 Per Day
– Transportation Wear & Tear = $10,000 Per Day
– Employees = $90,000 Per Day
Cost-benefit calculations:
Options Pay the $2MM Wait 3-Days
Revenue Per Day $1,200 * 1,500 = $1.8MM $1.8MM * (1-60%) = $0.72MM
Cost Per Day $200K + $10K + $90K = $0.3MM $0.3MM
Daily Contribution Margin $1.8MM – $0.3MM = $1.5MM $0.72MM – $0.3MM = $0.42MM
3-Day Contribution Margin $1.5MM * 3 = $4.5MM $0.42MM * 3 = $1.26MM
Payment to Opposition Party $2MM N/A
Net 3-Day Contribution Margin $4.5MM – $2MM = $2.5MM $1.26MM

*Wait 3-Days daily revenue is calculated by using 40% of production capacity due to road blockage.
Conclusion: Financially, it makes sense to pay the $2MM demand right away.
Prompt 3: What non-financial elements would you consider in determining whether the client company should pay or wait for
the Kazakh military to clear the road?
Additional facts and assumptions:
• Short-term vs. long-term profits
• Precedent
• Political instability
• Selling reserves
• What if we could be assured, with 100% certainty, that this extortion-type behavior would never happen again?

60 | P a g e
Prompt 4: How could the client company mitigate the risk of future politically motivated protests?
Possible Answer:
• Community Engagement
• Local leaders on company board
• Employ villagers
• Corporate social responsibility (CSR) efforts
Prompt #5: What would you recommend the company do and why?
4. Performance Evaluation
• Average Candidate
Gets all math correct.
• Good Candidate
Gets all math correct and is thoughtful on qualitative aspect of the case.
• Excellent Candidate
Drives the case. Weighs the financial and qualitative costs/benefits. Delivers a concise recommendation that includes next steps
focused on limiting the likelihood of future production stoppages.

Case 05 - Verizon to Turn Around Directory Assistance Business

Consulting Firm: Deloitte Consulting (Strategy & Operations) second round job interview.

Case Interview Question: Your client Verizon Communications Inc. (NYSE: VZ, NASDAQ: VZ) is a global
telecommunications company. The company’s headquarters are located in the Verizon Building
in Lower Manhattan, New York City, United States. Verizon provides directory assistance
services in the $50 billion United States telecommunication market. They also offer standard
telecommunication services, including telephone, broadband internet, wireless, and television
connectivity.
In recent years, there has been a growing threat from electronic competitors (like Skype,
Google) who provide the same directory assistance services, but for far cheaper. In the past five
years, your client Verizon has lost 4.5 million customers and their revenue from directory
assistance services has declined at a 20% CAGR (Compound Annual Growth Rate) in the past
three years. They’ve approached you to help them turn around the directory assistance segment only. What should they do?

Additional Information: (to be given to you if asked)


1. Product
▪ Verizon charges $1 per call for directory assistance services.
▪ Their service consists of basic directory assistance: customers call and receive address and phone number, that’s it.
▪ Directory assistance calls are either answered by a person or picked up by an automated answering system.
2. Company
▪ Verizon has a unionized workforce
▪ Verizon is using an old data system

61 | P a g e
Exhibit 1: Number of Unique Callers from Year 2004 to 2008 (in millions)

Exhibit 2: Information About Calls Received

Average calls per caller

Type Number of calls % of total


High volume 50 5%
Mid volume 10 15%
Low volume 11 80%
Exhibit 3: Costs per Call
▪ Variable costs
▪ Staffing (Only applies to calls answered by staff): $1
▪ Software licensing (Only applies to automated calls): $0.10
▪ Fixed costs
▪ Utilities: $1 million / year
▪ Capital upgrades: $250,000 / quarter
▪ SG&A: $3 million / year

Exhibit 4: Automated vs. Staffed Calls


% of calls that are automated
▪ State 1: 25%
▪ State 2: 50%
▪ State 3: 25%
▪ State 4: 75%
▪ State 5: 75%
▪ State 6: 50%

Notes to Interviewer:
This is largely a numbers case. The interviewer should focus the initial conversation on the market, competitors and the
company to help the interviewee understand the issue. Then ask the interviewee what information would be needed to estimate
profits for the next few years.
As the interviewee asks for additional information, provide the four Exhibits. Collectively, the 4 exhibits allow the interviewee
to construct an income statement (See the one in “Possible Answer” section). After an income statement has been generated,
ask him/her: how long until the company’s profits in directory assistance business become negative?
Then, ask how to turn around that trend. After the interviewee brainstorms, see if “sell the business” is an idea. Then ask about
the possibility of selling the business, such as:
▪ How much would the business be purchased for?
▪ What are some challenges in selling this sort of business?

62 | P a g e
▪ What are some possible response from competitors?

Possible Answers:
Exhibit 1:
▪ The number of subscribers from year 2004 – 2008 can be used to estimate the number of subscribers in the next two
years.
▪ It is safe to assume the downward trend will continue and round to even numbers, so 5 million in 2009, 4 million in
2010.
Exhibit 2:
▪ Average calls received can be used to estimate the average number of calls made per subscriber: (50 * 5% ) + (10 *
15% ) + (1 * 80% ) = 4.8 calls per subscriber,
▪ It is safe to round 4.8 calls to 5 (even just from eyeballing)
▪ From Exhibits 1 and 2, we can derive revenues for year 2009 and 2010.
▪ Year 2009: 5 million subscribers * 5 calls per subscriber * $1 per call = $25 million
▪ Year 2010: 4 million subscribers * 5 calls per subscriber * $1 per call = $20 million
Exhibit 3: Costs
▪ Fixed cost = $1 million + $250,000 * 4 + $3 million = $5 million per year
▪ Variable cost = cost of automated calls + cost of staffed calls
▪ Automated call cost = $0.10 * number of automated calls
▪ Staffed call cost = $1.00 * number of staffed calls
But, we need Exhibit 4 to get the number of automated calls and staffed calls first.

Exhibit 4: Calls Breakdown


It is safe to assume each state has a uniform number of calls, so the average of numbers means (25% * 2 + 50% * 2 + 75% *
2) / 6 = 50% of calls are automated, and 50% are staffed.
Take the total # of calls from Exhibits 1 and 2, divide into automated and staffed calls, then multiple by the cost of each type
of calls, we can set up an income statement table:
Year 2009 2010
Total Revenues $25 million $20 million
Fixed cost $5 million $5 million
Variable cost
Cost of automated calls 25 million * 50% * $0.10 = $1.25 million 20 million * 50% * $0.10 = $1.0 million
Cost of staffed calls 25 million * 50% * $1.0 = $12.5 million 20 million * 50% * $1.0 = $10.0 million
Total variable cost $13.75 million $11.0 million
Total Costs $18.75 million $16.0 million
Profits $25 – $18.75 = $6.25 million $20 – $16.0 = $4.0 million

Recommendation
Based on our analysis, Year 2009 and 2010 will have projected profits of $6.25 million and $4.0 million, respectively. However,
due to the steep downward trend in the number of unique callers, the profit will vanish in about 2 – 3 years. In order to turn
around the directory assistance business, the client could take the following measures:
▪ Upgrade their data system
▪ Cut cost by pushing more calls to automated system
▪ Increase price per calls
▪ Add additional services, such as direction assistance, traffic monitoring assistant, etc.
▪ Imitate competitors like Skype or Google
▪ Acquire new competitors
▪ Sell the business
However, there are certain risks associated with the above mentioned measures:
▪ It may be difficult to reduce unionized workforce
▪ Customer response from re-pricing calls
▪ Customer user experience difference between automated calls and staffed calls

63 | P a g e
Case 06 - Swiss Food Giant Nestle Losing Its Innovative Reputation

Consulting Firm: IBM Global Business Services (GBS) 2nd round job interview.

Case Interview Question: The client Nestle S.A. (SIX: NESN) is a large multinational diversified foods and consumer
products company, founded and headquartered in Vevey, Switzerland. Nestle originated in a
1905 merger of the Anglo-Swiss Milk Company and the Henri Nestle Company. Today, the
company operates in 86 countries around the world and employs more than 280,000 people.
Revenue is CHF 109.72 billion in fiscal year 2010.
You have been brought in as a consultant because recently Nestle is experiencing stagnant
market share. What’s worse, the client is quickly losing its innovative reputation as it is more
frequently merely adding product line extensions rather than new products. This loss of
innovation has been well documented. What are the drivers of this trend? What would you do
to turn around the trend?

Additional Information: (to be provided to candidate if requested)


1. Customers
Market research has shown that customers do value innovation.
2. Competitors
Other companies are still being innovative. In fact some of the innovations of other companies are similar to new products
which the client Nestle worked on, but discontinued.
3. Company
The Nestle company has over 10,000 products, including baby food, coffee, dairy products, breakfast cereals, confectionery,
bottled water, ice cream, pet foods, etc. About 50% of their products have an unchanging market share. 40% have declining
market share (largely due to offbrands) and 10% have increasing market share. Some of the products, but not all, are in mature
product lines. It has been well documented that mature product lines seldom result in innovations.

The Nestle company has a reputation for producing the highest-quality products.
Nestle spends a great deal on research & development (R&D). There has been no decrease in the percentage of sales spent on
R&D, but is now much more fragmented. R&D used to all be co-located, and a study showed that scientists spent 30 percent
of their time on research. The directors desire to be in the middle of everything has resulted in scientists spending a lot of their
time preparing reports about what they are doing.
The company has become much more risk-averse over the past few decades. Failures are penalized much more heavily than
they used to be. Great disincentives exist to share information.
The company has no formal process in place for developing new product, and is largely driven by the gut instincts of directors
of the division. There is no focus on product potential or the development paths to pursue for promising candidates.
Possible Solution:
At a minimum, the candidate should discuss the following areas:
Address the need for new product development. Create processes for assessing which products to pursue and which not to not
pursue. These processes may include analysis of cost structure, market dynamics, revenue stream, cannibalism, etc. Also ensure
that any new processes are monitored to maintain accuracy.
Create a more innovative and friendly environment. The culture has created disincentives to be innovative and take risks, a
trend that needs to be reversed.
Investigate ways to create incentives to share information and take educated risks. Investigate ways to get more bang for the
research buck. One possibility is finding ways to focus more scientist time on conducting research instead of writing reports.
Focus product development efforts on non-mature product lines. Suggest doing more investigation on which product lines
have the highest probability for successful innovation.

64 | P a g e
Case 07 - Kodak to Improve Profit Margin for Digital Cameras

Consulting Firm: Booz & Company final round job interview.

Case Interview Question: The client Eastman Kodak Company (NYSE: EK) is a multinational consumer good /
consumer electronics company headquartered in Rochester, New York, U.S. Long known for
its wide range of photographic film products, Kodak is re-focusing on two major markets:
digital photography and digital printing.

One of Kodak’s products, Digital Camera has never made a profit since its inception although
the division has enjoyed double digit growth in the last 4 years. Investors are getting impatient
with the negative earnings. Questions to you, a turnaround specialist hired by the CEO of
Kodak are the following:
1. What is a good target profit margin for the client (can be zero)?
2. Should the client exit the digital camera business?
3. What recommendation would you make to the CEO regarding the digital camera division?

Additional Information: (to be given to you if asked)


▪ Kodak sells digital camera through various retail channels: Wal-Mart, Best Buy, RadioShack, Circuit City, on-line stores
like Amazon, photo shops, etc.
▪ Kodak does have different segments of digital cameras, but for the purpose of this case, we will assume that on average
Kodak digital camera is priced at $200.
▪ Quantity sold has been increasing, but client has been losing market share across all segments.
▪ There are 4 major competitors for the client in digital camera market:
Firm % Market Share Average Price
Canon 40% $220.00
Sony 15% $300.00
Samsung 10% $260.00
Nikon 10% $250.00
▪ 3 factors affect customer buying decision: brand image, good resolution and good lenses.
▪ Kodak has brand equity because of its presence in related markets (imaging and photographic materials and equipment,
camcorder, etc) and also use similar lenses to its competitors.
▪ Unlike large conglomerate competitors, e.g. Samsung and Sony, Kodak is not involved in any other auxiliary product or
services.
▪ Client’s cost per camera is the same as biggest competitor Canon – $205 per camera.
▪ Cost structure for Kodak and for biggest competitor Canon are:
Cost Description Competitor Canon Client Kodak
Direct material 60% 40%
Direct labor 15% 40%
SG&A (Selling, General & Administrative Expense) 10% 15%
R&D 10% 7%
Profit Margin (to be calculated by candidate) 5% -2%

▪ Client’s assembly is done in the U.S, while competitor Canon’s assembly is done in a plant in Shanghai, China.

Possible Answers:

1. Candidate may want to use a REVENUE/COST or similar framework to structure his/her analysis.
2. Do not try to probe the digital camera “average price” issue – this is a simplification, but the other issues are more important
in this case.
3. Conclude that client is losing share because of its poor resolution problem and determine by comparing costs that this is
most likely due to inferior material.
4. Use the numbers and table to quickly determine that the client is making a loss due to the expensive labor – assembly in U.S.
This is also why the client is forced to buy inferior materials.
5. Determine that by outsourcing client can upgrade its direct material quality to be competitive with market leader Canon and
make a 3% margin (R&D structure is fixed for client).

65 | P a g e
Outstanding Answer:
1. Note that closing plant will have union issues and outsourcing will have an initial investment, so a 3% margin may not be
worth the effort.
2. A low cost player (with inferior parts and outsourced assembly) may fetch high margins but margins are not likely to last.
3. Probe marketing synergies with other units – Does a presence in digital camera help the camcorder, or digital printing and
photographic film division?
4. Do not be afraid to suggest exiting the business if your analysis does not find any ‘nuggets’.
5. If you do recommend staying in the business make sure you outline why staying in a commodity type business will help the
client.

Case 08 - Cable News Network CNN Losing Money in Tucson

Consulting Firm: Monitor Group 1st round job interview.

Case Interview Questions: Your client Cable News Network (CNN) is a small holding company that owns three cable
television companies in the Northeast: (1) Rochester, New York; (2) Philadelphia, Pennsylvania;
and (3) Stamford, Connecticut. Each of these three companies is profitable, and each has been
experiencing steadily growing sales over the past few years.
However, the management feels that the Northeast is not the fastest growing area of the
country, and, therefore, acquired another cable television company in Tucson, Arizona a little
over a year ago. Despite every effort of management, the Tucson company’s sales have been
stagnant, and the company has been losing money ever since then. How would you analyze this
situation? What might be the cause of the poor performance of the Tucson cable company?
What could be done to turn around the failing Tucson cable company?
Additional Information: (to be given to you if asked)
1. Customers: The Tucson area is smaller than Philadelphia, but larger than Rochester and Stamford. Tucson’s population is
also growing at 12% per year on average. Per capita income is higher than in Philadelphia and the same as in Rochester and in
Stamford.
2. Costs: Operating costs in Tucson are essentially the same as in the other markets. The cost of programming is based on
number of subscribers and is equal across the nation. Operating costs are composed of variable items: sales staff, maintenance,
administration and marketing. Only maintenance is higher that in the other markets, due to the larger land area serviced. Fixed
costs relate to the cable lines, which is a function of physical area covered.
3. Company: The Tucson cable company has attempted marketing efforts in the past, such as free Disney programming for
one month, free HBO for one month, free hookup, etc. These programs have been modeled after the other three markets.
Cable penetration rates in the three Northeastern markets average 45%. The penetration rate in Tucson is only 20%. These
rates have been steady over the past three years in the Northeast. The penetration rate in Tucson has only risen by 2% in the
past three years in Tucson.
4. Competition: There is only one real substitute good for cable television: satellite dishes. However, many communities are
enacting legislation that limits their usage in Tucson. They are also prohibitively expensive for most people.
Possible Solution:

The real error of management results from their failure to recognize another “substitute” good: no cable television at all;
television reception is far better in the desert Southwest than in Northeastern cities. The lower penetration rate is most likely a
result of different climate conditions and lower interference in Arizona.

66 | P a g e
Case 09 - Automaker Chrysler to Produce Cars in Europe

Consulting Firm: Booz Allen Hamilton (BAH) first round summer internship interview.

Case Interview Question: Our client Chrysler is an automobile manufacturer headquartered in Auburn Hills, Michigan,
United States and owned by holding company Fiat Chrysler Automobiles. Chrysler is one of
the “Big Three” American automobile manufacturers. It sells vehicles worldwide under its
flagship Chrysler brand, as well as the Dodge, Jeep and Ram Trucks.
In the 1960s the company expanded into Europe, creating the Chrysler Europe division, formed
from the acquisition of several French, British and Spanish companies. Its Europe division is
responsible for selling cars to Europe. Recently, the Chrysler Europe division is making a huge
loss. In 2015, their total loss was USD $1.1 Billion. They want to learn the reasons for this loss
and what they need to do to turnaround the Europe division. What would you do?

Additional Information: to be provided to candidate after relevant questions


• Our client Chrysler’s Europe division sells directly to car dealers.
• There’s no material change at the cost side.
• Revenues have declined for the last 3 years from USD $5.5B in 2012 to $4B in 2015.
• Average price of a Chrysler vehicle sold in Europe has declined from $25K (in 2012) to $20K (in 2015).
• Quantity sold in Europe has also declined from 220K (in 2012) to 200K (2015).
• Our client Chrysler’s cars are mostly preferred by a niche segment who likes big and high-end cars. Our niche segment is
comparably small but it was profitable.
• Demand is still strong for our cars in Europe and we have a loyal customer base there.
• There is no information on competitors or competitor pricing.

Possible Solution:
Tip: In this case, “one of the Big Three American automobile manufacturers” is the most important statement. As we know,
Big 3s have very broad customer base and different automobiles. Starting the case aware of this point prevents you asking
irrelevant questions. Rather you can focus on the other important statement, like: “selling cars to Europe”.
First, I talked about a couple of ideas specific to the automobile industry and the nature of demand in Europe in order to find
the real problem. The interviewer liked all of them; however none of them were the real problem causing the decline in the
price. Then I found the real cause, which was a currency issue.
Currency changes. In 2012, 1 Euro was $1.3 on average, and in 2015 it was $1.1 on average. Our average price was USD $25K,
or 19.2K Euro in 2012, it is USD $20K or 18.2K Euro now. Approximately 15% decline in the value of dollar makes us worse
of while selling in Euro.
Recommendation
We found that our client Chrysler has been making loss because of the currency effect, more specifically dollar’s declining value
against Euro. Since our client Chrysler is one of the big 3 auto manufacturers, then it has manufacturing facilities also in Europe.
I recommend our client to produce cars in Europe to sell to the European market. Therefore our client doesn’t suffer from the
currency effect.
Risk
Of course, capacity is an issue here, too. If the client’s European facilities are not enough to handle additional capacity, then
our client should decide, based on a cost-benefit analysis, whether the costs of increasing capacity can be compensated by the
increasing revenue. Since this is longer term solution, in more short term, our client should try to make agreements with the
local dealers to increase the price or to receive the payment in terms of dollars. Since switching costs are higher for the dealers,
they may be willing to make an agreement according to their margins. Besides, our client should expand its sales to other
segments rather than high-end niche segment. Then there might be serious volume advantage.

67 | P a g e
Case 10 - Fitness Center Chain to Target Gen X and Gen Y
Consulting Firm: McKinsey & Company first round full time job interview.

Case Interview Question: Your client is Heavy Things Fitness, a small fitness center chain with 15 locations in a minor
metropolitan area in the United States. The chain was first established during the fitness boom
in the late 1970s. After decades of continual growth, Heavy Things Fitness has seen
membership decline by alarming rates. Over the past 5 years, it has seen total membership
numbers decrease an average of 6% annually.
The client Heavy Things Fitness has asked us to determine what is causing this trend and how
they can reverse this trend to begin growing again. Based on what you know about the fitness
industry, what do you believe is causing this membership attrition?

Possible Answers:
The focus of this case is on market segmentation and determining/targeting the optimal
segment. The candidate shoulddemonstrate an understanding that profit is the ultimate goal. Various strategies will have
different implementation costs and potential revenues, and the candidate should return to these numbers at every step.
If they ask for more information on the specific city/metropolitan area, tell them it has a population of about 1-1.5M, similar
to Providence, RI; Milwaukee, WI; or Jacksonville, FL. No other information is relevant (such as location within the US), as
this is not the focal point of the case.
Relevant Additional Information:
• Gym memberships in this city have been growing 3% each year, both in this city and in all regions of the country.
• Heavy Things Fitness gyms offer cardio rooms, free weights, and weight machines.
• Membership prices and payment plans are in line with their closest competitors.
• Heavy Things Fitness targets serious fitness enthusiasts, believing that these are the most loyal customers.
Question 1: Based on what you know about the fitness industry, what do you believe is causing this membership attrition?
Possible Answer:
Potential Causes:
• Changing fitness trends, such as towards class-based fitness or high intensity training
• Negative attributes of Heavy Things Fitness, such as poor customer service or deteriorating facilities
• Competitors offer better services, such as personal trainers or basketball courts
• Shifts in city or industry demographics, away from Heavy Things’ target demographic
Unlikely Causes:
• Decreasing city population (as the rest of the city’s gyms are seeing increasing memberships)
• Competitors offer lower cost (already established that Heavy Things’ costs are comparable to competitors
When interviewee asks for industry or client customer demographic information, provide Exhibit 1 and 2, respectively.
Exhibit 1: Local Interest in Gym Memberships, Demographic Segmentation
A. Local Interest in Gym Memberships, by Age

68 | P a g e
B. Local Interest in Gym Memberships, by Gender

Exhibit 2: Client Customers, Demographic Segmentation


A. Client Customers by Generation

B. Client Customers by Gender

69 | P a g e
Question 2: Given the information in Exhibits 1 and 2, what do you believe is the source of Heavy Thing’s declining
membership?
Clarifying Note: The legend order corresponds with the Generation graphic in Exhibit 1. In other words, the top portion of
the chart represents Seniors, and the bottom represents Millennials.
Possible Answer:
Extractable Facts:
• Local gym memberships are trending towards Gen X and Y and away from Millennials, Baby Boomers and Seniors.
• Genders seem to have shifted towards men, then back to women.
• Heavy Things’ membership is heavily concentrated in the Millennial generation.
• Heavy Things’ membership is evenly split between men and women.
Key Takeaways:
• Heavy Things’ membership gender profile parallels local industry gender profile
• Heavy Things’ membership generation profile is focused on the declining Millennial population rather than the growing Gen
X and Y populations.
Potential Conclusions:
• Target Gen X and Y customers
• Become more of a niche gym for declining millennial customer segment
Question 3: After reviewing this data, Heavy Things Fitness leadership agrees that they should target Gen X and Y customers.
What are some possible ways that Heavy Things Fitness can target these growing demographics? What would be the risks of
each of these solutions?
Possible Answer:
Good Potential Solutions:
• Marketing campaign targeting consumers aged 30-50
– Spokesperson in that age range
– Ads run in areas of interest for these consumers
– Explanation that this gym is for these consumers (a la Planet Fitness ads)
• Offer new services more to the liking of middle aged customers
• Make prices more appealing to middle aged customers
– If price is believed to be a primary concern, lower prices
– If not, raising prices would help offer more services and prevent younger customers with less disposable income from joining.
• Make the gym an exclusive club where only those members can join.
Top level candidates will address the inherent risk of cannibalization when switching from one target demographic to another,
and include solutions that will minimize such cannibalization. If the candidate does not mention cannibalization, lead them to
it. Some such solutions would be:
• Convert some existing locations to more Gen X and Gen Y-friendly establishments, while keeping the rest the same, enabling
current customers to go to another Heavy Things Fitness gym if theirs gets shut down.
• Develop a brand extension in brand new clubs that target these customers.
• Divide the physical space of each gym into two distinct gyms (like a combination Pizza Hut and Taco Bell)
Question #4: We presented your findings to the client at a recent status meeting, and Heavy Things Fitness leadership liked
the idea of a brand extension of 5 facilities targeted at Gen X and Y customers, called “Fast Class Fitness”, which will focus on
the class-based fitness programs that our client’s research has shown appeals to Gen X and Y customers. They want us to
determine whether it is more profitable to convert existing Heavy Things properties into Fast Class properties, or to expand
into new properties in which to open Fast Class centers. How would you go about it?
Pertinent Information (let the candidate brainstorm what information is necessary to calculate revenues and costs before
providing):
Revenues
• Membership dues at Heavy Things facilities = $50 per month
• Membership dues at Fast Class facilities = $75 per month

70 | P a g e
• Average memberships at Heavy Things facilities = 300
• Average memberships at Fast Class facilities = 400
Costs
• Rent of all facilities (including cost of fitness equipment), Heavy Things and Fast Class = $10K per month
• Labor at Heavy Things facilities = 2 employees at $36K per year each
• Labor at Fast Class facilities = 4 employees at $48K per year each
• Retention of members from converted Heavy Things gyms = 33% of each gym’s membership
– This is a small city, so when a club is converted, members can simply move to a neighboring location
– In other words, when a gym is closed, 1/3 of the members will join a different Heavy Things Fitness gym, but 2/3 of the
members will leave Heavy Things for a competitor.
• Assume Heavy Things can maintain these membership levels
Possible Answer:
Sample Calculations:

However, the fastest way to calculate this is to realize that the revenues and costs from Fast Classes are constant, so the only
necessary calculation is subtracting the additional cost to maintain the 5 heavy things locations ($80K) from the revenue gained
by keeping all 15 Heavy Things locations ($50K).
Either way, the correct conclusion is that it is more profitable to convert the 5 locations rather than expand.
Question 5: Mr. Olaf Dungren, the CEO of Heavy Things Fitness, is about to walk into this office, and he’d like to hear our
recommendation. How would you summarize our findings?
Possible Answer:
Answers may vary, but should include:
• Recommendation to convert five Heavy Things fitness centers to Fast Classes fitness centers.
• Justification based on the ability to attract new members and increase profitability at a more efficient rate than by expanding
to new properties.
• Potential risks to this suggestion that include five not being the optimal number to convert, increasing properties serving as
advertising that would generate additional memberships, the 33% estimate of retention being inaccurate.
• Next steps, such as hiring class teachers for the new gyms, developing an advertising campaign to build the Fast Class brand,
and hiring contractors to convert the Heavy Things gyms to Fast Class centers.

71 | P a g e
72 | P a g e
Case 01 - Gillette to Grow Share in Women’s Shaving Market

Consulting Firm: Gallup Consulting first round full time job interview.

Case Interview Question: Our client The Gillette Company is a consumer-packaged goods (CPG) company based in
Boston, Massachusetts, United States. The Gillette Company was founded by businessman
King C. Gillette in 1901 as a safety razor manufacturer. Today it has grown into global
consumer product company with multiple lines of business including toothpaste, batteries, hair
care products, etc.
The client The Gillette Company is the global market leader in almost every market they play
in. Recently, they have come to our consulting firm specifically asking how they can further
grow their market share in the razors and blades market. What do you tell them?
Additional Information: (to be provided only after relevant questions)
• Razors and blades or shaving market is divided into two segments: wet-shaving and dry-hair removal (e.g. waxing, electric,
etc).
• Wet shaving is further divided into cartridge-systems and disposables in shaving.
• Our client The Gillette Company has 70% of the men’s shaving market and 35% of the women’s cartridge-system market.

Possible Answers:
I was not given a lot of information from the interviewer during the actual case interview, so I would recommend using the
following additional data to flesh out the detail of the case as needed:
• Percentage of men who use hair removal products regularly: 95%
• Percentage of men who shave as their primary form of hair removal: 90%
• Client’s share of men’s shaving market: 70% (70% of disposables and 70% of cartridges)
• Percentage of women who use hair removal products regularly: 90%
• Percentage of women who shave as their primary form of hair removal: 70%
• Client’s share of women’s shaving market: 50% (35% of cartridges and 65% of disposables)
• Disposable systems cost on average half the amount per shave as cartridge systems.
• Cartridge shaving costs on average half the amount per hair removal as dry-hair removal.
Candidates or case recipients should then have to ask relevant questions to extract this data. An analysis could then determine
that there is little room for growth in the men’s market and the client would need to focus on growing its share in the women’s
market.
Potential strategies to explore would include:
• Grow the total women’s market – encourage more women to shave.
• Grow market share (especially the cartridge-system share) – encourage existing customers to migrate from cheaper disposables
to more expensive cartridge systems
• Increase price – introduce marketing aimed at shifting the image of shaving up-market to enable the client to sell at a higher
price. i.e. address the price discrepancy between wet & dry.
Through some basic math on sheets the interviewer gave me but then kept, I determined that the client The Gillette Company
has 50 of every 100 women who wet-shave; but only 35/100 of all women who remove hair. Certain countries had different
attitudes; people along coasts were more likely to remove hair than in middle of countries. I then listed 4 major reasons that
women weren’t wet-shaving:
• Fear of being cut
• Wet shaving considered a chore
• Masculine – shaving not seen as feminine
• Shaving considered “lower class”
Our firm took these reasons and implemented various marketing programs to change consumer perception and improve % of
total market that client had.

73 | P a g e
Case 02 - American Airlines to Devise Strategies to Spur Growth
Consulting Firm: Huron Consulting Group first round full time job interview.

Case Interview Question: Your client is American Airlines, Inc. (NASDAQ: AAL). Headquartered in Fort Worth,
Texas, United States, American Airlines is the world’s largest airline company when measured
by fleet size, revenue, number of passengers served, kilometres flown, and number of
destinations served. American Airlines, together with its regional partners, operates an extensive
international and domestic network with an average of nearly 6,700 flights per day to nearly 350
destinations in more than 50 countries. The company as of 2015 employs over 113,000 people.
As you may already know, competition is very fierce among the major airlines in the United
States, with plane ticket prices being driven steadily downward. After a year of weaker than
average growth, the management of American Airlines has hired our consulting firm to devise
a set of strategies to spur growth. How would you go about this case? Assume management’s
goal is profitable revenue growth, rather than cost cutting.

Possible Answers:
1. Case Overview
This is a qualitative version of a “growth strategy” case. There is only one calculation in the case and fairly limited quantitative
data. The goal for the candidate should be to generate a wide range of suggestions rather than look to dig very deeply into one
particular suggestion.
The interviewer or case giver should only give Exhibit 4 if the candidate notes that for routes served by few competitors, the
company could raise prices. In addition, encourage them to make assumptions where necessary. However, the candidates should
look to gain insights from the qualitative data presented in the Exhibits, but these are not exhaustive of all possible solutions.
2. Information Gathering
Additional Information: only give to candidates if requested
• Pricing handled by computer system – currently based on class of seat (first, business, economy class) and time to flight – all
inclusive in ticket price.
• The air transportation industry is highly fragmented overall as there are over ten competitors in the market, each of which
holds a relatively small share. There is no dominant player.
• However, many routes are only served by 1 – 2 airline companies.
• There are two major segments in the air transportation market: vacationers and business travelers.
• Business travelers make up a slight majority of business class seats and a vast majority of first class seats. Essentially all
economy class seats are taken by vacationers.
• For Exhibit 4, assume the variable cost with filling a seat is negligible.
3. Exhibits
EXHIBIT 1. Percentage of Respondants Rating Factor “Very Important” When Choosing Airline

74 | P a g e
EXHIBIT 2. Average Percentage of Seats Sold By Month

EXHIBIT 3. Price Sensitivity Survey – Percentage of Respondents Agreeing With Following Statements:

75 | P a g e
EXHIBIT 4. Price Elasticity – Price/seat from New York JFK Airport to Houston IAH Airport

* Assume the average capacity of American Airlines’ domestic flights is 200 seats.

4. Detailed Analysis
Any solution should look at all possible areas of revenue growth: price and volume. Within price, the candidates should likely
rule out pure price increases due to the competitiveness of the air transportation market. However, one potential solution would
be unbundling the ticket price (i.e. baggage check-in, snacks, etc. are extra, rather than part of the ticket price). In addition, they
should note that vacationers are much more price sensitive, thus these strategies will be more effective in economy class and
potentially business class.
For volume, the candidates should examine ways to increase the number of customers, both by taking customers away from
competitors and inducing new customers to enter the market. Also, they should look at ways to increase the number of flights
per customer. One suggestion would be to introduce a new frequent flyer program or loyalty program as this would increase
the number of flights per customer and make them less likely to defect to a competitor.
From Exhibit 1, the candidate should notice that quality of service is highly important to business travelers and thus this could
provide a competitive advantage.
From Exhibit 2, the candidate should notice that the airplanes are almost fully booked during the winter season (typical
vacationing season) and relatively empty during the summer. Thus, a strong suggestion would also include some ways to boost
summer sales.
Finally, from Exhibit 3, the candidate should notice that unbundling the baggage cost from the ticket price is likely to increase
sales (as stated earlier) and that improving the company’s listing on ticket price comparison websites will make them more likely
to reach the proportion of consumers who use these sites.
From Exhibit 4, since variable costs are negligible, the candidate should look to maximize revenue. First, they should find the
number of seats occupied and then multiply by price per seat to find maximum revenue:

Thus revenue is maximized at a price of $700 and an occupancy rate of 70%. If the candidate has seen Exhibit 2, a very strong
point to raise here is that occupancy is not constant over the course of the year. However, 70% appears close to the average
over the year, so they could argue that this result makes sense as an average figure.
These suggestions are not exhaustive of possible suggestions, any ideas that make sense in the highly competitive and price-
sensitive air travel market are acceptable.

76 | P a g e
5. Conclusion & Recommendation
First, the client American Airlines could look to lower its listed ticket price by making some of the additional services into an
extra charge, thus capturing the price-sensitive vacation market.
Next, they should boost the quality of their service for the first-class seats, which will provide sustainable differentiation over
the competitors.
Finally, they should look into customer loyalty or frequent flyer programs to boost overall sales and discounts or package deals
with resorts during the summer months to boost sales in the typically under-booked summer.

Case 03 - Bell MTS to Improve Customer Experience and Reduce Churn


Consulting Firm: Arthur D. Little first round full time job interview.

Case Interview Question: Your client is Bell MTS (formerly Manitoba Telecom Services), a regional telecommunications
provider in Canada. Bell MTS operates telecommunications services in the Canadian province
of Manitoba. The company’s head office is located in MTS Place on Main Street, in Downtown
Winnipeg, Manitoba.
Recently, the client Bell MTS sold Allstream Inc., their national business internet provider, to
U.S. fibre optic provider Zayo Group for $520 million in cash. The senior managements of Bell
MTS are now looking to use the cash from this deal to accelerate growth for the company. Your
consulting firm has been hired to help them develop growth strategies. How could they
accomplish this?

Possible Answers:
1. Case Overview & Suggested Framework
Since the goal of the case is growth, the candidate should be looking for ways to either (1) raise the revenue per customer, or
(2) increase the number of customers.
There are two paths that the candidate is expected to follow while solving the case. First, although they will find that they
cannot directly increase the rate of customer acquisitions, they can increase the number of customers retained in the long term
by lowering the churn rate (the percentage of customers who leave the company each month). Once they establish that customer
service is the problem, they should systematically look through each contact point between the customer and company and
suggest improvements that MTS could make. Second, to boost the ARPU, the candidate should look at the wireless spectrum
auction and decide what Bell MTS should bid for the licenses.
*Note that ARPU (average revenue per user) is a monthly figure.
2. Information Gathering (Q&A)
Additional Information: only give to candidates if requested
• The client Bell MTS has 600,000 wireless subscribers, making it the 4th largest wireless provider in Canada.
• ARPU (average revenue per user) is $45, versus $60 at the big 3 carriers (Rogers, Telus and AT&T Canada).
• The lower ARPU is from lower pricing power as MTS must offer a discount to compensate for the inconvenience of its
regional network, versus the big 3’s national networks.
• Churn is 2% per month versus the big 3’s 1.5%.
• Customer relations seen as the major driver of churn – customer service seen as being even worse than at one of the big 3.
• There is a wireless spectrum auction being held for Ontario spectrum licenses for the Greater Toronto Area (GTA).
• If MTS were to win the wireless spectrum auction, it would increase its ARPU for all existing customers to $47.
• In addition, it would gain 50,000 new customers at $50 ARPU.
• The annual upkeep on the new wireless network would be $400,000.
• MTS has a corporate discount rate of 11%.
3. Detailed Analysis
(1) Increase the number of customers

77 | P a g e
First, in order to lower churn (which over time will have the same effect as acquiring new customers), the candidate should
look at all possible interaction points between the company and customer. Some potential areas are listed below:
• initial sales,
• secondary sales (either marketing by the client or the customer wanting to change their services),
• billing (ease of payment), and,
• service/assistance (helplines, repairing broken phones etc.).
This is not an exhaustive list and the candidate should be structured in identifying contact points and creative when suggesting
improvement.
(2) Wireless spectrum auction
Second, when looking at the value of the wireless spectrum, the candidate should calculate the NPV of the wireless spectrum.
The incremental revenues come from two places.
• First, the company gains $2 per month per existing customer (ARPU goes from $45 to $47), which is a total of $2 * 600,000
= $1.2 million per month.
• Next, it gains $50 per month for 50,000 new customers, for a total of $50 * 50,000 = $2.5 million per month.
• Now, $1.2M + $2.5M = $3.7 million per month for 12 months gives $3.7M * 12 = $44.4 million for the year.
• Finally, deducting the $400,000 annual fixed cost means that profits on the new wireless spectrum will be $44 million per year.
At a 11% discount rate, the NPV of the wireless spectrum license is $44M / 11% = $400 million.
4. Conclusion & Recommendation
We determined that there are two main avenues of growth available to MTS: (1) increasing customer service to reduce churn
and (2) acquiring an Ontario wireless spectrum license.
First, the company should bid $400 million in the current auction for the Ontario spectrum license as this will allow the company
to expand its network and charge higher fees to its current customers.
Second, the company should invest the remaining $120 million cash towards several measures to improve the overall customer
experience and thus reduce churn, such as: automated billing, better use of customer data to tailor sales pitches and training for
sales staff.

Case 04 - Aetna to Grow Student Health Insurance Business

Consulting Firm: Deloitte Consulting 2nd round full time job interview.

Case Interview Question: Our client is a business unit within Aetna Inc., a large national health insurance company.
The client focuses primarily on the higher education student health insurance market, selling
medical insurance, prescription drug insurance and dental insurance to students at large
universities. The client is the market leader, both in terms of customers (~500,000) and revenue
(~$500M). The overall market for higher-education student health insurance is estimated at 2.5
Million potential customers and $2.2 Billion in potential revenue.
This market requires a unique, two-staged sales model. The “first sale” is often made to the on-
campus student health center to gain access to the student population. The “second sale” is to
the students, who are the ultimate consumers and purchasers of the insurance products. In
certain segments of the market, a broker may also act as an intermediary to the school.
The client’s operating margins have declined significantly in the face of aggressive competition and shrinking higher education
budgets. Additionally, Health Care Reform presents a potential risk for future growth and profitability – notably, students age
26 and under can remain on their parents’ health insurance (versus having to buy student health insurance) and health insurers
will be required to spend 80% of all revenue on medical expenses. The client’s current operating margins are 9%, down from
25% five years ago.
Deloitte Consulting has been engaged to identify profitable avenues of growth for the client as a part of the client’s three-year
growth strategy. Additionally, we have been asked to deliver prioritized recommendations on potential growth strategies.

78 | P a g e
Questions:
1. What are the potential avenues of growth for the student health insurance company?
2. Our client felt a large part of the addressable market was not being addressed (the small school market). They specifically
wanted a strategy for moving “down market” and going after smaller schools. At a high level, what qualitative and quantitative
criteria should the client consider when assessing this market?
3. (Quantitative problem) In developing the “down market” small school analysis, our primary concern related to the
attractiveness of this segment was whether healthy operating margins could be achieved. What type of information would you
need to calculate operating margin percentage for this segment? What is the operating margin percentage for each of the four
product/customer scenarios?
4. Given the outcome of your previous operating margin analysis, what recommendations would you develop for the client in
terms of pursuing the small school market?
5. Ultimately, multiple opportunities were identified for the client to grow, including increasing share in the core market,
pursuing small schools and selling new products to schools and to students – how would you evaluate the opportunities and
prioritize them?
Note to interviewer: Reveal one question at a time to the candidate (do not share all questions with candidate up front) as each
question resets the case for the candidate.

Possible Answers:

Question 1a. What are the potential avenues of growth for the student health insurance company?
Note to Interviewer: Look for the candidate to provide structure in assessing the growth opportunity. If the candidate starts
listing random opportunities, ask how they would structure an approach to devising ideas. This question was intentionally
developed to assess the candidate’s ability to deal with ambiguity and provide a variety of solutions based on his/her academic
and professional experience. The framework and comments below are not exhaustive.
Possible Answer:
Existing Products/Business Models New Products/Business Models
Identify new segments/channels to sell
existing products:
Pursue small school segment
New (universities) Target non-consumers (uninsured); Offer new
Customers/Markets Pursue secondary schools (private / products to new customers
charter schools)
Pursue individual students where schools
do not offer insurance
Defend and grow the core business
Develop new products that can be offered to the
Optimize or increase price
current customers (customer are both students
Enhance marketing and sales force
AND schools)
effectiveness to increase penetration of
Capture student wallet share for adjacent insurance
existing product offerings.
Existing products (tuition insurance, electronics insurance,
Steal share in the large school market
Customers/Markets auto insurance, renters insurance, etc.)
Develop additional areas of
Consider adjacent health services for current
differentiation (functionality, reliability,
students (mental health, wellness)
convenience)
Consider adjacent services for health centers
Establish cost leadership and set prices
(billing)
accordingly

Question 1b. Based on the data sheet (Exhibit #1), what are the potential avenues of growth for the student health insurance
company?

79 | P a g e
Exhibit #1: Data Sheet for Question 1

Note to Interviewer: Provide the candidate with the data sheet (Exhibit #1) after he/she provides an approach. Ask the
candidate to synthesize the comments and provide additional recommendations. The numbers in parenthesis below correspond
with the numbers on the accompanying data sheet. Note that both 1 & 2 can be analyzed as a pair, as can 3 & 4. These trends
below correlate with the charts on the data sheet.
Possible Answer:
Key Trends
• (1) Lower enrollment schools (<10K students) have a significant portion of students (42%) fragmented across thousands of
schools.
• (2) The client earns a significant portion of its revenue from schools with more than 10K students.
• (1 2) There is an opportunity for the client to diversify its current revenue base by pursuing the small school market, which is
comprised of almost 90% of all schools.
• (3) The client is not well diversified in terms of number of products offered.
• (4) The client has difficulty selling its ancillary products to both students and schools that purchase the medical product.
• (3 4) There is a significant opportunity to grow revenue by increasing the penetration of non-core products.
Question 2. Our client felt a large part of the addressable market was not being addressed (the small school market). They
specifically wanted a strategy for moving “down market” and going after smaller schools. At a high level, what qualitative and
quantitative criteria should the client consider when assessing this market?
Note to Interviewer: candidate should recognize this is a market attractiveness question and provide relevant criteria for
assessment. The market attractiveness criteria and corresponding comments provided below are NOT exhaustive. Look for the
candidate to touch on many of these areas, and possibly offer other areas for investigation.
Possible Answer:
Criteria Qualitative Quantitative
Market size Segment the market to understand potential Quantify revenue potential by considering # of
customer behavior. Consider key factors in schools, # of students, insurance requirements,
insurance buying behavior. and premium projections by segments developed.
Market growth Assess maturity of the market. Consider Historic market growth rates. Model projected
market trends and healthcare reform. number of schools currently in the market and
likelihood to join.

80 | P a g e
Competitive Conduct customer interviews, focus groups, Market share analysis. Win/loss rates for sales.
intensity voice of the customer analysis. Conduct
competitor assessment.
Strategic fit Determine how strategy required to win in N/A
new segment (cost versus differentiation)
aligns with existing business strategy.
Capability needs Understand future state capability needs Requisite investments (capital expenditures) to
from the new market. Conduct gap analysis. enter the new market.
Profitability Interview potential clients to develop Model potential margin scenarios based on various
understanding of revenue and cost revenue, market size and cost structure
assumptions. assumptions.
Likely penetration / Assess customer needs in the market. Model sensitivity analysis based on market growth,
market capture Consider client’s current share in other competitive intensity, margin and pricing
(share) segments. requirements.

Question 3a. Our primary concern related to the attractiveness of this segment was whether healthy operating margins could
be achieved. What type of information would you need to calculate operating margin percentage for this segment?
Note to Interviewer: After the candidate indicates what data he/she would need, provide the data sheet (Exhibit #2) and ask
him/her to calculate the operating margin for the four scenarios.
Exhibit 2: Data Sheet for Question 3

Possible Answer:
The candidate should ask for information about the cost structure of the company (fixed versus variable) as well as the product
pricing, penetration (number of customers) or total revenue.
Question 3b. What is the operating margin percentage for each of the four product/customer scenarios? Each scenario
presents a different mix between the product sold and the size (in terms of revenue) for the customer.
Possible Answer:
Using the data sheet (Exhibit #2), the candidate should calculate the expected costs under each of the four scenarios and then
calculate the operating margin percentage. They will need to calculate the number of customers in each scenario and apply the
fixed and variable costs.

81 | P a g e
Question 3c. (Follow-Up Prompt Questions After Analysis is Completed) What observations/trends do you notice about the
cost structure?
Possible Answer:
Key Trends:
• Fixed costs are more tenable with larger customers.
• Higher priced product cuts variable costs in half.
• Medical expenses and selling expenses are fixed regardless of the school size or product.
Question 4. Given the outcome of your previous operating margin analysis, what recommendations would you develop for
the client in terms of pursuing the small school market?
Note to Interviewer: the candidate likely will not give a simple yes or no answer in terms of pursuit – they should identify
scenarios under which it would make sense to potentially pursue the market – use the prompt questions as necessary if they are
not able to move beyond the “OK answer”.
Possible Answer:
The candidate should recognize that under the provided cost structure, none of the scenarios led to an operating margin that
was in line with the current, overall operating margin (provided in the case background section); better answers will center on
finding ways to make the market viable (by applying levers to improve profitability or inventing a new business model)
Whether to pursue How to pursue Prompt Questions (if
required)
OK Client should not N/A Should the client pursue
Answer pursue the segment the small school market?
because it is not
profitable enough.
Good Client should pursue Reduce SG&A or selling costs through standard cost Is there anything they
Answer the market if they can reduction methods such as business process redesign, could do to make the
apply different levers to contract renegotiation, etc. The candidate may suggest unprofitable small market
make it more price increases or reducing medical costs, but it’s segment more attractive?
profitable. important to note that neither is possible under Health
Care Reform.
Best Client should pursue Create an online B2C model that sells directly to students Would a different
Answer the market with a and bypasses the two intermediaries (brokers and business model be
different business schools) – similar to other industries that have appropriate for this
model. transitioned into direct, online distribution (e.g. books, market?
airplane tickets, etc.)
Question 5. Ultimately, multiple opportunities were identified for the client to grow, including increasing share in the core
market, pursuing small schools and selling new products to schools and to students – how would you evaluate the opportunities
and prioritize them?
Possible Answer:
The candidate should establish a framework for screening or prioritizing multiple opportunities. Good components would
include:

82 | P a g e
a. Strategic Impact and Fit
• Competitive differentiation
• White space / new market opportunity
• Disruptive potential (low cost)
• Address unmet marketplace needs
• Alignment with current business unit and corporate strategy
b. Financial Impact
• Revenue potential (market size)
• Income / margin
• Administrative cost reduction
• Future cost avoidance
c. Cost to Achieve
• Investment (capital) costs
• Ongoing operating costs
d. Time to Implement / Ease of Implementation
• Alignment with existing capabilities
• Innovation complexity
• Availability of partners or acquisition targets.

Case 05 - Unilever USA to Eliminate 5 Brands from Portfolio


Consulting Firm: Ernst & Young (EY) Advisory first round full time job interview.

Case Interview Question: Our client Unilever is a global consumer packaged goods (CPG) company co-headquartered
in Rotterdam, Netherlands, and London, United Kingdom. Its products include food,
beverages, cleaning agents and personal care products. One of the oldest multinational
companies, its products are available in around 190 countries. For this case, we’re focusing on
Unilever’s U.S. business only.
Unilever USA owns over 50 brands. Recently, it is looking to shed half of its brands in an effort
to boost growth, simplify its operations, be more nimble, and cut costs. How would you decide
which brands to eliminate in order for Unilever to reach its growth targets? What are the
additional considerations that the client should be aware of?

Possible Answers:
1. Case Overview
The client Unilever is looking to exit any industry or brand that it does not have a market leadership position in. Make sure the
interviewee grasps this motivation, and it informs his/her analysis of which brands to eliminate. Bonus points for the
interviewee who appreciates why the client is willing to cut its brand portfolio – in order to become more nimble in responding
to consumer trends.
2. Additional Information
• The client Unilever is a large, multinational CPG company, with ~$6B in annual sales for its U.S. operation.
• Assume Unilever USA has a 10-brand portfolio spanning beauty, personal care products, baby care, cleaning supplies, pet
food, etc. Client wants to shed 5 brands.
• If asked about any increases in costs, say there has been nothing unusual, but that Unilever will benefit from eliminating the
costs associated with maintaining “unproductive” brands.
3. Detailed Analysis
Question 1. How would you choose which 5 brands to divest from the 10-brand portfolio? Show Exhibit 1.

83 | P a g e
Exhibit 1. Unilever USA’s Brand Portfolio
Annual Profit Unilever’s market Consumer
Brand Category
sales margin share perception**
Upward-Facing Wash Beauty $980M 18% 2nd 3
Soapvasana Personal care $110M 12% 4th 4
Happy Baby Shampoo Baby care $660M 22% 3rd 4
Cleaning
Cat Cow Cleaner $415M 14% 2nd 3
supplies
Downward-Facing Dog
Pet food $915M 27% 4th 2
Food
Sun Salutations Soak Personal care $125M 29% 5th 3
Triangle Diapers Baby care $1.05B 33% 2nd 4
Cleaning
Warrior Two Detergent $1.150B 19% 1st 5
supplies
Bird of Paradise Feed Pet food $230M 26% 3rd 3
Headstand Hair Color Personal care $190M 36% 5th 2
** A proxy for brand equity, measured on a scale of 1-5, where 1 is perceived as the as cheapest and lowest-tier brand in that
market, 5 as the highest-tier brand.
• Are some metrics more important or relevant to Unilever than others?
• Reiterate that size of the business may not matter – a billion-dollar in sales brand can go on the chopping block if Unilever
does not have a market leadership position in that space.
Possible Answer:
The interviewee’s criteria for which brands to eliminate should be similar to the criteria he/she will see in Exhibit 1.
The interviewee should point out the following observations from Exhibit 1:
• Top 5 brands are bringing in ~80% of sales revenue.
• Quickly identify the obvious market leaders for Unilever USA (Warrior Two Detergent, Triangle Diapers) across all the
metrics.
• Acknowledge the trade-offs of eliminating smaller brands that may be higher-margin, have strong brand equity, or give
Unilever access to a certain retail channel.
– Interviewee does not need to calculate the annual profit of each brand; he/she can just point to the trade-off inherent in
divesting smaller brands that may have with higher margins.
• Pet food is probably not strategic fit with CPG’s overall focus on personal care products.
The client Unilever might look into eliminating the following 5 brands (and give the rationale):
• Downward-Facing Dog Food – does not have a market leadership position.
• Bird of Paradise Feed – does not have a market leadership position.
• Headstand Hair Color – relatively small annual sales and low consumer brand perception, despite being high-margin.
• Soapvasana – relatively small annual sales and low margin, but strong consumer brand perception.
• Sun Salutations Soak – relatively small annual sales and low consumer brand perception.
The interviewee should name some risks associated with divesting each of these brands, such as:
• An exit from the Pet Food market means there is less portfolio diversification for Unilever – and Downward-Facing Dog
Food is a large brand in terms of annual sales.
Question 2. After proposing to sell the 5 brands identified in Question #1, if the interviewee does not dive into next steps,
prompt him/her to discuss:
• How to calculate the sale price for the 5 brands?
• How to evaluate potential buyers?
Possible Answer:
To calculate the sale price – interviewee can suggest a Discounted Cash Flow (DCF) analysis or a Comparables analysis.
Potential buyers are:
• Other CPG firms such as Procter & Gamble (P&G) and Nestlé.
– Pro: Less aggressive in price negotiations than private equity (PE) firms
– Con: May become more competitive vis-à-vis Unilever; may end up leveraging Unilever brands’ retail channels or hiring
Unilever employees.

84 | P a g e
• PE firms specializing in turning around “orphaned” CPG brands
– Pro: Do not directly compete with Unilever
– Con: Traditionally, aggressive in extracting value from the deal.
Question 3: Suppose Unilever divests the 5 brands you identified. Propose a strategy that will drive company growth post
brand-rationalization.
Possible Answer:
This part of the case is broad, and tests the interviewee’s ability to tackle ambiguity in a structured way.
A strong candidate may suggest levers for growth such as:
• Appeal to new customer segments in the same distribution channels:
– Demographic trends – brands that target Baby Boomer or Millennials, which are growing segments
– Social-economic segmentation – for example, grow Unilever’s market share in premium or all-natural products.
• Look for new distribution channels:
– Brick-and-mortar – undertake co-branding campaigns with major big-box retailers; pharmacies; Costco/Sam’s Club, etc.
– E-commerce – grow Unilever’s market share in Amazon.com, Soap.com
4. Performance Evaluation
This case tests the interviewee’s ability to:
• Sift through a lot of information quickly and ignore the extraneous data.
• Put him/herself “in the client’s shoes” as the CPG client undergoes the process of brand rationalization, and refreshes its
growth strategy (intentionally ambiguous prompt).
A good candidate will drive the case after the 5 brands are eliminated, discussing the potential buyers and the growth strategy,
and will deliver a “Recommendation and Next Steps” slide that succinctly ties the whole case together.

Case 06 - Burberry Saw Sluggish Growth in European Market

Consulting Firm: L.E.K. Consulting first round full time job interview.

Case Interview Question: Our client Burberry Group Inc. (LSE: BRBY) is an iconic British luxury fashion house
headquartered in London, England. Established in 1856 by Thomas Burberry, originally
focusing on the development of outdoor attire, Burberry has moved on to the high fashion
market developing pattern-based scarves, trench coats, and other fashion accessories. Its
distinctive plaid indicates taste, sophistication, and wealth.
Within the past five years, Burberry launched an ambitious growth strategy — taking the
primarily European brand global. However, investors haven’t responded as positively as
anticipated. The CEO of Burberry Group has hired your consulting firm to evaluate the growth
strategy. How would you go about it? What would you recommend to the CEO?
Additional Information: (to be provided upon request)
Provide the following information if asked (key facts are in bold)
• Growth Target: To achieve 5% annual growth through entering the American, Asian, and Middle Eastern markets.
• Distribution: Luxury department stores, online, and retail stores.
• Product Divisions: Men’s, Women’s, and Accessories.
• Competitors: Gucchi, Coach, Ralph Lauren. Competitors’ revenues up 3%, slight decrease in market share.

Possible Answers:
1. Case Overview
The prompt of this case is intentionally vague. The candidate must systematically “discover” the problem. The prompt clues
the candidate to focus on growth but let the candidate take the case in any direction. Only clarify if the candidate is really
lost/stuck. It’s very important for the interviewer or case giver to read the entire case before giving it.

85 | P a g e
2. Detailed Analysis
Part 1: The candidate must first identify why the client missed its revenue target.
The candidate must specifically ask for the following pieces of information:
• Actual Revenue by year and geography
• Target Revenue by year and geography
Once the candidate asks for all of the above, provide Exhibit 1. Otherwise, just read off the items requested.
Exhibit 1: Target and Observed Revenues of Burberry
a. Revenue Targets
Year 2011 2012 2013 2014 2015
Asia 13 14 14 15 16
Europe 87 88 88 87 82
Middle East 25 27 30 31
North America 57 63 69 76
Total 180 219
b. Actual Revenue
Year 2011 2012 2013 2014 2015
Asia 13 15 15 17
Europe 88 89 88 86 80
Middle East 23 24 30 30 32
North America 57 63 70 78 89
Total 181 210
Once the candidate has correctly identified the problem: Tell him/her that revenue in the UK has declined by 30% over 5 years.
The candidate must combine the two charts (Revenue Targets and Actual Revenue) to create the view below and perform some
basic math.
Year 2011 2012 2013 2014 2015
Target Actual Target Actual Target Actual Target Actual Target Actual
Asia 13 13 14 15 14 15 15 16 16 17
Europe 87 88 88 89 88 88 87 86 82 80
Middle East 23 23 25 24 27 30 30 30 31 31
North America 57 57 63 63 69 70 76 78 89 89
Total 180 181 189 191 198 203 208 210 219 218
Key takeaways:
• Europe is the problem region.
• North America has strongest growth is helping conceal declining or sluggish growth in other markets.
Part 2: Ask the candidate: “Why do you think this has happened?” As the candidate is brainstorming, hand out Exhibit 2.
Exhibit 2: Brand Perception of Burberry, Gucci, and Coach

86 | P a g e
The candidate should layout several buckets/causes for the client’s drop in Brand Perception.
Guide the candidate to the following:
• Cheap counterfeit identical hats and scarves have flooded the UK market. Now low/middle consumers income are sporting
Burberry’s signature plaid. Wealthy customers have fled the brand.
• Ask “How would counterfeits impact the brand?” if the candidate is stuck.
Part #3: What should the client Burberry do?
The candidate should outline several options.
If the candidate asks for additional information, you can tell the candidate “I have no more information.”
If the candidate does not move towards a recommendation, say “The CEO of Burberry is about to walk in. I want you to
update the CEO.”
3. Performance Evaluation
Expected:
• The candidate outline a structured approach to identify the problem focusing on revenue.
• The candidate syntheses the exhibits providing takeaways.
Good:
• The candidate prioritizes throughout the case and provides the “why”.
• Candidate maintains composer especially when running into “dead ends”.
Excellent:
• The candidate drives the case
• All parts of case demonstrate structured thinking and are highly creative.

Case 07 - Duolingo to Offer Video Interpreter Service to Hospitals

Consulting Firm: Roland Berger Strategy Consultants first round full time job interview.

Case Interview Question: Your client Duolingo Inc. is a software and information technology (IT) startup based in
Pittsburgh, Pennsylvania, United States. The company’s main business is to provide an on--
demand video communication service that uses portable devices such as laptop web cameras or
videophones to provide sign language or foreign language interpreting services to customers in
need.
For the past 5 years, the client Duolingo has had great success in the airline industry. The
company has established business partnership with every major airline companies in the US.
Now, the CEO, also one of the co-founders of the startup, wants to grow his business even
further and wants your advice on developing a viable growth strategy. How would you go about
it? What would you recommend the CEO do?

Possible Answer:

1. Case Overview
• Market evaluation (realize that the client already dominated the airline industry and the airline industry is not growing)
• New market entry (creatively come up with other uses of this technology in other industries, such as in healthcare)
• New product marketing and evaluate pricing strategy.
2. Suggested Framework
• Current industry analysis (growth, market share, competitive landscape, etc)
• Entering a new market (competition, barrier to entry, comparative services, etc)
• New service evaluation (pros and cons, replacing existing service, etc)
• Marketing and pricing strategy (value-added service, competitive pricing, etc)

87 | P a g e
3. Gathering Data
• Is there a growing need for interpreters in the airline industry?
• What is the client’s current market share in the airline industry?
• Can the client’s technology be modified and applied to other industries?
• What is(are) the existing interpreter service(s) in the healthcare industry?
• How is the client’s service compare to these existing services?
• How much does(do) these existing service(s) charge?
• Will insurance company cover these services?
4. Additional Information (to be given after relevant questions)
• Interpreter needs in the airline industry:
– No growth in the past 5 years
• Market share:
– Remote video interpreter service dominates the market at 80%. The client Duolingo has 60% market share in the remote
video interpreter business and the rest of the market is very fragmented.
• New industries brainstorms:
– Retail
– Hospitality and travel
– Government and public sector
– Healthcare (client chooses to focus on the healthcare industry)
– Legal
– Insurance
– Utilities
– Banking and finance
– Real estate
5. Quantitative Analysis
A. Healthcare market
Demand: 1 in 7 patient needs an interpreter
Existing service: in-person interpreter
Price: $30/hour + $50 travel cost
Average time needed: 2 hours
# of patients per year in the US: 35M
# of hospitals in the US: 5,000
Calculations:
35M / 5,000 hospitals = 7,000 patients/hospital
7,000 / 7 = 1,000 patients in need of interpreter services
Cost of an interpreter = $30/hour * 2 hours + $50 = $110
1,000 * $110 = $110,000 (total cost to each hospital for in-person interpreter service)
B. Client’s video interpreter service
• New market entry: no R&D needed, existing technology can be directly used in the healthcare industry.
• Comparative services: in-person interpreter.
• Competition: no other video interpreter service available in the market.
• Who still requires an in-person interpreter: patients with severe or terminal illness, dementia, or suffering from emotional
disorder (10% of patient population).
Current fee ($/year) in each airport: $10,000 training and service charge + $0.02 per-minute fee
If hospitals decide to switch to the Client’s video interpreter service
Among the 1,000 patients in need of interpreter services:
1,000 * 10% = 100 patients still requires an in-person interpreter
1,000 * 90% = 900 patients can use video interpreter service

88 | P a g e
New cost to hospitals:
100 * $110 = $11,000 (cost for in-person required patients)
900 * $0.02/min * 120 min (2 hours) + $10,000 training and service charge = $12,160 (video interpreter service cost)
Total = $12,160 + $11,000 = $23,160 (total cost to provide interpreter service to all 1,000 patients)
New cost/Current cost = $23,160/$110,000 = 21%
Conclusion: Switching to the client’s video interpreter service will save each hospital 79% in patient interpreting service cost.
6. Recommendation
a. The client Duolingo should expand its video interpreter service into another industry, since there is no growth potential in
the airline industry.
b. The healthcare industry is very promising, because it is very costly and time-consuming for hospitals to provide in-person
interpreter services to patients.
c. If the client Duolingo applies the same pricing structure from the airline industry to the healthcare industry, it is able to save
each hospital more than 85% in patient interpreting service cost each year.
d. The client’s selling point: competitive price, value-added service, on-site service 24/7, etc.
e. Barriers to entry/risks: new technology implementation, etc.

Case 08 - Indian Pigments Manufacturer Constrained by Capacity

Consulting Firm: Oliver Wyman India Office first round full time job interview.

Case Interview Question: The client Indian Chemical Industries (ICI) is a manufacturer of commodity chemicals in
India. The company is a leading manufacturer of pigments. ICI has a wide distribution network
for efficient and effective delivery. The company has been serving its clients for the past 50
years.
The CEO of ICI has hired a team of management consultants from Oliver Wyman to see if
there are any problems with the company. The CEO also would like to hear your
recommendations regarding any corrective course of action that could help to resolve the
problem (if any) identified. How would you go about it?

Possible Answer:
Candidate: May I take a minute or two to go over my thoughts and come up with a structure?
Interviewer: Sure, go ahead.
Candidate: We can look at both the internal and external factors affecting the client company ICI. In the external factors, we
can look at market growth, market share, product portfolio and substitutes. And on the internal side, we have factors such as
the profit margins, price and volumes. Which side would you like me to start first?
Interviewer: Whatever works for you.
Candidate: OK. So how many products does the company have?
Interviewer: They have only one product line which is commodity pigments, hence there are no product mixes and there are
no substitutes available.
Candidate: OK. What is the price of the client’s pigments product? And how does the client’s price compare to competitors’?
Interviewer: Price of the product grew at 15% per annum. However, it is irrelevant as it is a commodity product and we don’t
have control over it.
Candidate: In that case can I know the market share and the growth of revenues of the company?
Interviewer: The client has a dominant position with a 55% share in this market and overall the pigments market is growing at
10% per annum in India. The client had 15% gross profit margins, while industry average was only 10%-12%.

89 | P a g e
Candidate: I see. Let’s concentrate on the market size growth and the price increase. So essentially if they were to maintain the
55% market share, then their revenues would have grown by 15% increase in price times 10% increase in market size, which is
26.5% ((1.15*1.1) – 1 = 0.265). What is the actual growth in their revenues?
Interviewer: Revenues grew only 15% per year.
Candidate: So, the market price has increased by 15% and the overall market size by 10% which should have increased their
revenues by 26.5%, but their revenues increased only by 15% actually. This could be because their sales volumes are not growing
fast enough. What is the growth in the volume of sales of the company?
Interviewer: The sales volumes have remained almost constant. What do you think might be the reasons behind this?
Candidate: One possible reason for constant sales volume could be capacity constraints.
Interviewer: Yes, in fact the client’s plant is operating at 100% capacity and they are not able to meet the additional market
demand.
Candidate: Then whatever increase in revenues happened, it was because of increase in price.
Interviewer: Yes, price was an issue here. That will be all. Thank you for your time.

Case 09 - UnitedHealth Group to Grow the Size of Its Operations


Consulting Firm: KPMG Advisory first round full time job interview.

Case Interview Question: Our client UnitedHealth Group Inc. (NYSE: UNH) is a large diversified managed health care
company based in Minnetonka, Minnesota, United States. It offers a spectrum of products and
services through two operating businesses, UnitedHealthcare and Optum, both subsidiaries of
UnitedHealth Group. The company serves approximately 70 million individuals throughout the
United States.
Beginning in 2011 and continuing into 2012 the client UnitedHealth Group implemented an
enterprise reorganization. They have brought in a team of management consultants to
determine an appropriate strategy for improving their profitability through growth in the size
of its operations. They hope to reach their profitability goals over the next three years. As one
of the consultants working on this case, what would you do?

Possible Answer:
This profitability case has no single right or wrong answer. Rather, the interviewer is interested in seeing how you approach the
problem. The key is to appear logical and to demonstrate an ability to move from the specifics of the case to the general issues
involved in improving profitability anywhere. In other words, the case isn’t about health care at all. So don’t panic if you’re not
an expert in the health care industry.
Remember that the interviewer is principally looking for you to demonstrate analytical skill: How you think, how you structure
a problem, and whether you are skilled at building a framework for thinking about a situation. You should be able to draw on
models from competitive strategy, finance, marketing, operations, organization, or behavior. The point is this, however: You
don’t need to reference them – just use them.
1. Opening
A good way to begin this case is to question whether expansion of operations is the right way to achieve their profitability goals.
• Are they currently profitable? If not, growth just further destroys value and perhaps they must attack profitability from the
cost side.
• If they are currently profitable, then they have three options: increase sales, lower costs, or both.
Don’t assume that growth is the only answer, show the interviewer that you would consider all options and understand both
drivers of profitability – cost and revenues.

90 | P a g e
2. Probing
Once you structure the options, the interviewer will lead you down the relevant path for this case, so follow the lead. If you’re
told that the client company has low costs relative to the industry, then pursue the sales growth option. One way to structure
your thinking about this problem would be to use a Growth Tree framework. Use the tree to think aloud for the interviewer
and guide your questioning:
“If the client company has two possible growth options – they can expand into existing businesses or diversify into new
business. I think that growth in existing businesses should be considered first because that’s where they have core skills. In
existing businesses they can grow their product market or consider vertical integration…”
Feel out the interviewer: Where are the opportunities? What might be some options you can readily eliminate, e.g. is vertical
integration really a viable option for a health-care provider?
Continue to work through the growth tree systematically. If you ask about opportunities for pushing new products into existing
markets and the interviewer seems very interested, pursue this path by offering suggestions for potentially viable new products.

Case 10 - Seagate Continues to Grow Its Market Leading Position


Consulting Firm: IBM Global Business Services (GBS) final round job interview.

Case Interview Question: Seagate Technology (NASDAQ: STX) is one of the world’s largest manufacturers of hard
drives and data storage devices. Headquartered in Scotts Valley, California, United States, the
company had revenues of approximately USD $ 11.4 billion in fiscal year 2010.
The client Seagate has enjoyed fast growing during the last several years. Sales grew at over
100% in the past year. However, the top management of Seagate is worried about establishing
and maintaining its position in this highly competitive market. What should the client company
do to continue to grow its leading market position?
Additional Information: (to be provided to candidate if asked)
1. Company/Product
The Seagate company offers two primary products. The first is a new disk drive with capacities
of 7,000% of existing floppy disks. The other is an even larger drive with disk capacities of 70,000% of existing floppy disks.
The company owns patent on both the disk drive and the disks.
2. Customers
▪ Household use: There are three primary uses for the product in homes:
▪ data backup
▪ augmenting an existing hard drive
▪ enabling manipulation of large files (replacing floppy disks)
▪ Industrial and specialty use: Can be used by large offices and companies for storing images and backing up large
systems.
▪ OEM (original equipment manufacturer) suppliers: Computer manufacturers purchase the disk drive to include as
built-in options in their computer products.
The market is estimated to total USD $1 billion with a current penetration rate of 5 percent.
3. Competitors
The client company Seagate has two main competitors: Western Digital and Samsung. Market share, price for the disk drives
and disks, and product specifications are shown in the following table.
Company Market Share Price $/disk (size) Seek time (ms) Transfer speed
Low Capacity Product
Client: Seagate 60% $199 $14 (100 MB) 29 ms 1.4 Mb/sec
Competitor A: Samsung 10% $359 $10 (230 MB) 17 ms 2.4 Mb/sec
Competitor B: Western Digital 25% $243 $25 (230 MB) 13.5 ms 2.4 Mb/sec
High Capacity Product
Client: Seagate 50% $500 $100 (1 GB) 12 ms 5.4 Mb/sec
Competitor A: Samsung n/a n/a n/a n/a n/a
Competitor B: Western Digital 50% $500 $104 (1.5 GB) 12 ms 2.4 Mb/sec

91 | P a g e
4. Substitute Products
Product Price $/disk (size) Seek time (ms) Transfer speed
Floppy Drive $80 $4 (1.44 MB) 150 ms slow
CD ROM $250 $50 (300 MB) 150 ms 2.4 Mb/sec
Hard Drive varies varies 10ms 10 Mb/sec
5. Suppliers

The client company Seagate single-sources almost all of its components.


6. Complements
Computers hardware and software are both considered complementary products.
7. Place/Distribution
The client company has effective distribution channels in the United States but lacks similar networks abroad.
8. Promotion
The client company currently spends slightly more than the average on advertising and promoting the products.
Possible Solution:
Use 3C/4P framework to analyze Company, Customers, Competitors, Products, Price, Promotion, Place, etc.
The candidate should touch upon a few different topics. How easily copiable are the drive and disks (the company has a patent,
but how easy is it to sidestep)? He/she should also conclude that this is a razor/razorblade situation; the company should focus
on ways to increase market penetration.
Once a market leader is established, the high switching costs and network externalities will make this leader position very harder
to overcome. The hardware and software distribution channels will not support two standards. This will be a winner-takes-all
market eventually. The client company could try to increase market penetration and market share by:
▪ Going after software distributors
▪ Going after hardware distributors
▪ Marketing aggressively to increase market share
▪ Pricing drives at or perhaps even below costs (think about razor/razorblade business mo

92 | P a g e
M&A
93 | P a g e
Case 01 - Orange UK to Merge With T-Mobile UK to Form Joint Venture

Consulting Firm: Oliver Wyman first round full-time job interview

Case Interview Question: The year is 2006. Your client is Orange UK, a mobile telecommunications services provider
in the United Kingdom. Orange UK is trying to build a relationship with Deutsche Telekom
AG’s T-Mobile UK to provide their customers with 3G broadband data service. At this time,
3G is not common yet.
Your consulting firm has been hired by Orange UK to help address two questions:
(1) What types of partnership structures should they consider? List pros and cons as well as
other factors that would determine optimal partnership structures.
(2) What would you need to know to select the ideal partnership? Considering this situation,
what kind of partnership do you think would be the best?

Possible Answers:
The goal of this case is to reach the best kind of partnership for our client Orange UK, considering M&A (company or business),
joint venture arrangements (JV) and business alliances.
The interviewer should drive the candidate to list the pros and cons of each partnership structure (1-2 mins)
(1) M&A:
pros -> more control
cons -> not flexible, need strict due diligence, high cost, some overlap of business and backend function
(2) Joint venture (JV):
pros -> can combine each strength to create “specific” new business
cons -> if the scope of business is there is a lot of room for synergy, JV might not be preferable.
(3) Business alliance:
pros -> flexible and can pay fee only per usage
cons -> difficult to control the partner (special arrangement for customized service, etc).
After they list pros and cons of the different types of relationships, probe them to ask specific questions about Orange UK and
T-Mobile UK, and ask for background information.
Additional Questions that may be asked (examples):
• Can Orange UK develop 3G technology using their own R&D? What are the challenges?
• The attractiveness of T-Mobile UK’s 3G technology. Any patent? Any competitors?
• Any synergies Orange UK can expect beyond just implementing their 3G technology after the partnership?
• The geographical area of the business for Orange UK and T-Mobile UK.
• The demands for 3G mobile broadband data service. Is it growing? New trends? Likely to be sustainable?
After the candidate discusses questions above, the interviewer can tell the candidate that the scope of the partnership with T-
Mobile UK is not going to be beyond the usage of their 3G technology.
Since our client Orange UK cannot pursue synergies beyond the initial 3G use, a business alliance is preferable. Orange UK
can start with a business alliance by paying the 3G usage fee and if additional synergies arise, other options can be considered
later.
Performance Assessment
• Average Candidate
– Can speak about general pros and cons of different partnership structures
– Provides some ideas to determine which partnership form may be best
• Good Candidate
– Complete all “average candidate” requirements
– Does not need much assistance from the interviewer
– Aggressively thinks out of the box, considers long term business needs and strategy

94 | P a g e
• Excellent Candidate
– Completes all “good candidate” requirements
– Provides more ideas from broader perspective to determine which form of partnership is best (i.e. synergy arising from —
overlapping business and geographical areas is important when considering M&A; legal considerations of JVs, etc.)

Prompt #2: It was recently discovered that another company Vodafone, the competitor of Orange UK, is going to begin
providing 3G broadband mobile service for free for its existing customers. If Orange UK starts providing a similar service, the
initial cost (CAPEX) will be $1,080M. Orange UK would like to recover this investment within 3 years.
Orange UK’s monthly cost per customer with the new 3G service will increase by $5, from $15 to $20. Current monthly fee
per customer is $40 and they have 10 Million customers.
Should our client Orange UK start the similar service as Vodafone’s?

Possible Answer:
This question is basically asking
(1) how many additional customers the client Orange UK need to acquire to recover the cost of new 3G service, and
(2) if it looks realistic.
If X is additional customers and Orange UK need to recover the initial cost in three years, the candidate can come up with the
following equation:
current profit per customer/month = $40 – $15 = $25
profit per customer/month after 3G = $40 – $20 = $20
$25 * 12 months/year * 3 years * 10M = $20 * 12 months/year * 3 years * (10M + X) – $1,080M
Solve for X -> X = 4M
To see if it is realistic to earn 4M (40% more than current 10M customers) additional customers by starting 3G service for free,
the candidate will require additional information, including:
– The current market share of Orange UK
– The market growth rate of mobile telecom business
– The importance of 3G service for customers
Additional Information to be given:
• Orange UK has 35% market share and is the 2nd largest player.
• The largest player’s market share is 40%.
• Market growth rate of mobile telecommunication in UK is 2%.
Is it realistic to assume Orange UK can increase their customers by 40% to pay back capex in 3 years?
Note: The candidate can answer yes or no but must have sufficient evidence; push back on candidates answering yes to consider
what they would need to believe. Most candidates should arrive at a “no” answer:
• Support for “NO”:
– Acquiring 40% more customers in 3 years would require about 10% more customers per year, which is far beyond the market
growth rate of 2%.
– Because the competitor Vodafone already started their free 3G service, Orange UK should rather consider another way to
differentiate themselves from Vodafone (maybe combination of 3G service and other data contents service at some cost).
Performance Assessment
• Average Candidate
– Structure the equation to come up with additional number of customers Orange UK needs to acquire with some guidance.
– Ask questions to decide if it is feasible to acquire 4M additional customers.
• Good Candidate
– Complete all “average candidate” requirements
– Does not need much assistance from the interviewer
– Ask some good questions to decide if it is feasible to acquire 4M additional customers and reasons why
• Excellent Candidate
– Completes all “Good Candidate” requirements

95 | P a g e
– Support the reasons why Orange UK should/should not start free 3G service with both quantitative (Comparing the necessary
growth rate vs. market growth rate) and qualitative thoughts (2nd largest player, which area they should play and so forth)

Case 02 - Miller Brewing Company to Merge with Molson Coors


Consulting Firm: KPMG Advisory first round full-time job interview.

Case Interview Question: Our client Miller Brewing Company is an American beer brewing company headquartered in
Milwaukee, Wisconsin. The American beer market is dominated by three major players:
Anheuser-Busch InBev with 45% market share, our client Miller Brewing Company with
20%, and Molson Coors Brewing Company with 10%. The rest of the market is comprised of
many small insignificant players.
Recently, the #2 (our client Miller Brewing Company) and #3 (Molson Coors Brewing
Company) players are going through a merger. What is going to happen to the U.S. beer market
after the merger? Can you find the synergies in the merger, and can you discuss their relative
importance?

Additional Information: (to be provided only after relevant questions)


For the sake of simplicity, let’s call the “Big Three” players Firms A, B, C:
• Firm A: Anheuser-Busch InBev, 45% market share
• Firm B: our client Miller Brewing Company, 20% market share
• Firm C: Molson Coors Brewing Company, 10% market share
The big three companies (Firms A, B, C) each have only one major manufacturing facility.
Each of the big three companies has one major brand in each of 3 areas:
(a) Flagship brands which are highly profitable,
(b) Maintenance brands which are marginally profitable, and
(c) Cash cows / “milk” brands which are non-growth but provide a steady source of revenue to the companies.

Possible Answers:
1. Case Overview & Framework
I discussed synergies in the merger, segmenting them into marketing synergies, distribution synergies, raw material sourcing
synergies, and then possible brand synergies (i.e. do we use the merger as a chance to reposition brands, shut brands down,
start new brands, etc.?).
It was clear that the interviewer wanted me to spend much of my time discussing the issue of brands and their impact.
I then suggested 3 categories of strategic alternatives for Firm B, our client Miller Brewing Company:
(a) Keep flagship brands of both Firm B and Firm C after the merger,
(b) Shut one of the two brands down after the merger,
(c) Keep one, but reposition the second instead of shutting it down completely, and
(d) Combine the brands into a totally new brand.
Of my four options described above, I assessed each one in terms of the revenues and in terms of the costs. I then was able to
come to a logical conclusion about the relative level of risk – risk of market share loss, risk of profit loss, risk of customer
rejection, etc.
The issue of Firm A stealing market share while the new merged company (Firm B + Firm C) repositioned itself was important
to recognize.
2. Detailed Analysis
The initial case question relates to the identification of synergies in a merger. I would structure my approach to this by working
through the value chain:

96 | P a g e
A. Raw Materials purchasing – there may be some small economies of scale here in the acquisition of ingredients (e.g. hops)
but given these two companies are already major players they are probably both already operating at scale so the cost savings
here would probably be minimal.
B. Production – the potential for savings here depends on how the two companies currently operate. I would want to know
whether either company has excess capacity that could be utilized/eliminated post merger and whether there is the potential
for consolidating the existing manufacturing facilities. Depending on the answer to these questions the potential for savings
could be significant.
C. Distribution – again this depends on how the two companies currently operate. Are both distribution networks currently at
full capacity? Do they both distribute in different regions, creating complementarities and the potential to expand distribution
for the products of both? Do they both have in house distribution? Or is distribution outsourced? Depending on the answer
to these questions the potential savings could be anywhere from zero to extremely significant.
D. Sales and Marketing – from the interviewer’s comments, it appears that this is the area that the interviewer wanted to focus
on. There may be the potential for some basic savings through the elimination of duplication in head office marketing roles
and perhaps the consolidation of external agencies used for marketing work. There are also the more significant potential
synergies through the streamlining of branding.
It is here that you could get into discussing what to do with each of the various brands and clearly the most important decisions
relate to what to do with the flagship brands. Depending on where the interviewer wants to focus, I would probably recommend
approaching this by saying that for each of the brands you have three alternatives:
• keep the brand as is,
• reposition the brand, or,
• terminate the brand entirely.
In making this decision for each brand you would need to consider the incremental revenue impact for that brand, the
incremental revenue impact for our other brands, and the costs of the repositioning or closing down (if that is the decision).
This is a relatively loose sub-framework for this issue, but you would expect that where you go will really be driven by the
information that you get from the interviewer.

Case 03 - Amazon Buys Video-game Streaming Site Twitch for $970M

Consulting Firm: A.T. Kearney final round full time job interview
Case Interview Question: Twitch.tv is a live streaming video platform owned by Twitch Interactive Inc. Introduced in
June 2011, the site primarily focuses on video game live streaming, including broadcasts of
eSports competitions, in addition to creative content, “real life” streams, and more recently,
music broadcasts. Content on the site can either be viewed live or via video on demand.
For this case, you’re not analysing a client situation. Imagine that you’re just a third-party
observer and don’t have access to any of the information that would not be publicly available.
Let’s go back to the end of year 2014. Recently, Amazon.com (NASDAQ: AMZN), the web
retailer and cloud computing company, bought Twitch.tv for $970 million. Was this a good
purchase for Amazon to make? How would the deal change the dynamics in online video
industry?

Possible Answers:
1. Case Overview
This is a partner-style case that a candidate usually gets in the final rounds of interviewing with a management consulting firm.
Therefore, there is no clear answer or direction to the case. This “Additional Information” section below is designed to give
the interviewer sufficient information that they can discuss the business model of Twitch with the candidate and see what
direction that the candidate takes it.
Note that the candidate does not need to draw out all of the information listed below to do well in the case – in fact, they
probably will not. The goal would be to discuss different ways that Amazon could create value through their ownership of the
streaming website Twitch.tv.

97 | P a g e
2. Information Gathering
Additional Information: only give to candidates if requested, or if relevant to move discussion forward.
• Amazon.com is a major electronic commerce and cloud computing company based in Seattle, Washington, with revenues of
around $89B in 2014 and fairly low profits (it is only slightly profitable on average). Amazon has around 80M monthly unique
visitors in the USA, and around 170M monthly visitors globally in 2014.
• Twitch.tv is a video streaming website, where the video is played live, rather than viewing clips uploaded in the past (the
YouTube method of video). In October 2013, the website had 45 million unique viewers.
• YouTube was acquired by Google in 2006 for $1.65 billion, at the time it had around 100 million video views per day, today
it generates over 1 billion video views per day.
• Although YouTube brings in a significant amount of revenue, it has not become profitable, given the high costs of agreements
with content partners and general site upkeep.
• Twitch.tv currently (at the end of year 2014) has about 80 million average monthly viewers, up from 50M one year ago.
• Twitch.tv’s largest demographic is males aged 18-30, who make up more than 50% of the viewership.
• The three largest categories of videos on Twitch.tv are: (i) Video Games, (ii) Live Sporting Events, (iii) Concerts and Other
Music.
• Twitch.tv is the largest distributor of video game-related content in the world.
• Each month, Twitch.tv streams an average of 16 billion minutes of video from hundreds of thousands of video creators.
• Twitch currently (in August 2014) represents 43.6% of all live-streaming content on the web, more than every major sporting
franchise and sporting broadcaster combined.
• Twitch currently (in August 2014) is also the 4th largest user of internet bandwidth (behind Netflix, Google – including
YouTube, and Apple).
• Most of the live-streaming videos have the creator interacting with the audience – the website has a chat feature that creators
can see during the stream.
• Twitch currently uses Amazon Web Services (AWS cloud platform) for their secondary systems, and their own servers for
their main systems.
• Their most successful event had 32M viewers, three times higher than the series finale of “Breaking Bad”.
• The vast majority of Twitch’s revenues come from selling advertising to companies looking to reach their viewers (a small
portion comes from premium subscriptions).
• Some industry experts have estimated that Twitch charges around an 85% premium for advertising space than close
competitors (e.g., YouTube, Facebook).
• Any videos on the Twitch.tv site ‘demo’ different products live by reputable and influential creators, similar to how many
YouTube videos do (fun fact: many people familiar with the industry believe that the Flappy Bird craze started with a YouTube
creator).
• Amazon has, in the past, announced their intention to enter the video game industry, stating that they plan to compete against
Microsoft’s Xbox and Sony’s PlayStation.
• Amazon is one of the biggest digital distributors of music, with hundreds of royalty agreements in place (the other two being
YouTube and iTunes).
• A variety of internet companies (Twitter and Facebook being prominent examples) have experimented with inserting ‘Buy’
buttons on their site.
• Amazon currently offers free streaming television and movies to Amazon Prime subscribers.
• Twitch.tv allows content creators to choose when to schedule ads during their video (i.e., they can choose to have a periodic
‘commercial break’ or a single block of ads) and also what ads they would prefer to have on their videos.
3. Detailed Analysis
Although there is no single right answer to this case, here are a few observations:
• The purchase price of $970 million on a per-user basis is lower than that of YouTube ($1.65 billion). You could argue that
Twitch is unlikely to grow as much as YouTube did, but that’s difficult to predict at the current point in time.
• Live interaction between the content creators and viewers is a fundamentally different experience than a YouTube video,
where the creator can only interact through the comment section, if they do at all.
• Amazon’s main business is selling product online, not advertising (unlike YouTube), thus to support Amazon’s core business,
they need to find some way to sell product through Twitch. One way is to develop partnerships with video creators who ‘demo’
product, so that viewers can order that product directly from the video.

98 | P a g e
• Since Twitch may run into copyright complaints, much as YouTube did in its early life, Amazon will have the clout to negotiate
more favorable agreements on its behalf.
• All of the video categories that are the most popular on the Twitch.tv website tend to have merchandise sales associated with
them – so there are many opportunities for Amazon to sell their products to Twitch viewers.
• Twitch.tv’s dominance in the field of live-streaming means that there might be an opportunity to migrate some of the live
sports from the sports franchises onto Twitch’s website.
• Twitch.tv’s premium subscription could be integrated / merged with the Amazon Prime subscription, offering greater benefits
to both groups.
• If Amazon does decide to enter the video game industry, having the world’s largest video content distributor would grant a
massive marketing advantage – they could ‘demo’ people playing their console live on the platform.

Case 04 - De Beers Group to Enter Canadian Diamond Market

Consulting Firm: Alvarez Marsal first round full time job interview.
Case Interview Question: Our client, The De Beers Group of Companies, is an international corporation headquartered
in London, United Kingdom that specializes in diamond exploration, diamond mining, diamond retail, diamond trading and
industrial diamond manufacturing sectors. The company has business operations in 35 countries and mining takes place in
Botswana, Namibia, and South Africa.
De Beers Group, has been in the diamond industry for over 100 years. To date, all of their mining initiatives have been in
Central and Southern Africa, but De Beers Group is now looking to enter the Canadian diamond market. You have been hired
to determine how this can be accomplished. Note: They are looking to enter both the diamond mining and diamond retail
sectors in the Canadian market. How would you go about this case?

Possible Answers:
1. Case Overview & Suggested Structure
Although this is a market entry case, the candidate should notice from the phrasing of the case question that the case is not
about if they should enter the market, but rather how they should enter the Canadian diamond market. As such, they should
think about the operations of a diamond company: in particular, as prompted by the extra note, they should look at both the
mining portion and the retail portion. Within each portion, there is a calculation to choose the best option for the client. At the
conclusion of the case, there is an open-ended branding question that should force the candidate to think creatively.
2. Information Gathering
Additional Information: only give to candidates if requested
• The goal in the diamond mining sector is to maximize the absolute probability of finding diamonds.
• The client De Beers Group’s goals in the diamond retail sector are: maximize cash on hand at the end of year 1, and achieve
the best return on investment in terms of year 5 revenues.
• They have a budget of USD $250 million for the diamond mining investment and there is no benefit to spending below this
budget. They also have a budget of USD $300 million for the diamond retail market and money not spent out of this budget
will be counted as cash on hand at year-end.
• Competition in the Canadian diamond retail market is very intense – market research shows that there isn’t room for a new
diamond retailer to enter the space.
• There are three available diamond mines that the client De Beers Group could purchase: the Yellowknife mine, the Whitehorse
mine and the Watson Lake mine.

– The Yellowknife mine costs $200 million and has a 50% chance of finding diamonds.
– The Whitehorse mine costs $210 million and has a 55% chance of finding diamonds.
– The Watson Lake mine costs $220 million and has a 60% chance of finding diamonds.

99 | P a g e
• There is equipment available to help at finding diamonds: excavators and seismic sensors.

– Excavators cost $30 million and raise the probability of finding diamonds by 10% (10 percentage points).
– Seismic sensors cost $50 million and raise the probability of finding diamonds by 25% (25 percentage points).

• In the diamond retail market, the client company De Beers has identified three potential acquisition targets: People’s
Diamonds, Ben Moss, and W.K. Chan.
– People’s Diamonds will cost $250 million, has $225 million in year 1 revenues and will have a 5-year total sales
growth of 20%.
– Ben Moss will cost $275 million, has $275 million in year 1 revenues and will have a 5-year total sales growth of
20%.
– W.K Chan will cost $300 million, has $290 million in year 1 revenues and will have a 5-year total sales growth of
10%.

3. Detailed Analysis
First, looking at the diamond mining segment, since the client company has a $250 million budget for the diamond mining
investment, it can purchase seismic sensors to complement the Yellowknife mine only, and can purchase excavators to
complement any of the three mines.

Therefore, the best option is to purchase seismic sensors and the Yellowknife mine.
In the diamond retail sector, the client company wants to look at both year 1 cash on hand and the return on investment (ROI).
The cash on hand is $300 million, minus the purchase price, plus the year 1 revenues.
Acquisition People’s Diamonds Ben Moss W.K Chan
Target
Cost to Acquire $250 million $275 million $300 million
Year 1 Revenues $225 million $275 million $290 million
Cash On Hand $300+$225-$250 = $275 $300+$275-$275 = $300 $300+$290-$300 = $290
million million million

By this first metric, Ben Moss is the best choice. Turning now to ROI, the return is defined in this case as the year 5 revenues,
divided by the purchase price.
Acquisition Target People’s Diamonds Ben Moss W.K Chan
Cost to Acquire $250 million $275 million $300 million
Year 1 Revenues $225 million $275 million $290 million
Sales Growth 20% 20% 10%
Year 5 Revenues $225*(1+20%) = $270 million $275*(1+20%) = $330 million $290*(1+10%) = $319 million
ROI 270/250 = 1.08 (108%) 330/275 = 1.2 (120%) 319/300 = 1.063 (106%)

Again, Ben Moss is the best choice. Once the candidate arrives at this conclusion, ask the following question: “Although Ben
Moss appears to be a good partnership, there will be a branding issue. De Beers Group is a high-end diamond retailer and Ben
Moss is a low-end brand in the Canadian market. If De Beers Group does still acquire Ben Moss Diamonds, what are some
ways they could align Ben Moss with the De Beers Group brand?”

100 | P a g e
Potential solutions include:
• Change the name and look of the stores
• Provide a better service
• Have a marketing campaign to establish the new brand
• Establish partnerships in the community.
4. Conclusion & Recommendation
In the diamond mining sector, the client De Beers Group should purchase the Yellowknife mine and seismic sensors as this
will give them a 75% chance of finding diamonds.
In the diamond retail sector, the client De Beers Group should purchase Ben Moss as it has the highest cash on hand for year
1, as well as the best ROI by year 5.
To alter the brand, the client company should change the Ben Moss stores to De Beers Group stores, thus altering people’s
perception of it, they should also provide better service to reinforce their high-end quality.

Case 05 - Merck to Assess Potential Risk of a Bi-coastal Merger


Consulting Firm: Deloitte Consulting first round full time job interview.
Case Interview Question: Your client Merck & Co., Inc. (NYSE: MRK) is a major American pharmaceutical company
headquartered in Kenilworth, New Jersey. As of 2015, Merck & Co. is the world’s seventh
largest pharmaceutical company by market capitalization and revenue.
The client Merck & Co. has recently acquired Trius Therapeutics (NASDAQ: TSRX), a small
biotech company based in San Diego, California. They have asked you to advise them on the
merger, the structure of its sales force, and the potential risks going forward. The CEO of Merck
is coming to meet with us in about 30 minutes and would like to have a recommendation. Please
take a minute to think about the situation. How would you go about it?

Possible Answers:

1. Case Overview
This is a qualitative merger & acquisition (M&A) case where no numbers will be provided. Instead the case tests how the
candidate thinks. It is acceptable for the candidate to make many assumptions, as long as you find them reasonable. If the
candidate asks for numbers to help with their analysis, clearly tell him/her that no numbers will be provided. Strong candidates
will quickly recognize this is a qualitative case.
This case requires the interviewer to be very engaged in order to properly evaluate the candidate. Ask the candidate to walk you
through his/her thought process for solving the problem.

2. Additional Information

Provide the following additional information if requested. Candidate should make reasonable assumptions.

a. Large Pharmaceutical Company: Merck & Co.


• Headquartered in in Kenilworth, New Jersey in Northeast United States.
• Currently sells cardiovascular, diabetes, and oncology drugs.
• In the process of developing additional drugs in these therapeutic areas.
• Large, experienced sales force.
b. Small Biotech Company: Trius Therapeutics
• Located in San Diego, California, United States.
• One Autoimmune drug was approved by the FDA last year while two more drugs are currently in early stages of drug

101 | P a g e
development.
• Small, inexperienced sales force.

3. Detailed Analysis

The interviewer should have the candidate talk through the implications of the merger and what sort of impact it will have on
the corporation as a whole, as well as the sales force.
a. Corporate considerations:
• Human resources
• Profitability of new and old business, expenses, etc…
• Efficiencies (or lack there-of)
• Physical location of employees
• Manufacturing facilities, etc…
b. Sales force:
• Experienced vs. inexperienced sales reps
• Geographic location
• Impact on customers (physicians, payers, patients)
• Mix of products the current reps sell
• How should the sales force be organized going forward?
c. Risks:
• Inefficiencies due to bi-coastal merger
• High costs associated with employee relocation
• High employee turnover due to merger
• PR associated with merger/acquisition
• Regulatory risks associated with acquisitions, drug development, and drug sales
• Potential supply chain issues
• Selling issues due to sub-optimal sales force structure
Note to Interviewer: if the candidate has not thought about risks and strengths of the proposed merger, directly ask them to
do so now.
Prompt #2: How can you maximize the strengths of the merger? How can you minimize the risks presented by the merger?
Possible Answer:
Again, this is a purely qualitative response. The answers to this prompt will vary greatly depending on what risks and strengths
they listed in the previous prompt.
Push back on their reasoning often and to varying degrees of intensity, e.g. “Why do you think that?”, “Are you sure?”, “I
wouldn’t start there, but this is your case”, etc.
You are testing for poise, confidence, logic, and candidates being able to defend their position.
Prompt #3: The CEO of the Large Pharmaceutical Company is about to come in the room. What are you going to recommend
to them, and why?
Give the candidate sufficient time to create a C-level recommendation. A few minutes are acceptable.
4. Performance Evaluation
Expected:
• Candidate will develop a structured approach to finding a solution.
• Assumptions will be clarified and based on reasonable logic.
Good:
• Candidate will develop a structured approach as well as touch on many of the items listed on the previous slide. A good
candidate will clearly identify many of the corporate issues, sales force issues, as well as overall risks.

102 | P a g e
Excellent:
• In addition to the above, the candidate will add his/her own insight and provide a clear, logical, and reasonable
recommendation to the CEO.
• An excellent candidate will be comfortable with the case even though there is a clear lack of structure and numbers.
• Will maintain poise under pressure from the interviewer.

Case 06 - Gillette to Double Sales in 2 Years Post Acquisition


Consulting Firm: Boston Consulting Group (BCG) first round full time job interview.
Case Interview Question: You are the Vice President of market development for the Gillette Company, an American
consumer packaged goods (CPG) company headquartered in Boston, Massachusetts. The
Gillette Company was founded by King C. Gillette in 1901 as a safety razor manufacturer.
Nowadays the Gillette Company sells four major product lines generating $500 Million in
annual revenues:
(1) shaving products
(2) skin care (with skin protective and sun-blocking properties)
(3) baby care bottles (impact resistant)
(4) tampons (very absorbent features)
The year is 2006. Your company Gillette has just been acquired by Procter & Gamble (NYSE:
PG), a global consumer goods company that makes a wide range of products including pet foods, cleaning agents, personal
care products, and shaving products. Most of Procter & Gamble’s brands are global products available on several continents.
How would you recommend increasing the Gillette Company’s sales from $500 Million to $1 Billion in 2 years post acquisition?

Possible Answer:
Post acquisition, there are potential synergies in distribution between Procter & Gamble and Gillette related to shaving and
skin care products. Gillette should look to grow geographically to new markets. One criterion is to focus on areas that are
greatly exposed to the sun.
Another focus is to look at the ability of Gillette to finance the expansion so it may want to divest another product line if it is
not performing as well as sun care such as either the tampon or baby care lines.
The following additional information is provided about the various international markets:
Region Market Size (billions) Growth Trend
U.S. $50 Billion 3%
Europe $30 Billion 6%
Asia $10 Billion 12%
Latin America $10 Billion 12%

The candidate may want to ask more about the perception of Gillette’s sun care brand in each market to determine possible
performance.
Region Brand Ranking Value Proposition
U.S. #2 or #3 Functional Care
Europe #2 Aspirational U.S. lifestyle
Asia #1 Beauty and fairness
Latin America #1 or #2 Functional Care

Interviewer: Based on this information, which two geographical markets would you want to focus on first and second? Why?
The candidate then is asked what are the ways he/she will expand in each market.
Possible Answer:
Phase 1: Enter the Asian market can be first for its high growth and market potential. The Gillette Company can take advantage
of its core functionality for sun protection and sun blocking since the Asian market values maintaining fair skin to avoid suntans.

103 | P a g e
Market entry includes establishing suppliers, retail distribution, addressing the competitive reaction, marketing, any language or
cultural issues and local government regulations.
Phase 2: Enter the Latin American market based on the products’ skin care functionality.
Phase 3: Sell product to the distributors and acquire a supply chain.
Interviewer: If the retail price per case is $150, and the mark up margin by the distributor to the retailer is 50%, and the markup
from your company Gillette (manufacturer) is 100%, what is the cost of each case?
Possible Answer:
Take $150 / (1 + 50%) = $100 price to distributor.
Take $100 / (1 + 100%) = $50 cost from manufacturer
Interviewer: What are some savings and costs associated with the use of a distributor?
Possible Answer:
Significant cost savings can be accomplished by
Recognizing synergies from having Gillette use the same supplier and distribution networks already in place by Procter &
Gamble.
Cost savings from further economies of scale.
However, the main costs of discontinuing with Gillette’s current supplier are:
Contract costs of terminating the relationship short of the agreed upon expiration.
Sunk costs of the inventory that the current supplier will no longer be motivated to support as the relationship has ended.
Fees to the new supplier.

Recommended Conclusion
Gillette should look to leverage the synergies from Procter & Gamble such as similar product lines in skin care and access to
the global market using Procter & Gamble’s international suppliers and networks.
The expansion into the global markets should be evaluated along the preferences of each geographical region based on market
growth potential first in Asia and then Latin America. Each region would require different marketing strategies that match each
area’s preferences from functional to lifestyle aspirations benefit propositions.
The expansion has opportunities for significant cost savings from synergies in sharing suppliers and distributors that would
need to be above the costs of terminating Gillette’s existing relationships.

Case 07 - Dupont Enters Rechargeable Battery Market via Acquisition


Consulting Firm: Cognizant Business Consulting (CBC) final round job interview.
Case Interview Question: For this case, you are advising the CEO of DuPont (NYSE: DD), a global chemical company
based in Wilmington, Delaware, USA. The company’s manufacturing, processing, marketing,
research and development (R&D) facilities, as well as regional purchasing offices and
distribution centers are located all over the world.
As one of the largest manufacturers of basic chemicals, Dupont has been facing stagnant growth
and profits for years. Recently, they are considering the acquisition of a rechargeable battery
manufacturer to stimulate growth and profits. Is this a good idea? What are the issues they
should consider in making the final acquisition decision?

Additional Information: (to be given to you if asked)


The client Dupont is the largest national manufacturer in their product line.

104 | P a g e
Rechargeable batteries industry: (Do not give out this information unless specifically asked – this is the key to solving the case)

• Rechargeable batteries is a high-growth industry.


• Growth is being fueled by advances in information and telecommunications technology, especially the explosive
growth in the use of laptop PCs, cellular phones and other mobile devices that need to be powered by rechargeable
batteries.
• This industry is dominated by Japanese low-cost manufacturers with high volumes.
• Main customer base will be OEMs (original equipment manufacturer) of laptop PCs, tablets, cellular phones and
similar mobile equipment (ipod, MP3 players, portable video game players, GPS, etc).
• Key success factors in this industry are:
• Ability to bring product to market quickly.
• Constant innovation.
• Ability to draw a price premium for innovation before imitators and low-cost producers enter the market.

Possible Answer:
For a merger and acquisition (M&A) case, usually two key issues need to be considered: Internal and External.
1. Internal Issues

a. Fit with long-term objectives: Long-term objective of client is entry into high-growth markets.
b. Fit with core competencies:

▪ Required core competencies:


➢ Product innovation.
➢ Ability to bring to market quickly.
➢ Ability to meet strict customer requirements relating to product quality, delivery and reliability.
▪ Current core competencies:
➢ National distribution channels.
➢ Low cost chemicals manufacturing.
➢ Commodity products, low service level.
▪ Competencies that need to be acquired:
➢ R&D ability.
➢ Bringing product to market quickly.
➢ Servicing a small number of key customers.

2. External Issues
a. Competitors:

▪ Who are current competitors? – Low cost Japanese manufacturers


▪ What is competitor’s most likely response to client’s entry into the market? – Further price cutting or innovation? Talk
about game theory.

b. Customers and market growth:

▪ Who are the customers?


➢ OEMs (main market)
➢ Replacement market (insignificant share)
▪ What do customers want?
➢ Availability (reliable delivery schedules synchronized with their production schedules)
➢ Reliable product (Brand name manufacturers of electronic equipment)

105 | P a g e
➢ Constant innovation in products (miniaturization, longer battery life)
▪ What does market growth look like in the long term?
➢ Current and future profit margins? – Need to get more information
➢ Attractive growth prospects due to growth in laptop PCs and telecommunications industry

Case 08 - Fast Food Chain Great Burger to Buy Dunkin’ Donuts


Consulting Firm: McKinsey & Company final round full time job interview.

Case Interview Question: Our client is Great Burger, a fast food chain that competes head-to-head with McDonald’s,
Wendy’s, Burger King, KFC, etc. Currently, Great Burger is the fourth largest fast food chain
worldwide, measured by the number of stores in operation. As most of its competitors do,
Great Burger offers food and “combos” for the three largest meal occasions: breakfast, lunch,
and dinner.
Even though Great Burger owns some of its stores, it operates under the franchising business
model with 85% of its stores owned by franchisees. Individual franchisees own and manage
stores under the “Great Burger” brand, pay franchise fee to Great Burger, but major business
decisions, e.g., menu, look of store, are controlled by Great Burger.
As part of its growth strategy, Great Burger has analyzed some potential acquisition targets
including Canton, Massachusetts based Dunkin’ Donuts (DD), a growing doughnut producer with both a U.S. and international
store presence.
Dunkin’ Donuts operates under the franchising business model too, though a little bit differently than Great Burger. While
Great Burger franchises restaurants, Dunkin’ Donuts franchises areas or regions in which the franchisee is required to open a
certain number of stores.
Great Burger’s CEO has hired McKinsey to advise him on whether they should acquire Dunkin’ Donuts or not. How would
you go about this case? Should Great Burger acquire Dunkin’ Donuts as part of its growth strategy?

Question #1: What areas would you want to explore to determine whether Great Burger should acquire Dunkin’ Donuts
(DD)?

Possible Answer:
Some possible areas are given below. Great job if you identified several of these and perhaps others.
a. Stand alone value of DD - Growth in market for doughnuts
DD’s past and projected future sales growth (break down into growth in number of stores, and growth in same store sales)
Competition – Are there any other major national chains that are doing better than DD in terms of growth/profit. What does
this imply for future growth?
Profitability/profit margin
Capital required to fund growth (capital investment to open new stores, working capital)
b. Synergies/strategic fit
Brand quality similar? Would they enhance or detract from each other if marketed side by side?
How much overlap of customer base? (very little overlap might cause concern that brands are not compatible, too much might
imply little room to expand sales by cross-marketing)
Synergies (Hint: do not dive deep on this, as it will be covered later)
c. Management team/cultural fit
Capabilities/skills of top, middle management
Cultural fit, if very different, what percent of key management would likely be able to adjust
d. Ability to execute merger/combine companies

106 | P a g e
Great Burger experience with mergers in past/experience in integrating companies
Franchise structure differences. Detail “dive” into franchising structures. Would these different structures affect the deal?
The McKinsey team started thinking about potential synergies that could be achieved by acquiring DD. Here are some key facts
on Great Burger and DD.
Exhibit 1
Stores Great Burger Dunkin’
Donuts
Total 5,000 1,020
North America 3,500 1,000
Europe 1,000 20
Asia 400 0
Other 100 0
Annual growth in stores 10% 15%
Financials Great Burger Dunkin’ Donuts
Total store sales $5,500m $700m
Parent company revenue $1,900m $200m
Key expenses (% sales)
* Cost of sales 51% 40%
* Restaurant operating costs 24% 26%
* Restaurant property & equipment costs 4.6% 8.5%
* Corporate general & administrative costs 8% 15%
Profit as % of sales 6.3% 4.9%
Sales/stores $1.1m $0.7m
Industry average $0.9m $0.8m

Question #2: What potential synergies can you think of between Great Burger and Dunkin’ Donuts (DD)?
Possible Answer:
We are looking for a few responses similar to the ones below:
a. Lower costs
Biggest opportunity likely in corporate selling, general, and administrative expenses (SG&A) by integrating corporate
management
May be some opportunity to lower food costs with larger purchasing volume on similar food items (e.g., beverages, deep frying
oil), however overlaps may be low as ingredients are very different
Great Burger appears to have an advantage in property and equipment costs which might be leveragable to DD (e.g., superior
skills in lease negotiation)
b. Increase revenues
Sell doughnuts in Great Burger stores, or some selected Great Burger products in DD stores
Great Burger has much greater international presence thus likely has knowledge/skills to enable DD to expand outside of
North America
Great Burger may have superior skills in identifying attractive locations for stores as its sales per store are higher than industry
average, whereas DD’s is lower than industry average; might be able to leverage this when opening new DD stores to increase
DD average sales per store
Expand DD faster than it could do on own–Great Burger, as a larger company with lower debt, may have better access to
capital

Question #3: The McKinsey team thinks that with synergies, it should be possible to double DD’s U.S. market share in the
next 5 years, and that Great Burger’s access to capital will allow it to expand the number of DD stores by 2.5 times. What sales
per store will DD require in 5 years in order for Great Burger to achieve these goals? Use any data from Exhibit 1 you need,
additionally, your interviewer would provide the following assumptions for you:

107 | P a g e
Doughnut consumption/capita in the U.S. is $10/year today, and is projected to grow to $20/year in 5 years.
For ease of calculation, assume U.S. population is 300m.

Possible Answer:
You should always feel free to ask your interviewer additional questions to help you with your response.
Possible responses might include the following:
Market share today: $700M DD sales (from Exhibit 1) ÷ $3B U.S. market ($10 * 300M people) = 23% (round to 25% for
simplicity sake)
▪ U.S. market in 5 years = $20 * 300M people = $6B
▪ DD sales if double market share: 50% * $6B = $3B
▪ Per store sales: $3B / (1000 stores * 2.5) = $1.2M
Does this seem reasonable? — Yes, given it implies less than double same store sales growth and per capita consumption is
predicted to double.

Question #4: One of the synergies that the McKinsey team thinks might have a big potential is the idea of increasing the
businesses’ overall profitability by selling doughnuts in Great Burger stores. How would you assess the profitability impact of
this synergy?

Possible Answer:
Be sure you can clearly explain how the assessment you are proposing would help to answer the question posed. Some possible
answers include:
Calculate incremental revenues by selling doughnuts in Great Burger stores (calculate how many doughnuts per store, times
price per doughnut, times number of Great Burger stores)
Calculate incremental costs by selling doughnuts in Great Burger stores (costs of production, incremental number of employees,
employee training, software changes, incremental marketing and advertising, incremental cost of distribution if we cannot
produce doughnuts in house, etc.)
Calculate incremental investments. Do we need more space in each store if we think we are going to attract new customers?
Do we need to invest in store layout to have in-house doughnut production?
If your answer were to take into account cannibalization, what would be the rate of cannibalization with Great Burger offerings?
Doughnut cannibalization will be higher with breakfast products than lunch and dinner products, etc.
One way to calculate this cannibalization is to look at historic cannibalization rates with new product/offering launchings within
Great Burger stores
Might also cannibalize other DD stores if they are nearby Great Burger store–could estimate this impact by seeing historical
change in DD’s sales when competitor doughnut store opens nearby

Question #5: What would be the incremental profit per store if we think we are going to sell 50,000 doughnuts per store
at a price of $2 per doughnut at a 60 percent margin with a cannibalization rate of 10 percent of Great Burger’s sales?
Exhibit 2
Sales and profitability per store
Units of Great Burger sold per store 300,000
Sales price per unit $3 per unit
Margin 50%
Units of DD sold in Great Burger stores 50,000
Sales price per unit $2 per unit
Margin 60%
Cannibalization rate of DD products to Great Burger products 10%

Possible Answer:
While you may find that doing straightforward math problems in the context of an interview is a bit tougher, you can see that
it is just a matter of breaking the problem down. We are looking at both your ability to set the analysis up properly and then do
the math in real time.

108 | P a g e
Based on correct calculations, your response should be as follows:
Incremental profit = contribution from DD sales less contribution lost due to cannibalized Great Burger sales = 50K units *
$2/unit * 60% margin – 300K units * 10% cannibalization * $3/unit * 50% margin = $60K – 45K = 15K incremental
profit/store

Question #6: You run into the CEO of Great Burger in the hall. He asks you to summarize McKinsey’s perspective so far
on whether Great Burger should acquire DD. Pretend the interviewer is the CEO, what would you say?

Possible Answer:
You may have a slightly different list. Whatever your approach, we love to see candidates come at a problem in more than one
way, but still address the issue as directly and practically as possible.
Answers may vary, but here is an example of a good response:
Early findings lead us to believe acquiring DD would create significant value for Great Burger, and that Great Burger should
acquire DD
We believe that our client can add $15 thousand in profit per Great Burger store by selling DD in Great Burger stores. This
could mean $50 million in incremental profit for North American stores (where immediate synergies are most likely given DD
has little brand presence in rest of world)
We also believe there are other potential revenue and cost synergies that the team still needs to quantify
Once our team has quantified the incremental revenues, cost savings, and investments, we will make a recommendation on the
price you should be willing to pay.
We will also give you recommendations on what it will take to integrate the two companies in order to capture the potential
revenue and cost savings, and also to manage the different franchise structures and potentially different cultures of Great Burger
and DD.

Case 09 - United Airlines Considers Acquiring Tokyo-New York Route


Consulting Firm: Arthur D. Little 1st round full-time job interview.

Case Interview Questions: Your client United Airlines (NYSE: UAL) is one of the world’s largest airlines with 48,000
employees and a fleet of 359 aircrafts. The airline’s corporate headquarter is located in Chicago,
Illinois, and its largest hub is Chicago’s O’Hare International Airport. In May 2010, United
Airlines merged with Continental Airlines to form the world’s largest airline in revenue
passenger miles (RPMs) and second largest in fleet size and destinations after Delta Airlines.
Recently, the board of directors of United Airlines is considering acquiring an existing non-stop
route from Tokyo’s Narita International Airport to New York City’s Newark Liberty
International Airport (10,854 km, 6,745 mi). You have been hired to advise the board on this
potential acquisition. How can you determine if the acquisition of Tokyo-New York City route
is a good idea or not?

Suggested Framework:
Profitability analysis (Profits = Revenues – Costs) looks like the best approach for this acquisition case. Simply determine if
revenue less costs equals a positive profit. Then, analyze the factors that go into revenue and the factors that comprise cost to
come to a conclusion.

Possible Solution:
Revenues will be determined by occupancy rates and expected prices. Both of these will be determined by expected demand,
the competitive environment and the extent to which our client could win over passengers from competitor routes.
Operating costs will depend on expected fuel costs, incremental costs for landing rights, etc. It is also very important to estimate
the cost of cannibalization on existing Tokyo-LA, LA-New York routes. And, last but not least, it is important to note that
losing passengers to cannibalization is better than losing them to competitors.

109 | P a g e
Case 10 - Intuit to Buy Hospital Accounting Software Company
Consulting Firm: McKinsey & Company 2nd round full time job interview.
Case Interview Question: Our client Intuit Inc. (NASDAQ: INTU) is an American enterprise software company that
develops accounting, financial and tax preparation software and related services for small
businesses. It is incorporated in Delaware and headquartered in Mountain View, California. The
company’s major product is accounting program QuickBooks. QuickBooks is an accounting
software package geared mainly toward small and medium-sized businesses (generally under $1
billion in revenues) and offer on-premises accounting applications as well as cloud based
versions that accept business payments, manage and pay bills, and payroll functions.
After several strong years, Intuit is looking to make an acquisition to boost growth. The CEO
has narrowed the list of potential targets down to 3 IT companies: (1) BankWeb, (2)
HospitalAccount, and (3) Retail Inventory. Intuit has hired our consulting firm to determine
which target it should pursue. What would you recommend?

Possible Answers:
1. Case Overview
This mergers and acquisitions (M&A) case is designed so that the candidate will compare the 3 target firms on both a financial
and strategic basis. The prompt is purposefully broad; the candidate should recognize the need to analyze both elements of
each acquisition target. Ultimately, the candidate will be asked to pick a target and then suggest a buying price.
The case can be given as interviewer-led by asking the questions, or the interviewer can allow the candidate to lead by using the
information answering the questions.

2. Additional Information: (To be provided upon request)


• If asked for more information on acquisition goals: the CEO wants the acquisition to make both financial and strategic sense.
• BankWeb makes websites for regional banks, HospitalAccount makes accounting software for hospitals, and Retail Inventory
makes inventory software for clothing retail stores.
• If asked, Intuit does not work in web development, inventory tracking, or provide accounting software to hospitals.

3. Suggested Framework
a. Our client
▪ Financials
– Cash/debt: can they afford to buy?
– Profit
▪ Strategic
– Market: Geography, Market size/growth/share, Customer – who are they?
– Culture
– Capabilities: Strengths/gaps
b. Target (1) Bank Web
▪ Financials
– Multiples
– Profit: Revenue, Cost
– Synergy
▪ Acquisition price: Comparable Multiples
▪ Strategic
– Market: Geography, Market size/growth/share, Customer – who are they?
– Capabilities: Alignment?

110 | P a g e
– Competitors: Differentiators
– Ownership – want to sell?
– Culture – match?
c. Target (2) Hospital Account
▪ Financials
▪ Strategic
▪ Acquisition price
d. Target (3) Retail Inventory
▪ Financials
▪ Strategic
▪ Acquisition price
e. Other
▪ Regulatory

4. Detailed Analysis
▪ Intuit
- Strong financials; has $250 million in cash.
- Serve U.S. public companies under $1 billion in revenue.
- Across multiple industries.
- Help with reporting for SEC filings.
- Strong software development team, has offices located in both NYC and Mountain View, CA
- Considered industry leader in space.
- Culture: professional

▪ BankWeb
- Makes websites for regional commercial banks.
- Currently serving 5 regional banks on east coast of US.
- Provide groundbreaking websites that have won best website award from Forbes.
- Competes with internal IT departments and other website building companies.
- For regional banks, estimated at 10% market share.
- Flat market growth.
- Started by two college grads in 2006.
- Culture: start-up

▪ HospitalAccount
- Manufactures accounting software for hospitals.
- Developing software that accounts for disclosures required by new health care policies.
- Mid-tier quality.
- 2% market share
- Used to be 2% market growth; accelerated to 5% for the last 4 years.
- Has been run by the same CEO, the founder of the company, since 1995.
- Culture: professional.

▪ Retail Inventory
- Inventory tracking system for clothing retailers.
- Looking to expand into other retail categories with perishable goods.
- Competes well with other inventory tracking systems created by larger companies.
- 5% market share.
- Market growing at 3% CAGR.
- PE ownership, PE purchased the company 18 months ago.
- Culture: professional

111 | P a g e
▪ Other
- There are no regulatory concerns with the acquisition of any of the targets.

Question #1: From a strategic prospective, discuss the advantages and disadvantages of each acquisition target.
Possible Solution:
BankWeb
- Pros: Industry leading, excellent quality. Rapid growth.
- Cons: Flat market growth; not core to what the client Intuit does.
HospitalAccount
- Pros: Familiar capability set, positioning for new growth with healthcare disclosures, rapidly growing market.
- Cons: Middle of the road capabilities, small market share.
Retail Inventory
- Pros: Strong competitor with larger companies; poised for growth.
- Cons: Non-core capabilities, low market share.

Question #2: (Give Exhibit 1) From a financial prospective, which looks like the best acquisition target?
Exhibit #1: Financials of client Intuit and acquisition targets
Company BankWeb HospitalAccount Retail Inventory Intuit
Revenue $15M $35M $30M $100M
COGS $7.5M $28M $21M $60M
Gross Profit $7.5M $7M $9M $40M
SG&A $1.5M $7M $3M $10M
EBITDA $6M 0 $6M $30M
Sales people 15 70 30 100
Software developers 30 93 60 200
If asked:
• Do not expect synergies from BankWeb
• Do not expect synergies from Retail Inventory
• Prompt candidate to suggest synergies for HospitalAccount

Possible Solution:
Table #1: Calculations
Company BankWeb HospitalAccount Retail Inventory Intuit
as % of sales
COGS 50% 80% 70% 60%
Gross Profit 50% 20% 30% 40%
SG&A 10% 20% 10% 10%
EBITDA 40% 0% 20% 30%
Revenue/person
Sales people $1M $500K $1M $1M
Software developers $500K $375,940 $500K $500K

• BankWeb appears to be very attractive because of its high EBITDA margin (40%).
• A good candidate will recognize that synergies from the merger & acquisition (M&A) would make HospitalAccount’s
financials more attractive.
• A superior candidate will observe that HospitalAccount has far more sales people and software developers than necessary and
will make a specific suggestion as to how those numbers could be brought in line proportionally to Intuit’s numbers (look at
Revenue/person ratios for both sales people and developers)
• See below “Table #2: Revised Forecasts” (finding these precise answers is not necessary; the key is recognizing that Intuit’s
familiarity with accounting software development combined with its scale should help improve HospitalAccount’s financials).
Table #2: Revised Forecasts
Company BankWeb HospitalAccount Retail Inventory Intuit

112 | P a g e
Revenue $15M $35M $30M $100M
COGS $7.5M $35M*60% = $21M $21M $60M
Gross Profit $7.5M $14M $9M $40M
SG&A $1.5M $35M*10% = $3.5M $3M $10M
EBITDA $6M $10.5M $6M $30M
Sales people 15 $35M/$1M = 35 30 100
Software developers 30 $35M/$500K = 70 60 200

Question #3: Which target should Intuit pursue and why?

Possible Answer:
See “Table #2: Revised Forecasts”. If the expected synergies are achieved, Hospital Account will have EBITDA of $10.5M,
compared to $6M for Bank Web and Retail Inventory. Thus, Hospital Account looks like a much more attractive target.

Question #4: (Give Exhibit 2) What price should the client Intuit expect to pay for the target?
Exhibit #2: Recent Acquisitions (Sales, EBITDA, and Acquisition prices are in $ millions)
Company Market Industry Sales EBITDA Acquisition Price
Clothing Tracking US Inventory software 50 10 50
Retail Counter US Inventory software 70 15 75
Restaurant Tracker US Restaurant Inventory software 1,500 200 400
Japan Accounting Japan Accounting software 150 75 675
AccountMan US Accounting software 30 7 42
QuickCounting US Accounting software 48 12 72
Web Fast US Web development 30 8 96
Quick Web US Web development 5 2 30
Web Flash US Web development 40 10 120

Possible Solution:
Table #3: Calculations of comparable EBITDA multiple (Sales, EBITDA, and Acquisition prices are in $ millions)
Company Market Industry Sales EBITDA Acquisition Sales EBITDA Comparable
Price multiple multiple EBITDA
multiple
Clothing US Inventory 50 10 50 1.00 5.00
Tracking software
Retail Counter US Inventory 70 15 75 1.07 5.00 5
software
Restaurant US Restaurant 1,500 200 400 0.27 2.00
Tracker Inventory
software
Japan Japan Accounting 150 75 675 4.50 9.00
Accounting software
AccountMan US Accounting 30 7 42 1.40 6.00 6
software
QuickCounting US Accounting 48 12 72 1.50 6.00
software
Web Fast US Web 30 8 96 3.20 12.00
development
Quick Web US Web 5 2 30 6.00 15.00 12
development
Web Flash US Web 40 10 120 3.00 12.00
development

113 | P a g e
• The candidate should use the numbers provided in Exhibit 2 to estimate an acquisition price based on their choice of
acquisition targets.
• The candidate should develop an industry average and should ignore one of the companies from each industry.
• When calculating “Comparable EBITDA multiple”, the candidate should ignore Restaurant Tracker (too big), Japan
Accounting (wrong market), and Quick Web (too small).
Acquisition Price based on EBITDA multiple
– The comparable EBITDA multiple in accounting software industry = 6
– Expected EBITDA of Hospital Account = $10.5M
– Acquisition Price = $10.5M * 6 = $63M
Acquisition Price based on sales multiple
– The comparable sales multiple in accounting software industry = 1.4
– Sales of Hospital Account = $35M
– Acquisition Price = $35M * 1.4 = $49M
Therefore, the client Intuit should expect to pay around $49M for Hospital Account. If the owner of Hospital Account has no
intention of selling, the client Intuit could sweeten the offer, but should pay no more than $63M.

Conclusion
Sample Recommendation - Based on both strategic and financial fit, the client Intuit should pursue Hospital Account. This
acquisition will allow Intuit access to new customers while using its familiarity with the product to reduce costs at the target.
Potential Risks
▪ Cultural fit – is the CEO of HospitalAccount going to retire and is he the key to the organization?
▪ Need to examine HospitalAccount’s willingness to sell of ownership.
▪ Look at cultural fit.

Case 11 - Fiat Takes Full Control of Chrysler in $4.35 Billion Deal

Consulting Firm: Huron Consulting Group first round full time job interview.
Case Interview Question: Chrysler is one of the “Big Three” American automobile manufacturers (with General Motors
and Ford being the other two). Headquartered in Auburn Hills, Michigan, the company sells
vehicles worldwide under its flagship Chrysler brand, as well as the Dodge, Jeep, and Ram
Trucks.
Fiat Group is a major European car maker headquartered in Turin, Italy. The car company
contains many brands such as Ferrari, Maserati, Fiat, and Alfa Romeo. As of 2013, Fiat Group
is the second largest European automaker by volumes produced, and the seventh largest in the
world.
Like the other “Big Three” automobile manufacturers, Chrysler has been hit hard by the
automotive industry crisis of 2008–2010. The year is 2013. Chrysler is being acquired by Fiat
Group. The merger would create a new holding company named “Fiat Chrysler Automobiles (FCA)”. What considerations
should be made?

Possible Answers:
For every human capital case interviews, the candidate should be prepared to describe the methodology behind the consulting
engagement. One example could be:
• Evaluate – systematic and comprehensive review of existing human capital assets and policies to evaluate alignment with
organizational structure.
• Strategy – build a strategy for both short and long-term phases.
• Implementation – implement strategy, use informal and formal leadership to endorse and lead change.
• Re-evaluate – monitor the effectiveness of the implementation and its impact on human capital assets.

114 | P a g e
A strong candidate could divide the Fiat-Chrysler merger process into short and long-term issues:
a. Short-Term
▪ Internal communications
– Develop communication to inform employees of change in ownership and ensure employees of the firm’s intent.
– Deliver communication – live cast from CEO “town hall” meetings, website.
▪ Review
– As-is culture
– Shared services
– Talent
b. Long-Term
▪ Solicit Feedback
▪ Execute
– Develop new roles / functions
– Talent management to prevent talent drain
– Rationalize or eliminate redundant services
– Establish new culture
– Develop /implement incentives

Key Insights:

Human capital case interviews do not follow the same structure as traditional strategy case interviews. In human capital cases,
the interviewer is really looking for a great approach to a problem rather than the right solution. Human capital case interviews
tend to have very little or no on-the-spot calculations (although certain firms such as Mercer do) and require clear and concise
communication and approach

115 | P a g e
Guestimates
116 | P a g e
Case 01 - How to Solve Food Security Problem in India?

Consulting Firm: Bridgespan Group first round full time job interview.
Case Interview Question: Food security is a condition related to the supply of food, and individuals’ access to it. At the
1974 World Food Conference the term “food security” was defined with an emphasis on supply.
Food security, they said, is the “availability at all times of adequate world food supplies of basic
foodstuffs to sustain a steady expansion of food consumption and to offset fluctuations in
production and prices”. Later definitions of food security added demand and access issues to
the definition. The final report of the 1996 World Food Summit states that food security “exists
when all people, at all times, have physical and economic access to sufficient, safe and nutritious
food to meet their dietary needs and food preferences for an active and healthy life”.
We have been hired by the Indian government to solve the food security problem in India. How
would you go about it?

Possible Answer:
Candidate: What do you mean by food?
Interviewer: We also started there, good. If I tell you there were four baskets of food, what would be the four?
Candidate: Cereals, pulses, fruits & vegetables, and …spice?
Interviewer: No, oilseeds. Why do you think the food security problem happens?
Candidate: I think it boils down to the demand-supply gap.
Interviewer: Could you explain a bit more?
Candidate: The country’s population is rising, and food habits are changing. Agriculture still produces traditional types of food,
whereas consumption is changing amongst the new generation. Farm efficiency is also low in India. The supply is also being
affected due to changing weather because of global warming. Moreover, farmland is becoming less and less and there are certain
regulations there on changing marshland into farmland.
Interviewer: How will you estimate or quantify supply and demand? Take something – like cereals? What will you look for, in
the short term and long term, why would you choose it?
Candidate: I would start with long term, break it into short term.
Interviewer: What is long term for you – 2020, 2030, 2050?
Candidate: I would want to take 2030 – 10 or 20 year plan necessary – since you have to change a lot of macro level issues.
Interviewer: How will you estimate demand and supply?
Candidate: I would use the population data, past stats of how food consumption has been and look at other similar countries
for example China, and how their demand has changed. I would also look at income.
Candidate performed calculations taking: population = 1.2 Billion, 5% growth, next 5 years, per-capita income (real terms,
35000), extrapolate for 3 years.
Candidate: I would also look at ways to improve farming efficiency.
Interviewer: OK. Would you expect the same trends to continue at the same rate of growth, etc?
Candidate: I suppose that if there were any major regulatory changes, then there would be fundamental shift. But since our
client is the government, it would be in a position to anticipate those changes, and can hence account for them.
Interviewer: Very good. That would be all.

117 | P a g e
Case 02 - How Many Piercings Are Done in India in a Year?

Consulting Firm: Capgemini Consulting India Office first round full time job interview.

Case Interview Question: Estimate the number of piercings done in India in a year.

Possible Answer:
Candidate: Let me rephrase the case question, you want me to estimate the number of piercings done in India in a year. By
piercings, are you referring to human body piercings?
Interviewer: Yes, that’s correct.
Candidate: Which body parts are you referring to?
Interviewer: You can do ear, nose, tongue and eyebrows and miscellaneous.
Candidate: All right. Let me take a minute to structure my thought.
Interviewer: Sure go ahead.
(After a minute with a structure on the notebook)
Candidate: As you mentioned, piercings can be of 5 types: ear, nose, tongue, eyebrows and miscellaneous. I would like to deal
with each of these one by one.
Interviewer: Sure. Go ahead.
Candidate: Let us start with ears. We can divide the customers into two groups: Males and Females. Starting with females, the
category can then be divided into age groups <5 years, 5-10 years, 10-15 years and >15 years. Each age group can be divided
into urban and rural. Does the approach seem correct?
Interviewer: Yes, I think the approach is correct. However, owing to lack of time, let us skip the differentiation into urban and
rural. Can you do some quantitative analysis for me?
Candidate: Sure. In that case, I will try and estimate the number of people in each age group. Assuming an Indian population
of 1 billion and a sex ratio of 1:1 for easier calculations, the number of women in India would be 500 million.
Now, the life expectancy in India is around 60 years. Although, India is a country where the population is primarily young yet,
for the sake of calculations, I am assuming that the number of people in each age group is the same across each age group. Do
you think that is an assumption I can make?
Interviewer: Yes, that is all right.
Candidate: So, in that case, the number of women in the age group of <5 years would be 500 million / 60 * 5 = approximately
40 million. This would be the same across all the age groups <5 years, 5-10 years and 10-15 years.
Now, in India, most of the women especially in rural areas get their first ear piercing early in their life. After this, they can get
a second or third piercing. Concentrating only on the first piercing for the time being, I would assume that number of women
getting their ears pierced in the age group <5 years would be 60%, in the age group 5-10 years would be 30% (mainly coming
from urban areas) and in the age group 10-15 years would be 10%. In the >15 years age group, this number would be negligible.
Do you think this is a fair assumption?
Interviewer: Yes. Good. Please carry on. Estimate the number for any one of the categories.
Candidate: Sure. Now, obviously, these women would only get their ears pierced once. So, we have to divide the number by 5
which is the length of each age group. Additionally, they would get both their ears pierced. So, we have to multiply the number
by 2. Is that fair?
(Interviewer checks calculations)
Interviewer: Yes, that is fair. Let us now proceed to general questions now.

118 | P a g e
Case 03 - What is Annual Demand for Natural Gas in India?
Consulting Firm: A.T. Kearney first round full time job interview.

Case Interview Question: Natural gas is a naturally occurring hydrocarbon gas mixture consisting primarily of methane,
but commonly including varying amounts of other higher alkanes. Natural gas is a fossil fuel
used as a source of energy for heating, cooking, and electricity generation. It is also used as fuel
for vehicles and as a chemical feedstock in the manufacture of plastics and other commercially
important organic chemicals.
Estimate the annual demand for natural gas in India.

Possible Answer:
Candidate: To estimate the demand, we can look at aggregate per capita energy consumption of
the population and look at the share of natural gas in that.
Interviewer: This approach won’t work. Think about some other approach.
Candidate: We can focus on the various drivers of natural gas consumption.
Interviewer: What are they?
Candidate: The primary drivers are transport, domestic consumption and power generation.
Interviewer: This approach is fine, but you are missing 2 important industries.
Candidate: Another important factor is industrial consumption.
Interviewer: Right and the other important driver for natural gas demand is the feedstock in chemical manufacturing facilities
which basically use methane and natural gas as a starting point for some reactions. OK, let’s take transport and estimate the
demand from transport.
Candidate: There are 2 types of transport: personal and public transportation. In personal, I will estimate the distance traveled
by a person times the share of natural gas powered vehicles. For public, again I will estimate the total distance covered by public
vehicles. I will estimate the share of natural gas powered vehicles and use the number of such vehicles for the calculation of
total distance.
Interviewer: The approach is fine but to get a better clarity, the public transportation can again be split based on vehicle type
like buses, rails and autos. Now, let us proceed to power generation, which is a major consumer of natural gas in the country.
Estimate the demand there.
Candidate: What is the total production of power in India?
Interviewer: 150 gigawatt (GW).
Candidate: Can I get the share of thermal power production and then the share of natural gas based power production within
that?
Interviewer: The share of natural gas based power plants is 15GW.
Candidate: What is the amount of natural gas needed to produce 1GW of power?
Interviewer: You need about 1 billion cubic meters of natural gas per GW.
Candidate: Total demand will be 15 billion cubic meters.
Interviewer: This scenario was in 2014. Now estimate the demand in 2016.
Candidate: Any growth in demand can come from increase in consumption from existing power plants or if any of the existing
thermal plants are substituted by natural gas plants.
Interviewer: New natural gas plants take time to setup. But can you enlist factors that would drive the growth in the number of
natural gas plants.

119 | P a g e
Candidate:
1. Better availability of natural gas
2. Lower fuel (natural gas) cost
3. Decrease in transportation costs of natural gas
4. Environmental aspects and laws promoting natural gas
5. Better availability of plant technology
Interviewer: Let us concentrate on the first factor. I have another data for you – The current PLF (Plant Load Factor) of natural
gas plants is 60%.
Candidate: What should be the ideal PLF?
Interviewer: It should be around 90%.
Candidate: Can I know the reasons for low PLF?
Interviewer: It was because of minimal availability of fuel.
Candidate: So, we have supply side constraints. Do we have any data on a possible increase in domestic production of natural
gas?
Interviewer: It could be possible, as Reliance have found natural gas in Krishna-Godavari Basin.
Candidate: Are there any import restrictions on natural gas?
Interviewer: There are no restrictions on import, but the infrastructure for this is not adequate.
Candidate: Is work on building this infrastructure in progress?
Interviewer: There are a few natural gas terminals under construction.
Candidate: I guess these will remove the supply side constraints.
Interviewer: What would be the consumption if the supply side constraints were removed?
Candidate: Then the PLF will increase from 60% to 90% (1.5 times). Hence the total demand will be 1.5 * 15 = 22.5 billion
cubic meters.
Interviewer: Good. It was an excellent analytical analysis of the case. You also got most of the factors influencing the demand
for natural gas right. Thank you!

Case 04 - How to Draw Income Statement for a Consulting Firm?

Consulting Firm: Roland Berger Strategy Consultants first round full time job interview.

Case Interview Question: Roland Berger is a global strategy consulting firm headquartered in Munich, Germany. The
company was founded under the name Roland Berger Strategy Consultants in 1967 by Roland
Berger. In 2011, the company’s revenues were roughly USD $1.2 billion. The company, with
around 2,400 employees worldwide, is an independent partnership wholly owned by its
approximately 220 partners. Corporate organization is based on global offices, referred to as
“competence centers”, along functional and industry lines. As of 2015 it has a total of 50 offices
in 36 countries around the world.
Roland Berger operates as a generalist strategy consultancy and advises its clients on various
management issues. Roland Berger provides restructuring and marketing advice, but its practice
areas also include corporate development, corporate finance, information management,
operations strategy, performance improvement, and strategy development. While its clientele is largely in the automobile
industry and the capital goods sector, other industry specialties include energy and chemicals, engineered products and high-
tech, financial services, information communications, pharmaceuticals and health care, public services, and transportation.
Draw the income statement for Roland Berger.

120 | P a g e
Possible Answer:
Candidate: When drawing up the income statement for the firm, I would have to write down revenues and costs. Shall I start
with revenues?
Interviewer: Sure, let’s look at revenues first.
Candidate: The main source of revenue for the firm would be through consulting fees. I can estimate this in two ways – by
estimating the demand for our services, or by estimating it from the supply side. Which method do you think I should use?
Interviewer: Which method do you think is more feasible?
Candidate: I think estimating it from the supply side would be easier if we charge our clients on a per hour per consultant basis.
Is that the case?
Interviewer: Yes indeed that is the case. We charge our clients on a per hour basis.
Candidate: In that case, I am assuming that the rate charged will vary according to the position of the consultant. A principal
or a partner would charge a significantly higher rate than a junior or associate consultant. Is my assumption right?
Interviewer: Spot on. To simplify things, assume there are 4 levels of consultants: Junior Consultant, Consultant, Engagement
Manager, and Partner, and their rates are $300/hour, $500/hour, $1,000/hour and $2,000/hour, respectively.
Candidate: Alright. As far as I know, Roland Berger has 4 offices in North America: in Chicago, Detroit, Boston and Montreal.
The Chicago office has around 60 consultants, with 3 partners. Can I get some information on how they are distributed across
the hierarchy?
Interviewer: You can assume that there are 30 in the “Junior Consultant” level, 20 in the “Consultant”, 7 in the “Engagement
Manager” and as you said, 3 in the “Partner” level.
Candidate: With this information, I can calculate the number of hours that the consultants at different levels work, multiply
that with the rates and get a total number for the revenue.
Interviewer: To save time, I would not like you to do all the calculations. Could you just outline how you would go about
finding the number of work hours for the consultants?
Candidate: I can first assume that each consultant puts in around 50 hours a week, and multiply it with 52 weeks to get the
number of chargeable hours in a year. From this number, we will have to subtract hours for a number of reasons like:
Holidays taken by consultants
Time spent in between projects
Time spent on pro-bono projects or non-revenue generating projects, such as knowledge creation.
Interviewer: That seems pretty much exhaustive. Though I must say that your 50 hours per week might be a little too
conservative (chuckles). So do you think that the number you get after accounting for all this will be the same for consultants
across the hierarchy?
Candidate: No. As you go up the hierarchy, your role will gradually shift from consulting to developing business and meeting
clients. This will also lead to a lot of travelling during the week itself. Although the total number of hours put in by them might
be higher than the average, due to these factors I think their chargeable hours will be lesser than the junior consultants.
Interviewer: That’s right. This pretty much summarizes the revenue from consulting services. Now let’s move to the cost side.
What will be the different costs incurred?
Candidate: The different costs incurred would be as follows:
Salaries of consultants (and other functions)
General and Administrative Expenses
Subscription fees for international knowledge repositories, business journals, etc.
Business Development expenses – to meet new clients, and bid for projects
Marketing costs
Interviewer: That seems to be the gist of it. Since we are really running short on time, could you tell me which of these would
form the major chunk of our costs?

121 | P a g e
Candidate: The salaries would definitely the biggest chunk of all costs.
Interviewer: Yes indeed, but there’s another cost which is significantly higher than all the other costs apart from salaries. What
will that be? You might want to expand your Gener al and Admin expenses head.
Candidate: The general and admin expenses will mainly consist of computers/software, office rental and other miscellaneous
expenses. Among these, office rental will be the highest because the location of the offices are generally in big expensive cities.
Interviewer: Exactly. Office rent forms a pretty big part of our costs. I guess that should be it. Thanks for your time.

Case 05 - More People Play Football Than Watch Football Match in UK

Consulting Firm: Oliver Wyman 2nd round job interview.

Case Interview Question: In an average week, do more people play football (soccer), or watch a football match in the
United Kingdom (UK)?

Possible Answer:
More people play football than watch a football match in the UK.
At first sight this looks like a brainteaser question. The job candidate should realize that it can
be solved as an estimation/guesstimate or market sizing case. The key to approaching this type
of case questions is: to start by clarifying the question; then to identify any assumptions you can
make to simplify the problem; next to break down the problem logically, before finally getting
into the detail of the numbers. Interviewers will be impressed with people who can approach a
problem conceptually, and also who can find ways of getting to a roughly right or “80/20”
version of the truth quickly, which can be refined later.
1. Start by raising any questions you have to clarify the parameters of the analysis.
In this case, you might want to confirm that “watching a football match” means watching a match in person, not on TV; and
that “playing in a match” means playing in an organized manner with 2 full teams and a defined pitch, rather than a kick-about
with friends in the local park, or with your kids in the garden. Football could include 5-a-side as well as full XIs, but does not
include non-standard football, e.g. Eton Fives or other variants.
2. There are normally some simplifying assumptions to make up front – and you should always lay these out to the interviewer
and confirm she/he is happy with them, before proceeding. The assumptions in this example could be:
Assume we are talking about people in England – and that the conclusion for England will be the same as for the rest of the
U.K., as there is no structural reason to believe the other nations behave differently.
Assume we are talking about distinct individuals rather than played occasions – so people playing > once a week or watching
> once a week does not count.
Assume you only need to prove that the number playing is greater / less than the number watching – rather than the absolute
numbers involved in each.
3. Once you’ve clarified the scope and agreed your simplifying assumptions, you can start laying out a logical structure to
breaking down the problem.
For this case question, one approach would be to start by listing out all the different playing and watching “occasions”. This
allows you to show your interviewer that you’ve considered the problem as a whole, rather than diving down a particular line
of argument – and also allows them to alert you to anything you’ve missed. Don’t be afraid to use a piece of paper to write
these down.
For example, an occasion list might look like: Children, School PE lessons, Inter-school matches, Other children’s matches
(county or other local leagues), Adults, Adult amateur: 5-a-side, local Sunday leagues, company teams etc, Adult professional:
Premier League etc.

122 | P a g e
4. Once you laid out the occasions, you should then prioritize which ones to analyse, in this case, based on:
(a) Do you believe the numbers of players v.s. spectators will be asymmetric, or do the players and spectators cancel each other
out? If canceling out, you can ignore the occasion, for the purposes of the analysis (remember, you don’t need to know how
many, just more or less).
(b) Which occasions contribute the most to total player or spectator numbers: you should always focus your efforts on the “big
ticket” items which will have the most impact on the final solution.
A “prioritization matrix” might look like:
Occasion Number Playing Number Watching Priority to analyse
School PE lessons Significant None High
Inter-school matches Cancel out Cancel out Low
Other children Cancel out Cancel out Low
Adult amateur Unknown None Low
Adult professional Likely to be low Significant High

The logic behind this prioritization is as follows:


Most children (especially boys) will play football at school, but there are rarely any non-playing spectators other than the PE
teacher.
A child playing in a match (school or other league) will on average bring 1 parent or other friend to watch – so the number
playing and the number watching will cancel out.
Practically no-one watches adults play amateur football on a regular basis.
Professional, i.e. League, football players are a small proportion of all adults playing football – and certainly in proportion to
the number of spectators.
This leaves us with two occasions in favour of players – School PE (significant) and Adult Amateur (unknown); and one
occasion in favour of spectators – Adult Professional (significant). Therefore, if you can prove that School PE is greater than
Adult Professional, you can answer the question, without needing to work out the number of Adult Amateur players (currently
unknown).
I. First, start by working out how many children play football in school.
(a) How many school-age children are there in England?
English population = 50M (you would be expected to know this / know that the total UK population is around 60M, and most
of this is in England).
Population in each age bracket (i.e. one year) = 50M / 75 (average life expectancy) = 2 / 3 M.
Years of school playing regular matches = 9 (Ages 7 – 16, as most don’t start PE in their first few years at school, and tend to
stop playing in Sixth Form, or leave school).
Therefore, total children = 2 / 3 M * 9 = 6 M.
There are actually 7.5M children in school in the UK, so 6m of match-playing age in England seems sensible.
(b) Of school-age children, how many play football?
Remember, you are looking for the minimum likely number, to prove it is more than people watching football, rather than the
actual. You should also bear in mind that the proportion of children playing is very different for girls than boys. A cautious
minimum estimate could be:
Proportion of girls playing football = 0%
Proportion of boys playing football = 25%
Therefore, total players = 0% * (6M/2) + 25% * (6M/2) = 750,000
Therefore, the minimum likely number of people playing football in an average week is 750,000.
II. Next, work out how many people watch professional football.
There are two dimensions to the total number of spectators: number of matches * number of spectators per match.
(a) How many professional matches are there per week?

123 | P a g e
Assume that each professional team plays each week, then number of matches = number of professional teams / 2 = 92 / 2 =
46.
You may know the number of professional teams already – but don’t worry if you don’t. The best approach is to break it down
further – into number of leagues * teams per league. If you assume there are 4 leagues, and 20 – 25 teams per league, this gives
you a range of 80 – 100 teams, or an average of 90 teams, therefore 45 matches.
The interviewer will let you know if you are completely wrong on the number: the key thing is to show a logical approach to
estimation.
(b) What is the attendance at each match?
The first think to bear in mind is that attendance will vary significantly between the different leagues – so the Premier League
would have much higher attendance than Division 2. You could start by assuming that each League has half the attendance per
match than the previous league. This means you only need to estimate the average attendance at a Premier League match.
Premier League total = 30k (this is an estimate).
Other Leagues total = 30/2 + 30/4 + 30/8 = 15k + 8k + 4k (round up) = 27k.
All Leagues total = 30k + 27k = 57k.
All Leagues average = 57k / 4 = approx 15k.
The actual number for average attendance is 15.4k, so very close to this number.
(c) What is the total average attendance?
Total attendance = matches * spectators per match = 45 * 15k = 675k (you can multiply this out longhand).
Therefore, the likely number of people watching professional football is 675k.

Conclusion: 675k is less than the 750k minimum estimate for children playing football, which does not include the number
of adults playing amateur football. Therefore, more people play football than watch football in an average week.

Case 06 - How Many People Have You Interacted within a Year?

Consulting Firm: Analysis Group first round full time job interview.

Case Interview Question: How many people have you interacted with over the last year?
Additional Information: (to be provided if interviewee asks probing questions)
• Only count each unique person once. The interviewer complained that one Harvard Business
School MBA student neglected this detail and came up with a ridiculous answer like 16,000
people)

Possible Answer:
Break into manageable subcategories and estimate them separately.
• Business school – almost 2,000 people including MBA students, faculty and administrative staff, assume that I interact with
25%, so say 500.
• Social settings – Events occur once or twice per week, more around the holidays, so say 100 events per year. The average
number of people is on the order of 20 per event. Same people can show up at different events, assume that I see
the average person 4 times: 100 events * 20 people / 4 times = 500 people. Maybe 100 of these people are
also at my business school, so round down to 400 people.
• Everyday activities – dry cleaner, supermarket, grocery shopping, car wash, favorite pizza place, post office, etc. I typically
interact with a cashier and server, so assume 2 interactions per visit. Assume 3 errands or visits per day = 20 locations per week,
average visit interval is once every two weeks, so there are 40 unique locations * 2 interactions = 80 people. Round up to 100
to account for my neighbors, doorman, my doctor, dentist, and other people I see over and over.
• Random meetings – people who stop you to ask for directions, people you talk to on the subway and people who attempt to
steal your laptop or wallet – assume 2 people per week or 100 per year.

124 | P a g e
• Other meetings – people you meet on vacation, at sporting events, shows, etc. Assume 100 people.
Total number of people in a year = 500 + 400 + 100 + 100 + 100 = approximately 1,200.

Case 07 - How Many Beer Bottles Are in Circulation in the U.S.?

Consulting Firm: IBM Global Business Services (GBS) first round full time job interview.

Case Interview Question: How many beer bottles are currently in circulation in the United States?

Possible Answer:
At first glance this case seemed like a pretty straightforward population-based market sizing case. First I decided to figure out
the annual beer consumption to get an annual consumption of beer bottles.
I estimated that the population of U.S. is around 320M (as of 2016). We would have to take out
babies and young children who don’t consume beer (approximately 10%), which gave me the
number of beer drinkers as 290M.
Then I divided it into men and women as they have different consumption patterns – men
probably consume more. I estimated the number of men and women to be approximately equal
at 145M and 145M. Then I estimated that men probably drink 2 bottles of beer a week on
average, making it approximately 100 bottles a year per person (heavy beer drinkers and men
not drinking beer will average out), giving a total of 145M * 100 = 14.5B bottles. Women
probably drink 2 bottles a month making it a total of 145M * 2 * 12 = 3.5B bottles yearly. The total yearly consumption is 14.5B
+ 3.5B = 18B bottles.
This is where the trick was because my interviewer was not satisfied with a yearly consumption number, he wanted to have a
current circulation number!
Because glass beer bottles are recycled, I used the concept of “velocity of circulation” to come up with the number: yearly
consumption = current circulation * some velocity (# of times the bottle goes through the economy). I estimated the velocity
of circulation to be around 70 assuming an average 5-day purchasing cycle. Thus the current circulation of beer bottles equals
approximately 18B / 70 = 257M bottles. I received feedback from the interviewer that this number was pretty close to the
actual number.

Case 08 - How Many Penguins Can Live in Antarctica?

Consulting Firm: Bain & Company first round full time job interview.

Case Interview Question: Penguins have evolved. Now they can talk and they are organized as an advanced society.
The ambassador of Penguinland (former Antarctica) has reached out to your consulting firm
Bain & Company because he is worried about the growing population growth of
penguins. Currently, there are about 23 million penguins living in Penguinland and the
population is growing at an astounding 32% per year.
The ambassador of Penguinland needs your help addressing two questions: (1) What is the
maximum number of penguins that can live in Penguinland and (2) what is the best way to
accommodate everyone? How would you go about it?

Possible Answers:
1. Case Overview
Important: Try not to laugh…This is a real case from Bain & Company. It’s a hard case. The candidate will have to think out
of the box to solve it. Therefore, it’s not a case for beginners.
Before the candidate start his/her framework, ask him/her what he/she knows about penguins.

125 | P a g e
The candidate must calculate how many penguins can live in Antarctica. That means, the candidate should be thinking on
questions like what the penguins eat (fish), how far can they be from the ocean, etc.
The penguins are not involved in commercial activities. They still act as penguins. Orient the candidate to think out of the box.
2. Suggested Framework
The candidate should present a framework touching on penguin’s food habits, predators, space for building their nests, etc.
The key driver in this case is how far they can live from the ocean.
3. Additional Information: (to be given upon request)
• Predators: the penguins have successfully negotiated with all predators. The polar bears now live in a Zoo in San Diego and
the whales lives at Sea World.

• Food: There are a lot of food all over the ocean. It’s more than enough to feed every penguin.
• Penguin habits:
– They are all married and have a baby penguin
– They need space for both penguins to live and a nest for the baby
– Penguins do not need space for leisure. They need space just to sleep
– Penguins must eat everyday
• Penguinland (Antarctica) area: Provide Exhibit 1. Allow the candidate to approximate the map to a square.
Exhibit 1: Penguinland Map

4. Calculations
If the candidate does not realize that he/she needs to calculate the area of Antarctica, ask him/her to do so.
Calculation:
Approximating the map to a square:
Area = 4,000 km * 4,000 km = 16,0000,000 km2
Now that the candidate has calculated the Antarctica area, he/she needs to calculate how far can a penguin live from the ocean.
He/she will need to calculate how fast can a penguin walk and how many hours per day can a penguin walk.

126 | P a g e
Additional Information to be given:
• Penguins only walk and eat during the day. They must sleep the whole night.
• Penguins must eat every day or they will die.
• Even if a penguin stays in the ocean for only 1 second, it’s enough to eat one fish.
• The longest day of the year has 22 hours of sunlight.
• The average size of penguin leg is 20cm. The average size of a human leg is 1m.
• Humans can walk at an average of 5km per hour.
The candidate must realize that the walking speed can be written as a function of the leg length. Therefore, if a penguin has a
leg that is 1/5 of a human’s, the penguin speed will also be 1/5 of the human’s speed. In this case: 1 km per hour.
Since the longest day of the year has 22 hours of sun light (and 2 hours of night), the shortest day of the year has only 2 hours
of sunlight.
Since the penguins must walk from their nest to the sea and from the sea to their nest, they cannot live from more than 1 hour
(1km) from the sea.
Now that the candidate figured that the penguins can live only 1km away from the sea, he/she needs to calculate the total
habitable area and how many penguins can be fit in this area. The total habitable area can be approximated** to 4 rectangles of
1km by 4,000km (see figure below).

Total Habitable Area = (1 x 4,000) x 4 = 16,000 km2 = 16,000 * 1,000 * 1,000 m2 = 16 billon m2
**Note: The correct area calculation is (4,000 km x 1 km) + (4,000 km x 1 km) + (3,998 km x 1 km) + (3,998 km x 1 km) =
15,996 km2, but is OK to round to 16,000 km2.
Now, orient the candidate to estimate the number of penguins. He/she will need to know the size of the penguins.
Additional Information: to be given upon request
• Penguin size = 1 m2
• Nest size = 1 m2
Remind the candidate that the penguins need space to move from their nests to the sea. Therefore, they will need roads.
Tip: the penguins will need roads. Ideally, they will need 2 roads, so some penguins can go to the sea while others are coming
back the their nests.
The optimum solution is to allocate the penguin as follow:

127 | P a g e
According to this layout, it’s possible to accommodate 3 penguins (1 male, 1 female and 1 baby) for each 5 square meters.
Total maximum number of penguins = (Habitable area / Home layout size) * 3 penguins = (16,000,000,000 m2 / 5 m2) * 3 =
9.6 billion
5. Performance Evaluation
Expected minimum requirements:
• Correctly calculates the number of penguins
• Needed some help to understand the drivers to calculate the maximum penguin population.
Good candidate:
• Completes all “Expected” requirements
• Suggests that the map can be approximated to a square or circle
• Develops a framework that contains important issues such as Predators, Feeding Habits, etc.
• Understands that the penguins must walk 1km to reach the sea and then they need to come back.
Excellent candidate:
• Completes all “Good” requirements
• Understands that the calculations must consider the shortest day of the year
• Suggests the same allocation layout
• Could solve the case without much help from the interviewer.

Case 09 - American Airlines to Increase Number of Passengers per Flight

Consulting Firm: Arthur D. Little first round full time job interview.

Case Interview Question: The client American Airlines, Inc. (AA), commonly referred to as American, is a major airline
company headquartered in Fort Worth, Texas. It is the world’s largest airline when measured
by fleet size, revenue, scheduled passenger-kilometers flown, and number of destinations
served.
In 2011, due to a downturn in the airline industry, American Airlines’ parent company AMR
Corporation filed for bankruptcy protection. In 2013, US Airways and American Airlines
merged. Eventually operations were merged under one operating certificate to create the largest
United States airline which kept the American Airlines brand name.
As a part of its recovery plan, American Airlines has asked your consulting firm to propose a
plan to increase the number of passengers per flight. How would you approach figuring out the
bottom line of increasing one passenger per aircraft in their domestic flights?

Possible Answers:
1. Case Overview
This case has a lot of calculations and information that may mislead the candidate. This information is included on purpose
and the candidate should know what he/she have to use and what can be ignored.
The goal here is to calculate the additional profit of adding one passenger per flight (domestic flights only). After the candidate
presents the framework and ask questions about cost and fares, provide Exhibit #1. If you want to make this case harder, do
not provide the exhibit. Instead, give the Exhibit’s information upon request in a Q&A session.

128 | P a g e
2. Additional Information
Table 1: Aircraft models and capacity

*1: Fixed cost per 1000 mile flight distance


Table 2: Average load factor, fares and time per flight

Table 3: Variable costs

**2: Variable costs are represented as a percentage of the round trip fare
Table 1:
• # of aircrafts: how many aircrafts AA has in its fleet.
• Capacity: number of passengers that each aircraft can fly.
• Fixed cost: fuel, maintenance and flight crew.
Table 2:
• Average load factor: average number of passengers/aircraft capacity
• Average round trip fare: this average includes first, business and coach classes
• Average time on air: time that the aircraft is actually flying
• Average time on land: average time to clean, refuel, and board the passengers
Note that the total time per flight is the average time on air + average time on land.
Table 3:
• All variable costs are a % of the round trip fare. That is, the variable cost for a domestic flight is 25% * $200 = $50, variable
cost per flight = $50 / 2 = $25

3. Calculations
There is some useless information in the Exhibit. The candidate should ignore:

• All data regarding International flights: the case is about domestic flights
• Fixed Cost: the plane is flying anyway so the profit of an extra passenger should not consider the fixed cost.

The candidate should calculate the additional profit in dollars. Therefore, he/she may also ignore the aircraft capacity, and load
factor. He/she can solve this case by using only the number of domestic aircrafts, average time on air, average time on land,
variable costs, and round trip fares.

129 | P a g e
Assumption: the company operates 24 hours a day, and 360 days per year.
The easiest way to calculate the additional profit is calculating how many domestic flights AA operates per year and the profit
per flight that the additional passenger brings.
Calculating the number of flights per year:
• # of aircrafts (domestic only): 180 + 125 + 195 = 500
• Average time per flight = average time on air + average time on land = 2 + 0.5 = 2.5 hours
• Average flights per day per aircraft = 24 hours / 2.5 hours = 9.6 (can be rounded to 10 per day)
• Total flights per year = # of aircrafts * average flights per day * 360 = 500 * 10 * 360 = 1,800,000 flights
Calculating the additional profit per flight that an additional passenger brings:
• Average fare per flight = round trip fare / 2 = $200 / 2 = $100
• Variable Cost = (10% + 10% + 5%) * $100 = $25
• Additional Profit per passenger = Revenue – Variable Cost = $100 – $25 = $75
• Additional profit per year = $75 * 1,800,000 = $135,000,000 = $135 MM
There are other ways to reach the same result. The calculations above show the easiest way. Help the candidate only if he/she
gets stuck with the numbers.

4. Bonus Question
Question: Now that we know the impact of adding one passenger per flight, what can we do to achieve this goal? What else
can be done to increase AA’s profitability?
Possible Answer:
This is a regular “what else” question. The candidate should mention some of the ideas bellow:
• Increase the # of passengers:
• Change pricing strategy
• Offer a better customer service
• Offer a better “frequent flyer” program (AA Advantage program)
Increase AA’s overall profitability:
• Negotiate fuel prices (fuel represents 30% of the operational cost)
• Negotiate with labor unions to reduce salaries/benefits for new hires
• Focus on the most profitable routes
• Discontinue unprofitable routes
This case does not require a final recommendation.
5. Performance Evaluation
Expected:
• Had a good structure/action plan to calculate the additional profit
• Correctly calculates the additional profit, but needed some assistance
Good:
• Completes all “Expected” requirements
• Correctly identifies the irrelevant data
• Could list at least 3 ideas to increase the number of passengers and 3 to increase the overall profitability
• Could keep the interviewer engaged during the calculations
Excellent:
• Completes all “Good“ requirements
• Showed some knowledge of this market

130 | P a g e
• Could list at least 5 ideas to increase the number of passengers and 5 to increase the overall profitability
• Could drive the case and solve it without much help.

Case 10 - Coffee Chain Tim Hortons to Begin Accepting Visa Cards

Consulting Firm: Oliver Wyman first round full time job interview.

Case Interview Question: Tim Hortons Inc. (known internationally as Tim Hortons Cafe and Bake Shop) is a Canadian
fast food restaurant known for its coffee and donuts. It is also Canada’s largest quick service
restaurant chain. As of June, 2013, Tim Hortons has more than 4,300 restaurants, including
3,400 in Canada, and 807 in the United States. The company is headquartered in Oakville,
Ontario, Canada.
The year is 2012. In Canada, Tim Hortons does not currently accept Visa cards. Should it?

Possible Answers:
1. Information Gathering
Additional Information: to give to candidates if requested
• There are over 3,400 Tim Hortons stores in Canada (but for this case we can analyze a single store)
• Tim Hortons currently accepts cash, MasterCard and debit card.
• There are no specific barriers preventing the use of Visa cards (such as an exclusive contract with MasterCard).
• Visa is the most common credit card in Canada with 45% of the market (vs. 30% for MasterCard).
• There are two variable costs associated with accepting Visa: (1) Visa charges Tim Hortons a 2% fee on sales, and (2) The
merchant services company (i.e. credit card processing equipment company) charges 1.5% plus $0.25 per transaction.
• Fixed costs: Set-up fee is $10,000 per store, annual maintenance and record keeping is $5,000 per store.
• MasterCard charges exactly the same as Visa. Debit card costs are very similar.
The following data can be provided only if asked for, or you can choose to have the interviewee make assumptions:
• The average Tim Hortons serves 500 customers per day and the average customer spends $5.00 per transaction.
• Current breakdown of payments: Cash and gift cards 40%, Debit card 30%, MasterCard 30%.
• Accepting Visa would decrease the percentage of cash, debit and MasterCard transactions.
• Tim Hortons averages a 50% gross margin on its products.
2. Suggested Structure
Ask questions to ensure there are no structural barriers. Then proceed using a cost-benefit analysis. The decision criterion is
that the client Tim Hortons should accept Visa cards if it will increase its profitability.
Accept Visa if: Benefits > Costs
a. Benefits
• Increased revenues
– ease of use
– customer satisfaction
b. Costs
• Fixed costs
– IT/equipment
– installation
– record keeping
• Variable costs
– Visa’s fee
– Merchant service fee
In this case it is relatively simple to determine the costs. It makes sense to calculate the costs and then perform a break-even
analysis to see how many new customers we would need to attract in order for it to be profitable.

131 | P a g e
3. Detailed Solution
a. Cost of implementing Visa:
• Fixed Costs: $15,000 in year one, $5,000 in subsequent years.
• Variable Costs: assume new breakdown of payments is: Cash 30%, debit card 25%, MasterCard 20%, and Visa 25%

Since the profit margins of MasterCard ≈ Visa ≈ Debit, we are only concerned about the decline in cash transactions. Cash
transactions have higher profit margins (by 3.5%).
Total Cost = (fixed costs) + ($0.25 * # of lost cash transactions) + (3.5% * lost cash revenue)

Variable Costs: Cash transactions decrease by 10% (from 40% to 30%).

• # of lost cash transactions = 10% of total transactions = 10% * (500 trans/day) * (365 days/year) = 18,250
• Transaction fees = ($0.25) * 18,250 = $4,563
• Lost cash revenue = (# of lost cash transactions) * (average revenue/transactions) = (18,250 transactions) *
($5/transaction) = $91,250
• 3.5% * $91,250 = $3,193.75
• Total Variable Costs = $4,563 + $3,193.75 = $7,756.75 ≈ $7,750
• Total Cost = $15,000 + $7,750 = $22,750 in the first year and $5,000 + $7,750 = $12,750 for each subsequent year.

b. Break-even Analysis: for year 2 and beyond


Breakeven point revenues = Total Cost/gross margin = $12,750/50% = $25,500
Breakeven point new customers = Revenues/revenue per transaction = $25,500/$5 = 5,100 new customers/year
Is this realistic? Currently a Tim Hortons store gets (500 * 365) = 182,500 customers per year. Tim Hortons would require
5,100/182,500 = ~3% more customers to breakeven. Would this happen?

Key Points:
• This is a “cost-benefit analysis” case: identify all new costs and all new revenues associated with implementing Visa.
• The actual numbers used do not matter. Make reasonable assumptions. The key is to identify the drivers of the costs and the
revenues.
• The variable costs come from shifting higher-margin cash sales to lower margin credit card sales.

4. Conclusion & Recommendation


You could argue either for or against implementing Visa – it depends on the numbers used. However, Tim Hortons’s has a
very differentiated product (coffee and donuts) which most customers “need” every morning. They will likely change their own
behaviours to suit what Tim Hortons’s provides for payments opportunities.
• If you argue for Visa, you must justify why Tim Hortons will get more customers by accepting Visa. You may also talk about
better service/more choice for the customer, or improvement in brand equity. You might also recommend that Tim Hortons
run a pilot in a store to see if it increases their profits.
• If you argue against Visa, you must highlight that Visa transactions are less profitable than cash. You may also note that Tim
Hortons is a differentiated product, average transaction amounts are small, and cash machines (ATMs) may be located within
stores.

132 | P a g e

You might also like